Step 3 studying

Pataasin ang iyong marka sa homework at exams ngayon gamit ang Quizwiz!

What adverse events is subclinical hypothyroidism (often caused by chronic Hashimoto's) associated with?

Recurrent Miscarriages severe preeclampsia preterm birth low birth weight placental abruption (Also, anti-TPO antibodies are associated with pregnancy loss even in women who are biochemically euthyroid)

When comparing multiple screening tests, what epidemiological parameter is most useful for comparing the significance of a negative and a positive result obtained in an individual, irregardless of prevalence of the disease?

Likelihood ratio (doesn't vary with prevalence) the probability of a test result occurring in a patient with the disorder compared to the same test result occurring in a patient without the disorder

What is the treatment for rhino-orbital-cerebral mucormycosis?

Liposomal Amphotericin B (given IV... they could have just said IV but they wanted to trick you) surgical debridement

If a pregnant woman goes to a picnic then has intrauterine fetal demins the next day after getting nausea and diarrhea, what is the most likely cause?

Listeria Monocytogenes -> granulomatous infantiseptica -> intrauterine fetal demise

What is the prognosis of Basal cell carcinoma?

Local invasion without distal metastasis (e.g. common on eyelid and can lead to blindness if untreated but will not metastasize elsewhere)

What is pulmonary cachexia syndrome?

Loss of lean muscle associated with chronic lung disease (namely COPD) A BMI <20% or weight loss of >5% suggests this and PCS is seen in 20-40% of patients Don't confuse it with MDD because even though they may meet the criteria for MDD they don't have it because old people can't get depressed apparently (also some symptoms of MDD are apparently caused by COPD, *fatigue and decreased appetite from increased muscle use and flattening of diaphragm which presses on stomach*)

If a patient comes in with normal pressure hydrocephalus, what is the next best step in treatment?

Lumbar drainage of CSF (do this first to do the lumbar tap test [Miller Fischer test] which removes 30-50 mL of CSF and determines if there is an improvement. If there is, proceed with definitive tx) definitive tx - ventriculoperitoneal shunting

A patient comes in with 2 week history of headache, malaise, nausea and blurry vision for a few weeks. He recently found out one of his sexual partners tested positive for HIV. Mild generalized lyphadenopathy is present. Screening HIV imunoassay is positive. RPR is positive and fluorescent treponemal antibody test is also positive. What is the next best step?

Lumbar puncture (He has neuro symptoms so regular treatment alone won't treat the problem, he has stage 2 syphillis) Don't test again, he has neurological sxs and 2 test confirmatory for syphillis (need 2 tests because HIV can cause false positives)

UWSA2 - A 50M comes in with syncope. He had prior episodes of lightheadedness and palpitations yesterday, then passed out at breakfast today. He recently had a rash on his R calf that self resolved. He loves bird watching and went for a trip in Maine. What is the cause of his syncope?

Lyme Carditis (needs IV ceftriaxone/cefotaxime/Penicillin G) appears in approximately 5% of people with lyme disease and AV block is the most common manifestation of it (prolonged PR and 2nd or 3rd degree AV block)

UWSA2 - what is Olecranon Bursitis and what does it look like?

Large, soft knob, or "goose egg," and redness from inflammation of olecranon bursa. Localized and easy to see because bursa lies just under skin. (Need to aspirate the bursa to rule out septic bursitis)

How do you treat a 4mm SCC on the left vocal chord with no evidence of metastasis?

Laser excision with CO2 laser radiotherapy (probably preserves voice better)

Shoulder pain in what locations supports rotator cuff tendonitis?

Lateral shoulder Anterior - acromioclavicular or glenohumoral joint osteoarthritis or biceps tendonitis Posterior - usually referred from cervical spine (nerve impingement)

A 3 y/o female patient comes in with a UTI and hx of constipation. She is vomiting and requires multiple bags of IVF as well as IV antibiotics. 2 days later her symptoms are resolved and she is discharged home. What should she be given to prevent recurrence?

Laxatives Apparently pediatric constipation causes enuresis and recurrent UTI's. (compressing the bladder and obstructing emptying)

If a patient develops Atrial fibrillation post cardiac surgery, what is the next best step?

Leave it alone it will probably resolve. A fib is a common complication after cardiac surgery

UWSA2 - A 21M comes in with lightheadedness while running. He runs a lot and says he may be pushing himself too hard. On EKG he has increased voltage of QRS, 0.5mm downsloping ST depressions and symmetric T-wave inversions in V4-V6. What si the most likely cause?

Left Ventricular Hypertrophy (could be from athlete's heart or could be from HOCM, so get an echo)

What aretery supplies the lateral wall of left ventricle?

Left circumflex aa.

If a 2month old girl is brought in with a strong FH of atopic dermatitis, what can you suggest to reduce her risk of developing atopic dermatitis in the future?

Let her go to daycare (Early exposure to nonpathogenic organisms [daycare, farm animals, dogs] lowers risk of developing atopic dermatitis)

What is the management of asymptomatic bacteruria in women >80 years old?

Let it go I guess, most are transient and resolve within 2 weeks

What is the initial drug used in treatment of Parkinsons Disease?

Levodopa (may hasten destruction of substantia nigra cells, so may not be 1st line especially in younger patients with mild disease) Pramipexole (dopamine agonist) bromocriptine (dopamine agonist)

What is the prognosis of ankylosing spondylitis?

Life expectancy is not reduced (apparently most people don't have functional or employment disabilities.... really?)

UWSA2 - treatment of stress urinary incontinence

Lifestyle mod (*weight loss, dietary changes*) Pelvic floor exercises (Kegels) Pessary Pelvic floor surgery: urethral sling

treatment of nausea and vomiting in pregnancy

Lifestyle modification, trigger avoidance > *vitamin B6, antihistamines, antiemetics* > oral dopamine and serotonin antagonists > IV fluids and antiemetics > Corticosteroids > TPN or tube feedings (refractory)

If someone has an accidental ingestion, what symptom would you see if it was antifreeze? (ethylene glycol)

Rapid and deep breathing (hyperventillation) leads to metabolic acidosis which your body will want to compensate for with resp alkalosis

Atrial Flutter ECG and Treatment

Rate control: -Beta blockers -CCBs -Digoxin Conversion to sinus: -Ibutilide -Electrical cardioverson Anticoagulation (warfarin) If chronic: -Amiodarone -Dofetilide

UWSA2 - A 34M comes in with acute rhinosinusitis (ARS) with purulent discharge for past 4 days. What is next best step?

Reassurance (Apparently purulent discharge in ARS can't differentiate viral/bacterial. And either one typically resolves on it's own anyway. Only consider Abx if it lasts more than 7-10 days or worsens after initial improvement)

A 32 y/o G3P2 mother 10 weeks gestation comes in with nausea and anxiety. Her TSH is 0.12 (N 0.4-4) and T4 is 15 (N 4-12). What is the next best step?

Reassurance Apparently you never do any follow up tests ever according to UW. TSH is low because hCG directly stimulates TSH receptors while T4 is high because of high TBG. Don't ever order a free T4 ever or you deserve to die.

A 25yo F presents at 29w pregnant. When she was 10w pregnant she had platelets of 225k, and today they are 100k. What is the next best step for this patient?

Reassurance and observation gestational thrombocytopenia is 100-150k platelets and it is benign Preeclampsia is <100k and would need a 24h urine collection others in chart

What test can confirm Hirchsprung's disease in a newborn?

Rectal mucosal suction biopsy (fid absence of ganglion cells)

NBME5 - What changes, if any, should be made to medications in someone taking PTU for Graves' disease once TSH and T4 levels have stabilized?

Reduce dose of PTU by 30-50% (If it isn't decreased secondary hypothyroidsim may develop) (Note- 'stabilized' means T4 of 5.5 and TSH of 9.8 apparently)

what is standard procedure for perioperative medication management of the following drugs: B-blockers, Alpha 2 antagonists, CCB's, ACE inhibitors, diuretics, statins, SERMs?

SERMs (raloxifene) should be discontinued before surgery as they increase risk of venous thrombembolism

What medication can be used to prevent hot flashes in a woman going through menopause who has to stop combination hormone replacement therapy due to development of a DVT?

SSRI's (escitalopram) and SNRI's (venlafaxine) apparently can stop hot flashes in 50-70% of women?...

Medical treatment for Bulimia?

SSRIs

What is the treatment for Premenstrual syndrome and Premenstrual dysmorphic disorder?

SSRIs (if one fails switch to another as any single SSRI can fail)

What is late-life depression (>65yo onset) highly associated with?

Major neurocognitive disorder due to Alzheimer's dementia

What are the main side effects of valrpoic acid you should remember?

Pancreatitis liver injury thrombocytopenia teratogen

UWSA1 - If someone has a liver cyst that is tested and is "free of microbes" can it still be protozoal?

Yes, in fact it probably is...

NBME5 - Can you determine Cystic Fibrosis diagnosis in a child in utero?

Yes, it is tested with amniocentesis

Left Bundle Branch Block ECG

"LBBB" a block in the electrical conduction through the left bundle; evidenced by a wide QRS in lead 1 and V6

contraception side effects

"OCP's don't cause weight gain only breakthrough bleeding" QID 5811

UWSA2 - What advice should you give someone who recovered from acute angle glaucoma after treatment in hospital?

"You can't use certain over the counter cough and cold medicine anymore" -referring to decongestants (pseudofedrine) and anticholinergics

What does the Ixodes tick look like and what does it transmit?

"brown" -UW 2021 Lyme disease (Boriella burgdorferi)

After surgery a patient with cirrhosis develops asterixis and limited consciousness (likely hepatic encephalopathy). What is the most likely cause?

"decreased gut peristalsis" or constipation from opiod use leads to decreased ammonia clearance

UWSA1 - What is patellofemoral pain syndrome and how is it treated?

"runner's knee", which refers to anterior knee pain that worsens while going up or down stairs, walking hills, or prolonged sitting with knees flexed Tx - reduce exercise intensity, activity modification, short course of NSAIDs + *Stretching/strengthening exercises*

What is Pemphigoid gestationis and what does it look like?

'herpes gestationis' * pruritic during pregnancy & postpartum period * vesicular eruption on abdomen w/ sparing of mucous membranes *occurs 4th-7th mo. *white females tx - topical triamcinolone

UWSA2 - What is the treatment for developmental dysplasia of the hip?

(*Consult orthopedic surgeon*, no imaging needed) *if identified within 6 months can use a Pavlik harness* (keeps hip in flexed, abducted position). If treated early there are *no lasting sequelae*

(1) purulent cervical dischage and friable cervix vs. (2) thin grey cervical discharge and inflamed cervix vs (3) green discharge with punctate, friable cervix. What are the diagnoses and treatments?

(1) chlamydia +/- gonorrhea - doxycycline + ceftriaxone (2) bacterial vaginosis - metronidazole (3) trichomoniasis - metronidazole

Paget's disease of bone

(Osteitis Deformans) Skeletal disease of increase bone resorption and formation, which softens, thickens, and deforms bone. Characterized by bowed long bones, sudden fractures, frontal bossing, and enlarging skull bones that forma an acorn-shaped cranium. (*thickening of outer cortex + mild bowing, sclerotic lesions*) technetium bone scan may show increased uptake as well Can cause: headache, vertigo, tinnitus, progressive deafness, and optic atrophy and compression of the spinal cord. Tx - bisphosphonates (or calcitonin if BPs not tolerated)

What metabolic conditions cause hepatic encephalopathy?

(NOTE: trea these first and your HE will resolve apparently) hypovolemia *hypokalemia* - facilitates conversion of ammonium to ammonia metabolic alkalosis - decreases urinary ammonia excretion

UWSA2 - How do you differentiate between croup (laryngotracheitis, parainfluenza virus) and pertussis in an unvaccinated 15 month Male child?

(They both have "barking" or "whooping" cough which is always described as harsh) croup - *acute onset* "barking" cough, hoarseness, stridor, narrowing of subglottic space (steeple sign) pertussis - *longer prodroe (10-14 days)* followed by weeks of increasingly severe cough

UWSA1 - A patient is asking for preventative medications for gout, what should you give them?

1st line Xanthine Oxidase inhibitors (Allopurinol, febuxistat) OR Uricosuric drugs (probenecid) THENNNN *low dose colchicine* apparently don't give 2 different 1st line drugs because Allopurinol + Probenecid can cause a rapid drop in Uric acid and precipitate another attack

A patient presents with hypoglycemia. He has a presentation similar to insulinoma. What should you do?

(normally an abdominal CT scan) but UW says oral hypoglycemic agent screen to rule out an insulin secretagogue

UWSA2 - What does folliculitis look like?

(normally from pools or something with pseudomonas)

What is the treatment for Tourrette Syndrome?

*1st or second gen antipsychotics*, although 2nd are more preferred due to side effect profile of 1st if they have concominant OCD, impulse control problems and rage attacks, also add an *SSRI*

When are dTap vaccinations given in children?

2, 4 and 6 months and then 4 years

What is the treatment for Hidradenitis Suppurativa?

*All patients:* - weight loss, smoking cessation, clean area daily *Hurley Stage 1 (mild disease):* -topical clindamycin -intralesional steroids or oral abx for flare ups *Hurley Stage 2 (nodules, sinus tracts and scarring)* -oral tetracycline (Doxycycline) -oral clinda + rifampin if refractory *Hurley Stage 3 (diffuse involvement and extensive sinus tracts)* -TNF-alpha inhibitors (eg infliximab) -oral retinoids (acitretin) -surgical excision

What are the main risk factors for C. diff?

*Antibiotic use* *Gastric acid suppression* Hospitalization Advanced age (>65)

What is the first line treatment for violence/agitation during PCP intoxication?

*BENZOOOOOOOOOOOS* (not haloperidol like most people thought it was but it's only adjunctive)

What causes an increased risk of breast cancer in men?

*BRCA2 are the single greatest factor* (although all BRCA mutations increase risk) Klinefelter syndrome (IN THE PATIENT NOT FAMILY) -hepatic dysfunction -marijuana use -obesity

A 5M comes in with reddness and thick white discharge around glans penis with the foreskin retracted. In addition to teaching proper foreskin hygiene, what else should be done?

*Blood glucose test* (and abx of antifungals but that wasn't the right answer - topical mupirocin for bacteria, topical nystatin for candida) He has balantis/balanoposthitis which is essentially a UTI that spreads to the foreskin as well. Commonly see in uncircumcised kids and often C. albica. Can be a presenting feature of type I DM in children.

What bacteria do you get from human bites?

*Eikenella corrodens* (gram -) also: alpha hemolytic strep - strep viridans Staph aureus anaerobes (e.g. fusobacterium, prevotella)

Breast discharge evaluation

Bilateral discharge -> Pregnancy test or galactorrhea eval Unilateral discharge -> Age >30 -> Mammogram + US Age <30 -> Mammogram +/- US

What is a Desmoid tumor and what is it associated with?

A tumor of fibroblasts that can be superficial or intra-abdominal. If superficial can be deeply seated painless or painful masses. -If present with multiple colonic polyps suggests *Gardner's syndrome* (variant of Familial Adenomatous Polyposis syndrome).

Kawasaki's disease diagnostic criteria (5 of them)

*Fever >5 days* + >4 of the following -conjunctivitis (bilateral, nonexudative) -mucositis (strawberry tongue, injected/fissured lips/pharynx) -cervical lymphadenopathy -Rash (perineal erythema/desquamation; polymorphous, generalized) -erythema/edema of hands/feet, periungual desquamation

Aside from hydops fetalis and fetal demise, what else can Parvovirus B19 do to a fetus in utero?

*Generalized fluid accumulation that can be monitored with US* And anemia I guess

What is the risk of licorice?

*Hypertension* Hypokalemia

When should you place a cerclage in a woman with cervical insufficiency?

*If she has had >2 painless 2nd trimester losses* Or if she has painless cervical dilation (Apparently you have to wait for 2 abortions to place a cerclage and if she has no history of these abortions you do weekly US cervical length measurements with vaginal progesterone supplementation)

If a patient is started on a single medication diabetic regimen and suddenly they have many episodes of hyper/hypoglycemia (fluctuating BG levels), what is the most likely medication?

*Insulin secretagogues* Sulfonylureas (glyburide, glypizide, glimipiride, etc.) Meglitinides (nateglinide, rapaglinide, etc.) QID 11399 only 20% correct lol

In a patient with pernicious anemia, what is the most likely thing you'll see on endoscopy?

Absent rugae in the fundus (it causes autoimmune metaplastic atrophic gastritis which causes glandular atrophy [especially of the fundus], intestinal metaplasia and inflammation) QID 6158, 27% got it right

What drug is typically used in a person trying to quit drinking?

Acamprosate *Naltrexone* - apparently it decreases cravings

What changes should be made in pregnancy and after birth for a woman with HIV who is compliant with triple therapy and pregnant with 14 weeks gestation?

No changes to medication *infant should not be breastfed*

A 5 week old child is diagnosed with hypertrophic pyloric stenosis. The mother remembers that he was given antibiotics at 2 weeks old when her 2 year old daughter was sick. What antibiotics was he given and what were they prophylaxis for?

*Macrolides (azithromycin and erythromycin most commonly)* are commonly perscribed for postexposure prophylaxis for *pertussis* These drugs increase the risk of hypertrophic pyloric stenosis if given to children in the first 2 weeks of life Other RF for HPS are being bottle fed, and being a firstborn male child

Nonallergic Rhinitis vs Allergic Rhinitis

According to UW, if they are >20y/o they cannot ever ever ever ever ever have allergic rhinitis

NBME5 - What does herpangina look like and how is it diagnosed?

No diagnostic step needed, it's clinical and self-limiting, just give antipyretics (hand foot mouth disease/coxsackie B virus causes it)

NBME5 - What does Eczema herpeticum look like?

No diagnostic test necessary

What advice should you give a 75M who says he is afraid of quitting smoking now since he "doesn't have long to life left anyway" even though he hasn't experienced many symptoms?

*Mortality risk will fall below current smokers in about 5 years* (Also, may see initial increase in cough in first few weeks, then a decrease. The symptoms of withdrawal will be similar to younger quitters and effect on lungs will be less than younger quitters)

What is the main notable symptoms of Parinaud syndrome and what causes the syndrome?

Paralysis of conjugate vertical gaze due to lesion in superior colliculi (e.g., pinealoma)

UWSA2 - A 64F comes in with no complaints for a check-up. On exam she has mild peripheral field visual loss, enlargement of optic cups, and increased cup:disk ratio bilaterally, but normal pressures in the eyes. What is the most likely cause?

Acute angle closure glaucoma Apparently up to 50% of patients have no increased ocular pressure

UWSA1 - How do you differentiate UTI with cystitis and Acute bacterial prostatitis?

Acute bacterial prostatitis often has perianal discomfort and bladder obstruction. On Digital rectal exam you'll find prostatic warmth, edema and tenderness

When is excercise contraindicated in pregnant patients?

*Patients at risk for preterm delivery* -cervical insufficiency (even with cerclage) -preterm labor during pregnancy -preterm prelabor rupture of membranes *Patients at risk for antepartum bleeding* -placenta previa -persistent 2nd/3rd trimester bleeding *Patients w/ underlying condition exacerbated by excercise* -Severe anemia -HTN disorders of pregnancy (e.g. preeclampsia) -restrictive lung diseases -Severe heart distress

UWSA2 - What are the precipitating factors for hepatic encephalopathy?

-infection -GI bleed -metabolic abnormalities (Hypokalemia/natremia) -hypovolemia -*sedative use (Benzos, opiods?, narcotics)*

What is used to reverse warfarin anticoagulation in an emergency situation (intracranial hemorrhage)?

*Prothrombin complex concentrate* - contains vit K dependent clotting factors and normalizes INR in <10 minutes while FFP needs >2L, blood compatibilitys tests and takes more time... (absolutely F*** UW on this, 60% of people chose fresh frozen plasma because that's what it always is...)

In a patient with HIV and PCP with respiratory decompensation, why would you use corticosteroids?

Alveolar-arterial oxygen gradient >/= 35 mmHg on room air organism lysis stimulates an inflammatory response

What is the presentation of orthostatic proteinuria, and how is it diagnosed?

- protein excretion that increases during the day but decreases at night during recumbency (most common proteinuria in adolescents) - Dx with *split urine collection* (collect during day + during night, and compare) - 8 hr nighttime collection showing *<50%* of daytime protein = essential for Dx - benign

What is first and second line management for acute bacterial rhinosinusitis?

1st - amoxicillin-clavulaunate 2nd - doxycycline or fluoroquinolone

What antihypertensive drug should be given to a patient who has bipolar for 30 years and is being treated successfully with Lithium during that time?

Amplodipine

What are symptoms normally seen in a pineal gland tumor?

Pariaud's syndrome -*vertical gaze paralysis* -loss of pupillary reaction -*loss of optokinetic nystagmus* -ataxia

If a 34F comes in with a painless breast mass for the past 3 weeks what should you do next?

Mammogram (QID 4952 45% chose US, 32% chose Mammo. Apparently once a woman is over 30 you don't do US first because their breast tissue is too dense)

If you suspect a patient has fibromyalgia, what lab test should you do next?

*Thyroid studies?* (to rule out hypothyroidism? weird) CBC (r/o anemia) ESR (r/o inflammatory arthropathy)

What severe complication is associated with diptheria?

*Toxin-mediated Myocarditis* neuritis kidney disease

UWSA1 - how does PEG tube placement affect survival in advanced dementia patients?

No effect on survival, but does slightly decrease risk of aspiration pneumonia (Does not prevent it though)

If left untreated, how does developmental dysplasia of the hip present?

*abnormal acetabular development resulting in shallow hip socket and inadequate support for femoral head* -leg-length discrepncy -gait abnormalities (toe walking on affected side, Trendelenburg gait [pelvic drop standing on one leg]) -premature osteoarthritis -activity related pain in hip and groin -clicking related to activity

How can you reduce the risk of Alzheimer's in a patient with family history of the disease?

*aggressive treatment of cardiovascular risk factors, especially in mid-life (HTN, diabetes, weight loss)* cognitively stimulating activities and maintaining social relationships

strychnine poisoning

*antagonizes glycine* which is found in CNS; blocks Renshaw cell activity which allows uncontrolled and diffuse spinal reflex activity; onset is 10minutes to 2 hours post ingestion; CS nervousness, "sawhorse" stance; seizures (*episodic contractions similar to tonic-clonic in a fully awake patient*) TX is diazepam for seizures; body gets rid of it in 12-24 hours; keep temp normal and keep them breathing and they will live; may give sedative to help relax

What signs are there for phenytoin toxicity?

*first one is horizontal gaze nystagmus* then blurred vision, diplopia, ataxia, slurred speech, dizziness, drowsiness, lethargy, decreased mentation and eventual coma

What is the presentation of Analgesic nephropathy?

*long-term* use of salicylates, acetaminophen, NSAIDs causing chronic tubular injury hematuria, proteinuria, pyuria, but *sterile UCx* (looks a lot like nephritic syndrome but f*** you)

What complication can an untreated scaphoid fracture lead to?

*non-union* and avascular necrosis (not malunion with wrist deformity)

What is the pathophysiological cause of Non-alcoholic Fatty Liver Disease?

*peripheral insulin resistance* -> inc peripheral lipolysis, triglyceride synthesis and hepatic uptake of FFA -> inc oxidative stress and production of pro-inflammatory cytokines also.....inc transport of free fatty acids from adipose to liver, dec liver oxidation of FFA, or dec clearance of FFA from liver

If a patient has a toxic thyroid nodule (2.8 cm) and symptomatic relief from beta blockers, what is the next best course of action?

*pretreatment with methimazole* then consider surgery or radioactive iodine ablation Note: The answer will never be radioactive iodine ablation. This is like my 5th question wrong where I put RIA and it's never even the second choice. It will never be RIA

UWSA2 - Vestibular neuritis/labyrinthitis

*sudden onset severe vertigo w/ n/v x 1-2 days*; gradual lessening symptoms; probs w/ walking/balance. Termed ladyrynthitis when associated with unilateral hearing loss caused by viral infection Tx: Methylprednisone taper, Meclizine, Scopolamine, Compazine, Ondansetron (ddx Meniere's and BPPV [more chronic and repeating],

UWSA1 - Indications for AAA repair?

- >5.5 cm (or twice the normal value) - Rate of expansion >0.5 cm in 6 months or 1 cm in 1 year - Symptoms (abd, back, or flank pain; limb ischemia) If it doesn't meet these criteria reevaluate with US or CT every 6 months - 2 years depending on size

UWSA2 - MDD with psychotic features - treatment

- antidepressant plus an antipsychotic OR - ECT How decide between the two --> see how urgent the situation is... ECT is much faster acting (*NOTE - urgent situation could be suicidal ideation or refusing to eat/drink*)

Gastroparesis treatment

- lifestyle modifications (Frequent small-volume meals) - pro-motility agents (metoclopramide, erythromycin) - gastric pacemaker in severe cases

What should be done for prenatal care of someone with sickle cell disease on their first visit?

-*Baseline 24 hour urine for total protein* -baseline chem panel -serial urine cx -pneumococcal vaccination -folic acid supplement -aspirin serial fetal growth US (Not iron because they generally have iron overload from frequent transfusions)

NBME5 - cerebral palsy risk factors

-*Prematurity (biggest increase with 80x higher risk)* -*Low birth weight* -Intrauterine hypoxia -Intrauterine bleeding in pregnancy -Infection/toxin during pregnancy -Obstetric complications -Neonatal hypoglycemia

UWSA2 - Complications of Obstructive Sleep Apnea

-*Resistant HTN* -CAD -*Cardiac arrhythmias (a fib, bradycardia, ventricular ectopy) -Heart failure

What are red flags for pathologic causes of scoliosis that require further evaluation?

-Back Pain -neurologic symptoms -rapidly progressive curvature (>10 degrees/year) -vertebral anomalies on XR

What is the long term management for someone with Multiple Sclerosis?

-Beta-interferon -glatiramer-acetate

1 hour post abdominal surgery, a 63M patient develops SOB, CP and hypotension. He has an anterior wall MI on ECG. He also has bradycardia of 32/min. What are the next best steps?

1st - give IV saline and atropine to help with bradyarrythmia 2nd - if atropine doesn't work -> *temporary cardiac pacing* to stabilize him for PCI (You may need heparin in PCI, even though it's CI in patients with abdominal surgeries within 2 weeks, but you need to stabilize the patient first)

What is the cause of and presentation of Scombroid poisoning?

-Caused by consumption of *dark-meat fish* (e.g., bonito, mackerel, mahi-mahi, tuna) improperly stored at warm temperature. -Bacterial histidine decarboxylase converts histidine to histamine. *Histamine* is not degraded by cooking. -Acute-onset *burning sensation of the mouth, flushing of face, erythema, urticaria, pruritus, headache*. May cause *anaphylaxis-like presentation* (i.e., bronchospasm, angioedema, hypotension). -*Frequently misdiagnosed as allergy* to fish. -Treat supportively with *antihistamines; if needed, antianaphylactics* (e.g., bronchodilators, epinephrine).

What are some risk factors for pediatric dental carries?

-Diet -frequent sugary snacks/beverages -nighttime bottle or feedings -inadequate fluoride -Family: carries -Social: low socioeconomic status

UWSA2 - What do you see on CXR in a patient with Silicosis (Working in foundry work, quarrying, ceramics, glass work, and sandblasting)?

-Diffuse, small round opacities throughout lung -Eggshell calcification (calcified hilar lymph nodes)

Adverse effects of SGLT-2 inhibitors

-Genital mycotic infections (i.e. vulvovaginal candidiasis) -UTI -Osmotic diuresis (acutely) which may cause *hypotension* -*Hyperkalemia* -Increased LDL

UWSA1 - What is the definition of very low risk prostate cancer (unlikely to die in 10 years and can deal with surveillance rather than treatment... sounds dumb I know)?

-Gleason score of <6 with <3 cores (<50% of involvement with each core) -normal digital rectal exam -PSA <10ng/mL

UWSA2 - What is the management of Acute splenic sequestration seen in a patient with sickle cell disease?

-Initially replete volume with isotonic crystalloid fluids -Then transfuse with RBC (even if their RBC on CBC is 5.4, it's probably just because their RBCs are sequestered in the spleen) -then +/- splenectomy

What ECG finding necessitate further management in a patient who recently had coronary revascularization and now has signs of L heart failure?

-LVEF <35% -NYHA class II, III, IV heart failure sx (presence of any sx) -*LBBB with QRS >150msec*

PAD treatment

-Lifestyle modification (weight loss, smoking cessation, exercise) -Phosphodiesterase inhibitors (cilostazol) -Antiplatelet agents (aspirin) -Angioplasty and stenting -Vascular surgery Bypass

What are the risk facotrs of dupuytren contracture?

-Male, Age >50, family hx -*Diabetes melitus* -tobacco and alcohol use

What are normal recommendations for prostate cancer screening?

-Not recommended below age 55 -from 55-69 consider screening with PSA -Over 70 or life expectancy <10 years, don't bother (Question was a 55M and the answer was "Most men with prostate cancer die of other things"... I'm really not sure what they want anymore)

What is the biggest predictor of severity in a marker for acute pancreatitis?

-Older age -obesity -hematocrit (>44% -> severe [hemoconcentration from third spaces]) -CRP (>150 mg/dL 24-48h after admission -> severe) -*BUN* (>20 mg/dL on admission -> greater risk of death)

UWSA2 - An 11day M has jaundice on his face. What signs would lead you to investigate this jaundice further?

-Poor weight gain (not returning to birth weight yet) -inadequate elimination (<6 stools/day) -signs of dehydration (dry mucous membranes, lethargy)

UWSA2 - What are the physical exam findings of developmental dysplasia of the hip?

-Positive Barlow & Ortolani maneuvers (clunk when maneuvering hip) -Asymmetrical thigh/inguinal/gluteal folds -*apparent leg-length discrepancy* (during Galeazzi test where patient is supine with knees flexed)

UWSA2 - When do you need to get abdominal imaging for a patient with C diff. compared to just treating the disease?

-Severe (leukocytes >15,000 or Creatiine >1.5) or fulminant (septic shock, apparent ileus) disease -patients with severe abdominal pain (worried about toxic megacolon)

UWSA2 - What is it most important to ask for in a mother when screening for post partum blues vs PP depression?

-Their feelings towards the new infant (inability to enjoy new baby suggest depression) -(mood?)

severe aortic stenosis sounds

-a soft, single second heart sound -delayed and diminished carotid pulse (parvus et tardus) -loud and late peaking systolic murmur

What post-exposure prophylaxis should be initiated for sexual assault?

-azithromycin (chlamydia) -ceftriaxone (gonorrhea) -metronidazole (trichamoniasis) -tenofivir-emtricitabine with raltegravir (HIV)

NBME5 - Verapimil adverse effects

-bradycardia -*heart block* -impaired cardiac output

What is the management of aspiration pneumonia from soneone who was choking on food?

-clindamycin OR -combination beta-lactam + beta-lactamase inhibitor

Meralgia Paresthetica

-compression of the *lateral femoral cutaneous nerve * -pain & tingling sensation but *no motor loss* -cause: obesity, postural changes, tight clothing, pregnancy Tx - weight loss, avoid tight fitting clothes

What is the presentation of cerebral palsy in an infant and how is it diagnosed?

-delayed dissappearance of neonatal reflexes -persistant assymetric fisting -*early hand preference (before 1)* In Spastic CP (most common type) -hypertonia in first few months (with contractures or scissoring of legs) -hyperreflexia and sustained clonus -delayed motor milestones *Dx - MRI of the brain*

NBME5 - What are the suspicious symptoms of back pain that would make you want to investigate it further?

-fever -focal midline tenderness -weight loss -pain increased at night -focal neurologic symptoms -urinary incontinence/retention

NBME5 - How do you differentiate between alcohol and antifreeze intoxication?

Antifreeze -> anion gap metabolic acidosis Ethanol -> non-anion gap MA

How would you prevent Hepatitis B in an infant who's mother was infected with Hep B and had an active infection perinatally/third trimester?

-maternal antiviral in some cases -Newborn Hep B immunoglobulin and vaccine w/in 12 hours -routine immunization (1-2 months? and 6-18 months?) -serology ~3 mo after 3rd vaccine dose reason - infants remain immunologically negative or with mild LFT elevation as well as asymptomatic. *Serology at 9 months (usually) will allow you to detect HBsAg* which if neg means not infected (although if infected they have higher risk of chronic infection and usually have high viral load and HBeAg)

Describe how a minor thalassemia can affect iron studies and blood smears in a patient.

-microcytic, hypochromic anemia with *dec MCV and Mean corpuscular Hg* -*mildly elevated ferritin* from high RBC turnover -*RBC normal or increased* -*NORMAL red cell distribution width* as all cells are uniformly small (*iron def has increased RDW*)

UWSA2 - What is the presentation of haemophilus ducreyi?

-painful (tender = painful) -can be multiple ulcers -Large, deep ulcers with gray/yellow exudate -well demarcated, with soft, friable base -severe lymphadenopathy (can be suppurative and painful) -subsaharan africa, latin america and southest asia

UWSA2 - What is the presentation of Barotrauma in the lungs from NPPV or ventilation?

-pneumothorax -pneumoperitoneum -pneumomediastinum -*subcutaneous emphysema (sudden, spontaneous soft tissue swelling in upper chest, neck and/or face with crepitus. Usually self-resolves)*

When should you prescribe bisphosphonates for osteoporosis & osteopenia?

-pts. with low bone mass and history of fragility fracture -T score <-2.5 on DXA scan (criteria for osteoporosis) -Osteopenia (T score b/t -1 and -2.5) with a 10 year probability of major osteoporotic fracture >20% or hip fracture >3% based on FRAX calculator

ventricular tachycardia management

-stable sustained VT: antiarrhythmics (amiodarone, lidocaine, procainamide) -unstable VT with a pulse: synchronized cardioversion -VT (no pulse): defibrillation (unsynchronized cardioversion) + CPR (treat as ventricular fib)

UWSA1 - What are the most common migraine prevention meds?

-topiramate (Topamax), -beta-adrengergic blockers (e.g., propranolol [Inderal], atenolol [Tenormin]) -*tricyclic antidepressants (e.g., amitriptylin [Elavil])* -selective serotonin reuptake inhibitors (e.g., fluoxetine [Prozac]) -calcium channel blockers (e.g., verapamil [Isoptin])

What does Fetal Postmaturity Syndrome look like?

.-Small for gestational age -wrinkled, peeling skin -long thin body -low subQ fat -long fingernails -sparse lanugo hair -more scalp hair -meconium stained placenta (Green placenta, green skin baby) (Caused by post term placental insufficiency - late decels from placental dysfunction and fetal HR abnormalities)

UWSA1 - What is an early sign of Parkinson's that can suggest the diagnosis?

Asymmetric hand rigidity (Apparently PD symptoms are generally asymmetric at onset and stay asymmertic during progression)

standard deviation

1 - 68 (34% per side in, 16% per side out) 2 - 95 (47.5% per side in, 2.5% per side out) 3 - 99.7 (49.85% per side in, 0.15% per side out)

What is the management of barrett's esophagus based on the findings of initial endoscopy and biopsy? 1) no dysplasia 2) low grade dysplasia 3) high grade dysplasia

1) PPI plus endoscopy in 3-5 years 2) PPI + endoscopy in 6-12 months OR endoscopic eradication 3) endoscopic eradication therapy never do surgery unless can't do endoscopic ablation

What are the three instances where a person would have isolated Anti-HBc (total) positive with negative everything else for Hep B?

1) Window period of acute Hep B infection (IgM elevated and LFTs increased) 2) years after recovery from acute hep B infection (LFTs normal and IgM negative) 3) Many years after chronic Hep B infection when HBsAg has fallen to low levels (IgM will be negative but chronic liver disease will be detected on LFTs/US) *Measure IgM anti-HBc and LFTs to differentiate them*

Prostatitis Treatment

1) ciprofloxacin or TMP/SMX for an extended period of time. 2 Wks for acute; 6 wks for chronic Prostatitis is like an abscess. Use the same drugs as for cystitis and pyelonephritis but extend the length of therapy.

How long do you need to wait before surgery if a patient is on rivaroxiban?

1-3 days, no bridging required

UWSA2 - A 30F comes in at 36w gestation. She is developing symptoms of L sided heart failure. Her BP is 118/64. What should you do to manage her?

1st - loop diuretics, then add 2nd - *nitrates and direct acting vasodilator (hydralzine) to lower preload/afterload* 3rd - once acute decompensation treated, add a BB (NOTE: if the above all fails, then consider digoxin)

NBME5 - If an X-ray shows spondylolithesis what is the next best step in the patient for imaging?

No further iaging needed unless there are radiculopathy, myelopathy or lingering sxs

Kussmaul sign

1. Increase in JVP on inhalation 2. Associated with pericarditis / restrictive cardiomyopathy (not cardiac tamponade)

Amiodarone Mechanism of Action

1. Prolongs duration of action potential and refractory period without significantly effecting the resting potential 2. Relaxes smooth muscles causing vasodilation (especially in the coronary arteries) 3. Raises the threshold for VFIB

What is Euglycemic Diabetic Ketoacidosis and what causes it?

1. safety concern for *ALL SGLT2 inhibitors* -Caniglaflozin, Dapagliflozin, Empagliflozin, etc 2. patients have unexpected Sx: n/v/SOB/malaise 3. def: urine ketones but BG is not as high as expected (<250mg/dL normally)

when is reconstruction of cleft lip performed on a baby?

10 weeks The rule of 10 - 10 lbs, 10 weeks, 10g of Hg

When should you do a cell free DNA screen on a pregnant woman?

10 weeks or later (Checks trisomies and aneuplodies. For some reason I thought it was 15 weeks)

Management of Heart Failure

1st ACEi/ARB if ACEi not tolerated 2nd Diuretic therapy, Beta blockers (with EF <40% once euvolemic), Spirinolactone (with EF <35% and stable renal f(x) and K+) 3rd Isosorbide dinitrate/hydralazine (if african american), Digoxin (if symptomatic with spirinolactone), cardiac resynchroniation if QRS <150 4th Transplant/ventricular assistance device

What is the treatment of acute gout?

1st line - NSAIDs (indomethacin) -if there are contraindications then 2nd line - colchicine -if there are CIs then and <2 joints consider oral, IV, IM or intraarticular corticosteroids -if >2 joints consider coral, IV, IM of intramuscular corticosteroids

What is the treatment for torticollis (sustained contractions of muscles from dystonic rections/extrapyrimidal effects)?

1st line - diphenhydramine or benztropine

How do you treat onychomycosis?

1st line - terbinafine (oral), itraconazole (oral) 2nd line - griseofulvan, fluconazole, ciclopirox

Treatment of Graves in pregnancy?

1st trimester - Propylthiouracul 2nd and 3rd trimester - Methimazole

What is Nelson's syndrome and what causes it?

1° Adrenal Cushings → surgical removal of adrenals → loss of negative feedback to pituitary → Pituitary Adenoma

What is the treatment for tubercular meningitis?

2 months of 4 drug therapy -rifampin -isoniazid -pyramidazole -either fluoroquinalone or injectible aminoglycoside *Also, 8 weeks of glucocorticoids (dexamethazone or prednisone) which reduces morbidity and mortality significantly* then 9-12 months of isoniazid + rifampin

How long does salicylic acid take to treat plantar warts?

2-3 weeks for results to become apparent, then check every 2-3 weeks, and continues for 1-2 weeks after

If you suspect pheochromocytoma, what test should you use to confirm your suspicious? If that test is positive what is the next best step in management?

24 hour urine metanpeherine (more sensitive than vanillylmandelic acid excretion) next test is abdominal MRI

How long does it normally take until you can have sex again after recovering from an MI?

3-4 weeks

before starting alendronate for a postmenopausal woman what test should be run?

25-hydroxyvitamin D levels Not for any particular reason but before meds a postmenopausal osteoporotic woman should have CBC, electrolytes, 25-hydroxyvitamin D and DXA scan (even though the last is always for diagnosis)

How do you treat active TB in a pregnant woman?

3 drug therapy + pyridoxine (B6) supplementation Isoniazid, rifampin and ethambutol for 2 months, then isoniazid and rifampin for 7 more months (Pyrazinamide is teratogenic)

What is the standard feed from a G tube for a hospitalized patient?

30kcal/kg/day with 1g/kg/day of protein

If a 37M patient's father had colon cancer at 53, when should you offer them their first screening colonoscopy?

40 Apparently if they are at increased risk is is 10 years before their parent who had it OR 40 y/o, whichever is *EARLIER* 53% of people chose 43, 35% said 40 sooooo....

When does ALS normally present in a patient?

40-70 years old So if a 62 y/o comes in with tongue fasciculations and other suspicious stuff it's ALS and not a stroke or anything

UWSA2 - What is Zolendronic Acid (Reclast)?

Bisphosphonate

UWSA1 - Until what age can an umbilical hernia repair be delayed in a newborn patient?

5 years (normally close spontaneously in first years of life)

How long should you take antidepressants after your first episode of MDD?

6 months

What is the treatment for a mild acute asthma exacerbation (>20% reduction in peak expiratory flow rate + more asthma sxs, where mod/severe is >50% or diff speaking or accessory muscle use or hypoxemia) lasting 3 days after a viral URI? (pt normally on as needed SABA)

7 days of oral corticosteroids if mod/severe transfer to ED

NBME5 - What is a normal number of URIs for a child (5) to get each year?

7-9? (I just learned I was never a child apparently)

UWSA2 - What H of urine would suggest a struvite stone rather than a calcium stone?

>7 pH

What is an abnormal Mini mental State Exam (MMSE)?

<24

What is the management of supratherapeutic warfarin (measured by INR levels) in a patient with known history of clotting disorders?

<4.5 INR stop warfarin for 1-2 days or dec. dose 4.5-10 INR hold warfarin and restart when INR therapeutic, give low dose oral vitamin K (1-2.5mg) if there's an inc risk of bleeding >10 INR hold warfarin and give high dose (2.5-5mg) oral vit K If bleeding give fresh frozen plasma and IV 10mg Vitamin K

UWSA2 - What bilirubin level is considered normal in the first 24 hours after birth?

<5

What is the Mentzer index? What is it used for?

= MCV/RBC if <13 in thalassemia if >13 in iron deficiency

Standardized incidence Ratio

=observed cases/expected cases determines if occurrence of ds in sample is high/low relative to exp value from large pop

What level of BUN causes altered mental status? (Note, not true in real life by my experience)

>50

How many days of mania are needed for the criteria of a manic episode?

>7

If a patient giving birth has platelets of 100k and asks for an epidural should you give it?

Yes, of course. main contraindications are plt <70k and rapidly dropping platelets (as in severe preeclampsia)

UWSA1 - A 68M looks like he is developing contrast induced nephropathy after a a cardiac cath (3 days later). He also has a new onset blue toe on exam. What is the most likely cause?

Atheroembolism from the catheter isertion -> embolism in the toe and kidney -> blue toe syndrome and atheroembolic kidney damage *Will apparently see eosinophilia in serum and urine*

What is Paroxysmal Supraventricular Tachycardia (PSVT) and what does the ECG look like?

A dysrhythmia originating in an ectopic focus anywhere above the bifurcation of the bundle of His -> intermittent abrupt onset palpitations and possibly dyspnea, lightheadedness, chest pain and (rarely) syncope. narrow complex tachycardia, regular R-R and *retrograde P-waves* which are *inverted in inferior leads* tx - vagal maneuvers or adenosine (if stable), or immediate synchronized cardioversion (if unstable)

Disability Adjusted Life Year (DALY)

A measure of burden of disease, one DALY equals one year of healthy life lost due to premature death and time lived with illness, disease or injury.

UWSA1 - What is the pathogenesis of Factor V leiden?

A mutation in factor V, making it undegradable by protein C, leading to a hypercoagulable state *Activated protein C resistance*

What is a Noninferiority analysis?

A trial used to show that a new treatment is no worse than an old procedure (may use a one-tailed test)

What does Mastoiditis look like and what is the treatment?

A complication of acute otitis media, inflammation of mastoid air cells. ear protrudes forward and bulging tympanic membrane with loss of normal landmarks + light reflex Tx - IV antibiotics and myringotomy are initial treatments + drainage (tympanostomy or mastoidectomy)

what is a Chi-square test?

A statistical method of testing for an association between two categorical variables. Specifically, it tests for the equality of two frequencies or proportions.

In a patient with gout and hypertension, what is the first line therapy for blood pressure control?

ACEi and ARBs (Have mild uricosuric effects) diuretics decrease fractional excretion of urate which may exacerbate symptoms

A soccer player is running down the field and tried to pass around another player on the right when they feel a popping sensation in their knee with instant pain and swelling. One exam they hesitate to bear weight and have hemarthrosis on aspiration. What is the most likely cause?

ACL tear - common to have rapid onset hemarthrosis and joint instability Meniscal injury is more chronic and most people do not appreciate the njury when it occurs MCL injuries are generally when the person is struck on the lateral side of the leg

What is Tourette's syndrome associated with?

ADHD OCD Autism Spectrum disorder

A 33yo F presents 3 days after laparoscopic cholescectomy with fever, abdominal distension and tenderness with guarding, and diminished bowel sounds. She is on Acetaminophen and oxycodone Q6H for pain. XR shows distension of small and large bowel, stool in colon, and intraperitoneal free air. What is the next best step?

Abdominal CT with oral contrast to check for bowel injury leading to infection

What imaging test should be used to dx renal artery stenosis leading to HTN and AKI?

Abdominal duplex doppler ultrasonography (not angiography because AKI is contraindicated due to the use of dye)

Meniere's disease

Abnormal condition within the labyrinth of the inner ear that can lead to a progressive loss of hearing. (increased endolymph) Triad of symptoms - periodic, resolve spontaneously *dizziness or vertigo* *unilateral hearing loss* *tinnitus (ringing in the ears)*

What is seen on CXR in a patient with acute vs chronic Hypersensitivity pneumonitis?

Acute - scattered micronodular interstitial opacities Chronic - diffuse reticular interstitial opacities

how do you differentiate ASD vs VSD by heart sounds?

ASD - *wide fixed spitting of second heart sound*, mid-systolic ejection murmur and a mid-diastolic murmur VSD - loud, harsh, holosystolic murmur in the 4th intercostal space and a palpable thrill

UWSA2 - How do you deal with herpes in a mother in the perinatal period?

Acyclovir at 36 weeks, C-section if symptoms present

A 20 y/o F patient being treated for asymptomatic hypoparathyroid. She takes 50,000 units Vit D and 1.5g elemental Ca daily. Negative Chvostek's on exam. Her blood Ca is 7.8 mg/dL and PO4 is 5.2 mg/dL. 24h Urine Ca is 680 mg daily. What should be done about her present management?

Add a thiazide diuretic Changing Vit D to calcitirol is expensive and not too effective in comparisson. She has hypoCalcemia so you need to change something, and thiazides decrease urinary Ca excretion.

You suspect acute coronary syndrome 2 hours ago in a 53M patient, who has had one prior episode before both resolving in 10-15 minutes. He has a hx of GERD, gout, and bad knee (walks with a cane). ECG and troponins were normal. What is the next best step in management?

Adenosine myocardial perfusion imaging? Can't do ECG stress test because of the cane Can't do Holter monitoring because it's useless I guess...and can't help diagnose CAD in any way at all I guess

UWSA1 - A 39F diabetic pregnant woman at 39w gestation comes in with signs of labor. Her fundal height is 42w. Prior pregnancy is controlled and her diabetes is well controlled. Her cervix is dilated, fetus is vertex and estimated weight is 5.1kg. What should you do?

Admit for C-section Appatently all diabetic women, even if well controlled, get a C-section because they have increased risk of fetal demise, macrosomia and shoulder dystocia

What are the risk factors for a spontaneous abortion?

Advanced maternal age (>35) Previous spontaneous abortion Substance use disorder (Also - terratogens, uterine structural abnormalities, infection, chronic maternal disease [thrombophilia, thyroid, DM]) (NOTE: not caffeine, or C-section or anything else. This is the second ? they tried to trick me with this stuff)

What factor is associated with worse prognosis in a patient with poststreptococcal glomerulonephritis?

Age at diagnosis Adults more likely to develop CKD, kids recover in a few weeks

NBME5 - Which is more likely to cause erectile dysfunction, Obesity or long term alcohol use?

Alcohol (Even if it is described as 2/day)

a 42 y/o male is wandering around the streets in winter aimlessly. On exam he has mild hypothermia, disheveled and lethargic. he also has swollen, nontender bilateral cheeks consistent with salivary gland enlargement. What is the most likely cause?

Alcoholism -> Sialadenosis (benign, non-inflammatory swelling of the salivary glands) seen often in alcoholism, bulemia and malnutrition

How do you differentiate between abruptio placentae, Vasa previa, and placenta previa between clinical presentation of a prengant woman with vaginal bleeding?

All can have vaginal bleeding and fetal compromise AP - typically severe abdominal pain with tender, hypertonic uterus VP - painless minimal vaginal bleeding (from fetal blood) with rapid fetal exsanguination or death. PP - painless vaginal bleeding (maternal blood) so normally heavy and persistent. Also see signs of maternal hemorrhagic shock (hypotension, tachycardia) before fetal compromise

A 25 y/o M comes in with a 1cm area of complete smooth and circular hair loss with defined borders. What is the most likely cause?

Alopecia areata

What is HIV-associated lipodystrophy? How is it treated?

Alteration in fat deposition with no alteration in lean body mass, often *due to antiretroviral medications* -lipoatrophy - loss of fat in face, arms, legs, abdomen, buttocks (associated with NRTI's like stavudine/zidovudine) -fat accumulation - buffalo hump and visceral abdominal fat (abdominal girth without SubQ fat) both commonly lead to/are associated with *insulin resistance* in the future Tx - lifestyle modifications + statins (pravastatin, rosuvastatin, atorvastatin are most commonly used)

What is the best prognostic indicator for primary CNS lymphoma in an HIV patient who is just being started on medications?

An increase in CD4 count

What is Stratified analysis and what is the benefit of using it?

Analyzing data where the sample is separated into several subsamples according to specified criteria such as age groups, socioeconomic status, etc. it attempts to evaluate the presence of potential confounding or effect-modification at the analysis stage of a study

A 26 y/o male presents with low back pain for the past 8 months that is worse in the morning and better throughout the day.He has family history of back problems. On exam he has decreased ROM in lumbar spine and decreased chest expansion. What is the most likely cause, and first test you would order?

Ankylosing spondylitis XR of lumbar spine (*get repeat XR in 3 months to monitor the disease*) (associated with restrictive lung diseases due to apical fibrosis from dec costovertebral joint motion)

If you give an IM injection of epinepherine to someone in anyphylactic shock and it doesn't change their condition at all, what is the next best step?

Another IM injection of Epi (or slow infusion of IV if applicable)

How do you manage HIV associated thrombocytopenia?

Antiretroviral therapy (ART)

traumatic lumbar puncture vs. subarachnoid hemorrhage

Any RBC >6,000/mm3 could be either -TLP - see WBC (often 1 per ~750-1000 RBC) -TLP - high protein and glucose -SAH - Xanthochromia and discoloration of centrifuged CSF due to Hg breakdown

What are the indications for Valve replacement in aortic stenosis?

Aortic valve replacement is the only definitive tx

What is one of the main side effects of carbemazepine?

Aplastic anemia/neutropenia/thrombocytopenia due to bone marrow suppression

What is the prognosis for learning ability/disability for someone diagnosed with Turner's Syndrome?

Apparently Normal intelligence (Even though in the description it says they have an increased risk of learning disabilities)

UWSA1 - What mood symptoms can prednisone case?

Apparently depression

If you find out a 23y/o M has a bicuspid aortic valve what is the next best step in management?

Apparently get an echo for their first degree relatives because it runs in the family Autosomal Dominant with incomplete or sporadic penetrance (follow up echo every 1-2 years also but apparently that's not a good enough option)

How does alcohol cause neuropathy and loss of reflex?

Apparently it is directly toxic to neurons and causes axonal (toxic) neuropathy -> symmetric distal polyneuropathy

What is the treatment for mixed cryoglobulinemia syndrome?

Apparently retrovirals because only people with Hep C ever get it and no one else

How does stool frequency change in babies after the first month?

Apparently some kids can drop as low as one poop per week and still be regular, and it's common to have one poop every 2-3 days... So if that happens they don't have any obstruction...

What can help prevent preeclampsia recurrence in a pregnant patient?

Aspirin at 12 weeks gestation (Second trimester)

A 50 y/o M was shovelling snow and begins to have an MI. He is brought in by ambulance with cold extremities, hypotension, HR of 34 and crackles in the lungs. What is the best treatment?

Atropine? (not normal saline?) Apparently they have an RCA infarction leading to problems in the sinoatrial node -> increased vagal tone so you have to give an anticholinergic to treat it

What are the main features of Noonan Syndrome?

Autosomal Dominant Webbed neck Pectus excavatum Short stature Pulmonary stenosis (other cardiac abnormalities)

What is the treatment for cat scratch disease (bartonella henselae)?

Azithromycin (although in real life no treatment is generally needed as lesions self resolve)

If a patient with Sjogren's Syndrome develops a focal submandibular mass what should your first thought be?

B-cell non-hodgkins lymphoma (because SS leads to B cell infiltration into salivary glands)

How should you deliver a baby with shoulder dystocia?

BE CALM B - Breathe, do not push E - Elevate legs, flex hips/thighs against abdomen (McRobert's maneuver) C - Call for help A - apply suprapubic pressure L - enLarge vaginal opening (episiotomy) M - Maneuvers -deliver posterior arm -Rotate posterior shoulder (woods screw) - pressure to anterior aspect of posterior shoulder -Adduct posterior fetal shoulder (Rubins) - pressure to posterior aspect of posterior shoulder -Mother on hands and knees (Gaskin) -Replace fetal head and C section (Zavanelli)

How can you differentiate histrionic personality disorder and borderline personality disorder?

BPD - have self-harm, suicidal ideation, intense anger, chronic feeling of emptiness, and identity disturbance HPD - doesn't have that stuff

What age would you administer back blows for a choking victim as compared to abdominal thrusts?

Back blows <1 year old abdominal thrusts >1 year old

What is the best test to diagnose pneumothorax in an acute setting?

Bedside US apparently 19% chose this) -visualization of parietal and visceral pleura -*inability to detect lung sliding* in a non-acute setting use upright chest X-Ray posteroanterior view (52% chose this)

A patient has Acute coronary syndrome from an NSTEMI or unstable angina. They have had prior reactions to sinvastatin in their past. They also have JVD, pulmonary edema and an S3 auscultated. Which medication, normally given for ACS, should be avoided in this situation?

Beta-blockers - because of the possible acute decompensated heart failure (JVD, Pulm edema, S3) Can give all the other meds -dual antiplatelet (aspirin + PGY12 receptor blocker [clopidigrel]) -anticoagulation (heparin, enoxaparin, bilivarudin, fondaparinoux) -High intensity statins (prior adverse reaction does not mean you can't use it. Benefit > risks) -Nitrates

If a pregnant mother is at 36 weeks and 6 days, and goes into labor what should you do to manage them?

Betamethazone Apparently they need corticosteroids unless its >37 weeks

What is Saw palmetto used for and what is the most common adverse effect?

Benign Prostatic hyperlasia Mild Stomach Discomfort (*AND BLEEDING*)

What is the treatment for stage 2 syphillis?

Benzathine penacillin G, 2.4 million units IM weekly for 3 weeks

A 14F has 2 episodes of generalized seairures that started 1 hour ago and she has not returned to baseline yet. She is breathing fine, opening her eyes, but non responsive to questions. What should be done?

Benzodiazepines for abortive therapy

In addition to morphine, what should be given for medical management of ascending aortic dissection?

Beta blockers

If someone has worsening headaches localized behind their eyes and forehead, nausea and papilledema on exam, what should be done first?

Brain MRI with contrast (To rule out a mass lesion) (Can't do a lumbar puncture if there's a mass lesion because it could exacerbate sxs) (Note: if there is concern for subarachnoid hemorrhage do non contrast CT instead)

UWSA2 - What advice should you give to a 14M with sickle cell trait before he begins to exercise regularly or play sports?

Break often for rehydration (To prevent exertional rhabdomyolysis)

A 3 year old boy is brought in because of irritability and decreased appetite. Mother says he seems weak and older brother says he saw him eating woodwork and cement. What should be done next in his management and why?

Complete Blood Count Apparently this is lead poisoning? what the actual......

A newly diagnosed patient with SCC found in their larynx (Sx. diff swallowing, pain in both ears, exertional dyspnea, hemoptysis) was seen by and otolaryngologist who said the tumor was inoperable surgically. What is the next best step?

Combined Chemo and radiation (Works especially well with Head and Neck Cancers [HNC])

What is a paired t-test?

Compares mean values of a continuous variable between 2 categories/groups.

UWSA2 - differentiate condyloma accuminata (HPV) and molluscum contagiosum ony by look of the warts.

CA - painless, flesh-toned, smooth/verrucous, soft papules MC - Dome shaped, *central umbilication*

A 55M has CAD with 90% stenosis of LAD, 80% stenosis of RCorA and 75% stenosis of LCircumA. What is the next best step?

CABG (Apparently if you have more than 2 vessel involvement you have to do CABG rather than stent placement

What can be used to monitor response to treatment of diffuse lupus nephritis?

Complement (low in disease state) and anti-dsDNA antibodies

If you suspect orbital cellulitis (painful extraoccular movements, orbital swelling, fever, conjunctival erythema) what test should you run to rule out other similar presentations?

CT of orbits and sinuses - differentiates preseptal inflammation from orbital findings (preseptal cellulitis is more common but less severe) orbital cellulitis norally involves post septal structures like extraoccular muscles (->opthalmoplegia, painful eye movements) and orbital fat (->proptosis)

If a baby has abusive head trauma what test should be run on them?

CT of the head (even though literally 30 questions ago it said US of head for macrocephally now it's CT, f*** me why can't there just be one test that's used why does everything have to switch around and flip flop constantly???)

If you see someone with a solitary pulmonary nodule on XR after a recent lung infection and a 20 pack year smoking history, and they have no prior imaging to compare it to, what is the net best step?

CT scan

Someone who lives in California with no travel history develops Bell's Palsy for the past 3 days (unilateral facial plasy of upper and lower MN of CN VIII). He also has hepatomegaly on exam. What test should be run first and why?

CXR - apparently it's sarcoidosis Apparently Brain MRI is wrong because anything that would need that would likely only cause lower facial paralysis

UWSA2 - What is Cinacalcet and what is it used for?

Calcimimetic used primarily to reduce PTH (and Ca) in patients with hyperparathyroidism

If you put someone on chronic corticosteroids what else should you do to manage them for long term use?

Calcium and vitamin D

NBME5 - A 17F gives birth to a child and the child is found to be addicted to heroin. What is the next best step.

Call CPS (Not required to inform parents OR the child's own father... wft?? Guess men don't matter)

A 67M with oropharyngeal cancer on chemotherapy comes into the hospital. He has an absolute neutrophil count of 350/mm3. He has a superficial venous catheter for TPN. He has a fever and R eye pain with large, glistening and off-white lesions on fundoscopic exam. The lesions are 3 dimensional and extend from the chorioretinal surface into the vitreous. He also has vitreous haze and decreased visual acuity. What is the most likely cause?

Candida endopthalmus Tx - Systemic antifungals (amphotericin B IV) + intravitriol antifungals and vitrectomy (remove vitreous fluid?)

What problem are breast implants associated with?

Capsular contracture - fibrosis around implant causes pain, distortion of shape, implant deflation and rupture (They are NOT associated with any complications during or after pregnancy, including breastfeeding)

Before starting medical treatment for ADHD (stimulants), what test should be run on a child?

Cardiac History and Physical exam -baseline weight and vital signs, hx of cardiac disease or sudden cardiac death in the family Not history of substance abuse

What are the overrepresented health issues in women who have sex with women? (more common in WSW)

Cardiovascular Disease DM type II Obesity Cervical Cancer Breast Cancer Ovarian Cancer Depression, Anxiety Intimate partner Violence Bacterial Vaginosis

If a 65 y/o M patient has a left sided headache with Horner syndrome (ptosis, miosis) on the L side (ipsilateral), and a negative noncontrast CT of the head, what is the first diagnosis you should think of and how would you diagnose it?

Carotid artery dissection Dx - CT angiography of the head and neck

What is the difference in timeline between case control and cohort studies?

Case-control - find cases and look back in time from that Cohort - looks at exposure then measures for disease occurrence

If you have a patient with WPW syndrome (seen on ECG during an episode) and multiple episodes of syncope what is the next best step in management?

Catheter ablation (They are symptomatic and have documented supraventricular tachycardia)

Lactational mastitis

Caused by missed nursing sessions Presnts with a warm, tender quadrant of a single breast (without fluctuance) + axillary lymphadenopathy so DON'T GET CONFUSED. Can develop into a breast abcess (dx with breast US) Tx: Abx (that target S. Aureus) + continue breastfeeding *Dicloxicillin*

What is the treatment for Acute Chest Syndrome in soeone with Sickle Cell Disease?

Ceftriaxone + Azithromycin IV fluids pain control

Someone comes in with a 2 week history of large volume, greasy, foul-smelling, loose stools. Testing for ova/parasites is negative and it seems to be unrelated to meals. They have increased stool osmotic gap and mild microcytic anemia on lab testing. What is the most likely cause?

Celiac Disease (villous atrophy with increased intraepithelial lymphocytes on colonoscopy with biopsy)

If someone is on levothyroxine for iatrogenic hypothyroidism after a total thyroidectomy (from Grave's), but still has high TSH and low free thyroxine, what is the most likely cause?

Celiac disease Because they had Graves, they're more likely to have other autoimmune diseases and Celiacs causes malabsorption of levothyroxine

What is the management of cystitis and asymptomatic bacteruria in pregnancy?

Cephalexin Amoxicillin-clavulanate Fosfomycin

UWSA2 - cephalohematoma vs caput succedaneum

Cephalohematoma - swelling caused by bleeding between the osteum and periosteum of the skull *C*aput *S*uccedaneum - (CS for crosses sutures) Edematous swelling on the scalp caused by pressure during birth, this swelling may cross suture lines, usually disappears in a few days *Caput Succedaneum = Cross, Sutures & Symmetrical*

What is labor protraction and how is it treated?

Cervical dilation <1cm in 2 hours (active phase is 6-10cm dilated) tx - with labor augmentation (oxytocin or amniotomy)

What is periodic breathing?

Cessation of breathing lasting 5 to 10 seconds followed by 10 to 15 seconds of rapid respirations without changes in color or heart rate. (thought to be due to immaturity of CNS in infant up to 6 months old)

If a 26yo M from South America is found to have a Left Ventricular Apical Aneurysm in their heart, what is the most likely cause?

Chagas Disease (also dilated cardiomyopathy) (other cause could be MI, but not in this age group)

NBME5 - Other than gonorrhea, what else can cause Fitz-Hugh-Curtis syndrome?

Chlamydia apparently... maybe it's just caused by PID?

How do you differentiate necrotizing faciitis being caused by Group A strep (pyogenes) or clostridium dificiles?

Clostridium (and other anaerobic bacteria) are more likely to produce gas and cause crepitus, but Group A Strep is more common in general

If a patient has polycythemia and elevated EPO, what is the first thing you should think of?

Chronic hypoxia (not cancer)

A 15 y/o male previously healthy presents with muscle atrophy, facial weakness, dysphagia, ptosis, temporal wasting, thin cheeks and delayed relaxation of thenar/hypothenar muscles on contraction. He is also noted to have testicles smaller than average. What is the most likely cause?

Classic myotonic dystrophy (autosomal dominant)

Tetanus prophylaxis

Clean Wound: If <3 doses or uncertain # of doses then Tdap Dirty Wound: If <3 doses or uncertain # of doses then Tdap + TIG If >3 doses for Clean or Dirty: Tdap unless documented prior to receipt of Tdap

Other than metronidazole, how can you treat Bacterial Vaginosis?

Clindamycin

UWSA1 - What is the medical management of a carbuncle (boil?)?

Clindamycin, TMP-SMX, doxycycline (often they're MRSA so I guess that's something)

Management of Group B strep in a pregnant patient

If found, whether with clean catch urine during pregnancy or during screen at 35-37w, *IMMEDIATELY TREAT WITH AMOXICILLIN OR CEPHALEXIN!!!!!* Then intrapartum penicilin prophylaxis (All patients, regardless of culture within the next week, are assumed to have persistant resistant GBS colonization after treatment)

NBME5 - What is the treatment for cat-scratch disease?

It is self-limited usually so none (Treat if there is hepatic, ocular or CNS involvement) (Actually even in this explanation it says treat most cases unless there is ONLY mild lymphadenitis)

After orchiopexy what happens to the risk of malignancy?

It stays high... somehow... orchiopexy is moving the testicle to the scrotum, not removing it

What is a factorial study design?

It utilizes >2 interventions and *all combinations of these interventions.* It is also called a fully crossed design.

Anticoagulation during pregnancy

LMWH throughout pregnancy Unfractionated heparin before delivery WARFARIN is teratogen (bone/cartilage)!! - but can sometimes be used in 2nd/3rd trimester in patients with mechanical valves. Warfarin is preconception/postpartum mostly though

What is the main cause of Erythrasma and what does it look like?

Corynebacterium minutissimum

What tests should be run on an infant if they have macrocephaly?

Cranial Ultrasound (not XR) to check for open anterior fontanelle (or other causes?)

If you think a child might have pertussis but that's not an option, what does UW actually want you to think?

Croup - because of the barking cough + Stridor + other things Tx - corticosteroids + nebulized air (mild) or nebulized epinepherine (mod/severe)

What are the non-surgical treatments for Squamous Cell Skin Cancer?

Cryotherapy, electrosurgery and *radiation therapy*

What do the results of a forward bend test (for scoliosis) indicate?

Curvature >7 degrees (or >5 degrees if overweight) may indicate significant scoliosis If positive get a spinal XR

NBME5 - What is the best treatment for lung cancer that has not metastasized?

Cut that lobe of the lung out (Chemo, radiation and laser ablation are normally for non-resectable ancers or as adjunctive therapy)

Someone with uncontrolled HIV presents with new skin rash that resembles multiple small papules with central umbilication on face, trunk, neck and upper thighs. Some leasions have central necrosis or even a hemorrhagic crust. What is the most likely cause?

Cutaneous cryoptococcosis (resembles molluscum contagiousum but with central hemorrhage/necrosis) (Disseminated MAC has rare skin lesions and are usually ulcerating and nodular. Kaposi Sarcoma as purple papules) dx - *BIOPSY OF THE LESION*

A 2M has Burkholderia (pseudomonas) cepacia pneumonia, what is the most likely predisposing factor? What if there was also granuloma formation?

Cystic Fibrosis if granuloma formation *chronic granulomatous disease* (Catalase positive organisms - staph aureus, B cepacia, Aspergillus, Nocardia)

Which drugs can decrease oral contraceptive effects leading to potential pregnancy?

Cytochrome p450 inducers -antiepileptics (phenytoin, carbemazepine, ethosuximide, phenobarbital, topirimate)

What is the presentation of Kawasaki disease and how do you treat it?

Fever for 5 days with age <5 and this triad -Rash [peeling and erythema] (inguinal folds, perineum and trunk) -non-exudative conjunctivitis -mucositis (erythematous lips and tongue [strawberry tongue], oral mucosa) ***Tx with ASA and IV immunoglobulin*** *Associated with coronary artery aneurysm (get an echo)*

what are the main 2nd generation antihistamines?

Fexofenadine, loratadine, and cetirizine

UWSA2 - If you suspect Parvo B19 in a pregnant woman what should you do?

Get serology to determine infection status Tx - monitor with US frequently for anemia/hydrops fetalis and if they have it need to do intrauterine fetal transfusion

UWSA2 - Excess flatulence and greasy, foul-smelling diarrhea is what kind of infection?

Giardia

NBME5 - A 6h old M has periventricular calcifications on CT. What should the mother be counseled the baby might also have?

Deafness (BUT NOT BLINDNESS?!?! what about chorioretinitis? They say only rubella has that as if Toxo/CMV don't also)

NBME5 - What is the most important prognostic factor in a 66F with HTN and ESRD who currently has an infetion of his prosthetic joint?

Decision to remove the prosthesis (Not age or infecting organism, lol this blows)

How should Type 1 diabetics adjust their insulin doses (long or short acting) before excercise?

Decrease short acting insulin within 1-3 hours of exercise, proportionate to intensity of exercise (if exercise is prolonged [>60min] or before breakfast, dose of basal insulin should also be decreased)

What are the main complications of acute compartment syndrome?

Decreased sensation, motor weakness (within hours), paralysis (late), dec in distal pulses (uncommon) *Acute Renal Failure from rhabdomyolysis*

UWSA1 - Long QT Syndrome (LQTS)

Delay in polarization. Heart is made vulnerable to potentially lethal V Tach. Triggers for V Tach: Electrolyte imbalance, sudden loud sound, bradycardia, activity like swimming/basketball. Can be genetic or caused by medication

If someone comes in with a febrile illness after visiting India (drank only bottled water, up to date on vaccines, took prophylactic meds) and on exam you tourniquet their arm and petechiae start to develop, what is the most likely cause?

Dengue Fever apparently leads to hemorrhagic tendencies ---> *circulatory failure*

What complication can iron deficiency anemia (or cow's milk anemia which can also lead to IDA) lead to in a child?

Developmental delay -psychomotor delay -neurocognitive impairment with iron supplementation gross motor skills may improve but the remaining cognitive effects may be permanent

What is the most significant risk factor for heart related events?

Diabetes Not smoking, that's never the answer I hate f***ing everytthing and my life. Oh also CKD and non coronary atherosclerotic disease but who gives a S*** about those as long as UW can make me wanna kill myself

After renal transplant what is the patient at highest risk of developing (disease)?

Diabetes mellitus (from immunosuppressive drugs [glucocorticoids - dec insulin sensitivity, calcineurin inhibitors - can impair insulin secretion] and improved renal function [inc renal excretion of insulin and inc gluconeogenesis])

A 58 y/o male with diabetes, HTN, gout, GERD and erectile dysfunction presents after an MVA. He has absent cremasteric reflex, bilateral symmetric reduction in pain/vibratin sensation in both feet/hands. What is the etiology?

Diabetic neuropathy -hands and feet (stocking and glove) distribution means diabetes apparently Also ED and absent cremasteric reflex indicate it.

Intussusception treatment

Diagnosed by xray, ultrasound, air or barium contrast enema, which sometimes reduces the obstruction without need for surgery Surgery is indicated when barium or air contrast enema does not reduce the intussusception

UWSA1 - What are the most dangerous complications of granulomatosis with polyangitis?

Diffuse alveolar hemorrhage (and others? maybe glomerulonephritis?)

What is the treatment for diffuse esophageal spasm (uncoordinated, simultaneous contractions of esophagus)?

Diltiazem (CCB's) also Nitrates or tricyclides

a 46M presents for cardiac cath. He has a PMH of diabetes and HTN. He's on aspirin, metformin lisinopril. What changes in meds should occur before the procedure?

Discontinue Metformin day of procedure and restart it 2 days after because large doses of iodine contrast interfere with kidney function which increases the risk of lactic acidosis from metformin

NBME5 - If a 17F brings her 12M brother in but their parents are on a flight, unreachable by phone, what should you do?

Do an exam to see if there are any life threatening conditions

If someone is getting bariatric surgery (gastic band, gastric bypass, gastrectomy) and they want to get pregnant in the near future what should you tell them?

Don't get pregnant until a year after the bariatric surgery

UWSA1 - A 27F at 28weeks gestation G2 P1 comes in. She is asymptomatic. She is Rh(D) negative but the father is Rh(D) status unknown. Her initial titer was 1:2 and is now 1:16. What do you do?

Don't give Anti-D Immunoglobulins? (Apparently Rhogam is only to prevent alloimmunization but she has already alloimmunized if she's 1:16 so you just f*** off and let the baby die)

What is the treatment for a prolactinoma, even if they have visual defects or compressive symptoms?

Dopaminergic agonists work for most cases -bromocriptine -cabergoline

A 9y/o M has lyme disease arthritis in the L knee. How is it treated and what should you tell the parents about his prognosis?

Doxycycline even for a young kid apparently (21 day course) (or amoxicillin but doxy preferred for CNS penetration) He will likely have no lasting effects from the disease (is this actually true? did I learn everything wrong?)

What measurement technique should be used to diagnose osteoporosis in an elderly patient?

Dual energy X-ray absorptiometry (DEXA) recognize it spelled out

How do you diagnose and treat allergic pulmonary aspergillosis?

Dx - allergy skin test for aspergillus (and/or IgE >417 IU/mL) tx - oral prednisone (can use antifungal therapy with itraconazole or voriconazole to reduce aspergillus load

UWSA1 - A 72M comes in with hip pain increasing in severity over the past 2 mo. The pain is fine in the morning and worse throughout the day. On exam there is limited internal rotation of the hip, and this also elicits groin pain. What is the next best step in management?

Hip X-ray - check for osteoarthritis which it most likely is Apparently pain in the groin is common for OA of the hip

NBME5 - A 38F G2P2 has had heavy menstrual bleeds since the birth of her child 3y ago. She got a tubal ligation, and has a PMH of HTN and smoking. PE shows a slightly enlarged uterus and blood in vaginal vault (not brisk bleeding). She has mild anemia. She has a normal Pelvic US and negative pregnancy test. What is the next best step?

Endometrial biopsy? (literally only because of the enlarged uterus on exam because that's a risk factor for endometrial Ca... even though she's pre-menopausal)

What is the difference between erysipelas and cellulitis?

E - raised and sharply demarcated C - ill defined flat borders

A 22y/o F has RLQ pain with bowel movements and a normal exam other than pain on deep palpation of RLQ. She has nodulatiry on the rectovaginal exam. What is the likely cause and what problem is it associated with?

Endometriosis it leads to infertility

What is the medical management of pulmonary hypertension?

Endothelin receptor antagonists (i.e. bosentan, ambrisentan)

What is the next best step in a 48 y/o M patient found unresponsive on a park bench smelling of alcohol. While being evaluated in ED he vomits bright red blood. What is the next best step?

Endotracheal intubation (apparently you have to stabilize the patient first even though they have no resp distress) (Nevermind, have to do it because he's unresponsive and that warrants it)

A 16M has fever (101.5), fatigue, myalgias and bilateral anterior and posterior cervical lymphadenopathy for the past 10 days. he has no hepatosplenomegaly or cough. Tonsils are enlarged an erythematous bilaterally with white exudate. What is the most likely cause?

EBV apparently strep pharyngitis doesn't cause fatigue or, get this, posterior cervical lymphadenopathy

NBME5 - If a 10M is being treated for ADHD and still sometimes zones out in class and forgets what the class is doing what should you do next?

EEG to check for absence seizures... (Seems like a stretch but they're saying it's a post-ictal confusion rather than just being distracted)

If a patient presents with dyspepsia but they're >60 y/o what should you do?

EGD literally just got the other version of this question where they were 20y/o and it's H pylori testing so I put that and of course its not the same because why would UW ever be consistent

If someone has symptoms of gastroparesis from DM, what is the first thing you should do?

EGD - apparently to rule out mechanical obstruction...

A 62M comes in with anemia. Hg is 8.6, MCV is 70. Stool occult blood is positive. His EGD and pill capsule endoscopy are normal but colonoscopy shows several small, flat, cherry-red lesions in the R colon. What is this anemia most likely associated with?

ESRD (apparently these are angiodyslpasias don't bleed often, and when they do patients likely have associated ESRD, Von willebrands or Aortic Stenosis)

If a patient with history of mild internal hemorrhoids and diverticulosis has painless, large volume bright red blood in stools for the past 2 days, what is the most likely cause?

Eroded small artery of the colon (diverticular bleeding) hemorrhoids are normally small volume BRB in stool not large. colonic angiodysplasia is also wrong (AV malformation) because it usually has some history of ESRD, Von willebrand disease, aortic stenosis, etc.

UWSA2 - Erythematous, blanching, reticulated rash on upper and lower extremities, accompanied by a viral prodrome in a child means what?

Erythema infectiosum (parvo B19) "lacy, reticulated rash"

NBME5 - How often should you give mammograms to women?

Every 1-2 years starting at age 40-50 google: Age 45-54 - every year Age 55-75 - every other year

How do you diagnose brain death?

Exam c/w brain death: coma, no motor response, no pupillary response, no corneal reflex, no gag In addition to above, need *apnea test*: get the PCO2 to 40-60 and the patient normothermic, then d/c the ventilator and trend ABGs at 1,5, and 8 minutes. If PCO2 rises >20 without observed respirations, confirms brain death. EEG is not needed unless apnea test cannot be performed

What is the main treatment of salicylate poisoning with metabolic acidosis?

IV Sodium Bicarb to alkalinize the plasma (and urine?) to prevent protonation of salicylate into salicylic acid (Which can then pass through the blood brain barrier, renal tubules, etc.) Even with alkalosis they should still receive sodium bicarb

How do you differentiate between Impaired cardiac mobility (cardiogenic shock) and cardiac tamponade using only intracardiac pressures?

In tamponade you have inc R atrial and ventricular pressures with characteristic equalization of Right atril, right ventricular end diastolic and Pulmonary Capillary wedge pressures in impaired motility you may see PCWP > R ventricular end diastolic This was a rough question

A patient on chemotherapy comes in with a fever (102F) and chills for the past 24h. He also has mucosal pallor, tachycardia, and mild gingival erythema. leukocytes are 690/mm^3 with 20% neutrophils, Hg is 8.4, hematocrit is 25% and Platelets are 74k. What is the most likely problem, and how would you initially treat it?

Febrile neutropenia (<1500 absolute neutrophil count = neutropenia) Often when isolated this is caused by infections from pseudomonal species, so give them antipseudomonal beta-lactams from gram + and - coverage. *Pipercillin-tazobactam, cefepime, meropenem*

What test should be run if you are considering diagnosing Restless Leg Syndrome?

Ferritin levels (or iron studies) because secondary RLS is caused by anemia

UWSA1 - if a maternal serum AFP level is elevated at 15w gestation, what should be done next?

Fetal US

What non-pharmaceutical treatment is used for OCD?

Exposure and Response Prevention Therapy (They never just say CBT, they always use some made up bulls*** for it, like f*** man I knew this one and still got it wrong!) (Note: Dialectic behavioral therapy is also CBT but only for borderline personality disorder?)

Centor Criteria

Exudative pharyngitis Fever Tender anterior cervical LAD Absence of cough Modifier: <15 years old --> add 1 point >44 years old --> subtract 1 point Abx at >4, culture 3-4 and symptomatic tx -1-1 *culture = rapid antigen detection test, as throat cultures take too long to get back even though they're more sensitive*

NBME5 - What pulmonary function test is best for determining a patients' tolerance to major pulmonary resection as a treatment for lung cancer?

FEV1 (must be >60%)

How do you differentiate the different solid liver masses by imaging?

FNH - central scar (hyperdense with contrast, hypo without) hemangioma - centripital enhancement on triphasic CT (moving from periphery to center) adenomas - women on birth control with centripital enhancement on triphasic CT HCC - with symptoms, enhances with contrast, atypical central scar

What is the definition of infertility?

Failure to conceive after 1 year of unprotected intercourse (or 6 months for women >35)

What does the rash from Parvovirus B19 look like?

Faint, erythematous, reticular rash, often on face, arms and legs

How do you proceed in a 5 week old child with severe hypertrophic pyloric stenosis?

First, rehydrate and correct electrolytes then proceed with immediate pyloromyotomy

Someone on methotrexate develops macrocytic anemia. What can be given to them to alleviate the problem while staying on the drug?

Folinic acid (not folic) - it is an active form of folic acid and bypasses the block on DHFR

What is the treatment for ethylene glycol (antifreeze) ingestion?

Fomepizole infusion (not combined with ethanol in any way, as fomepizole is superior)

UWSA1 - A 12M presents with R leg pain after falling while playing soccer. He has had growing pains in his legs for years that awaken him at night but this pain is severe and localized. He has splenomegaly (5cm below costal margin). Hg 10.5, Plt 95,000, and point tenderness over right tibia. XR shows transverse proximal right tibial diaphyseal fracture. What is the most likely underlying cause?

Gaucher's disease? (This exists still?) Glucocerebrosidase deficiency leading to glucocerebroside accumulation in macrophages Apparently it's the anemia, thrombocytopenia and splenomegaly that give it away, but bony pain and osteopenia are common symptoms as well

What symptoms are possible with damage to your dominant parietal lobe (most often L parietal)?

Gerstmann Syndrome -> -acalculia - difficulty with simple arithmetic -finger agnosia - inability to name individual fingers -agraphia - impaired writing -*right/left confusion*

If someone has osteomyelitis from an infected foot ulcer and you have culture and sensitivity on the bacteria from the ulcer, how should you treat the osteomyelitis?

Get a bone biopsy apparently they're different bacteria someone kill me please

UWSA2 - A 35F has L breast pain for the past 2 weeks. There is no associated lump palpated or fever/ swelling. Pain is in a region 5cm from the nipple. What should you do?

Get a mammogram Apparently even without a lump as long as there is noncyclic, unilateral, focal breast pain it raises concern for malignancy (possibly inflammatory breast ca?)

A 77 y/o M comes in with recent memory loss. His TSH and B12 are normal. He complains of some problems sleeping. His MMSE ~26. What should you tell him?

Get a sleep study, he may have OSA which is causing mental deficiency (I seriously hate this test)

Immediately before an emergency surgery a patient develops a fib. He seems hemodynamically stable but his HR is 119. What should you do?

Give a beta blocker (or nondihydropyridine CCB - diltiazem) and proceed with surgery (If the surgery is non-emergency you can delay it to work it up. no need for anticoagulation as risk for thrombolysis is low during surgery)

UWSA1 - If a patient has a recurrent skin infection on their leg that has been treated with antibiotics but still recurred, What should you do?

Give antifungals - recurrent skin infections are commonly due to tinea pedis, lymphedema or chronic venous insufficiency

Someone from Arizona develops R sided facial paralysis of upper and lower face starting yesterday. He has a smoking hx. There are no wrinkles in the forehead when he tries to raise his R eyebrow. What is the next best step in management?

Give prednisone..... apparently for Bell's Palsy get this... because it spares the forehead, you apparently never assume it's a stroke or even test for it. Actually eff UW, for real this time. Only do an MRI if forehead spared because then it's intracranial

UWSA1 - If you intubate someone and they drop to hypovolemia, but maintain Po2, what should you do to them?

Give them fluids? The positive pressure of intubation is increasing intrathoracic pressure leading to low R ventricular preload thus decreasing BP

UWSA1 - What laboratory studies do you need for an initial discussion of obesity?

Glucose (or A1c), TSH, lipids, hepatic enzymes

A 77y/o M (PMH HTN, COPD, CKD) who lives at home with his wife who has dementia calls into the office. He has had a cough and fever for a few days but yesterday it got more productive and he got chills. What should you advise this patient?

Go to the ED immediately Apparently because he's older than 65 he's at a much higher risk of dying from complications of influenza pneumonia (and also the comorbidities)

What is Stress hyperglycemia and how does it normally present?

Insulin resistance associated with critical illness, caused by increased catecholamines/glucocorticoids. Presentation - Someone with sepsis/ICU admission/ severely ill who develops hyperglycemia, ketonuria/glucosuria, but with *normal A1c* and *no diabetic hx* RF - ICU admission, Temp > 39C (102.2F), severe illness, Sepsis, CNS infection

What are the risks of different herbal medications? (Gingko bilboa, Ginseng, Saw palmetto, Black cohosh, St. John's wort, Kava, licorice, Echinacea, Ephedra)

Gongko bilboa - increased bleeding Ginseng - increased bleeding Saw Palmetto - mild stomach discomfort Black cohosh - hepatic injury St. John's wort - drug interactions (antidepressants -> serotonin syndrome, OCs, anticoagulants (dec INR), digoxin. Also Hypertensive crisis Kava kava - Severe liver damage licorice - hypertension, hypokalemia Echinacea - allergic rxns, dyspepsia Ephedra - HTN, arrythmia/sudden death, stroke, seizure

A 39M comes in with 3 month history of nausea and burning epigastric pain. He has been taking ranitidine over the counter for 2 weeks. What should be your advice for him?

H pylori testing They changed it so now everyone gets H pylori and no one gets told to take meds for 2 extra weeks or switch to PPI (lansoprazole) dumb

A 31y/o M comes in with fullness in epigastric area and occasional nausea over the past 2 months. He has no other history and takes no other meds. What is the next best step?

H pylori testing??? Apparently he has dyspepsia so you instantly test for H pylori, contradicting the last 3 questions with similar presentations. 22% got it right QID 8961

A 54 y/oF comes in with symptoms of acute calculous cholecystitis but the abdominal US is inconclusive (no gallbladder wall thickening and negative Murphey's). What is the next best step in this situation?

HIDA scan (hepatobiliary iminodiacetic acid scan, a.k.a. cholescintigraphy) ERCP is only for choledocholelithiasis Can't go straight to cholescectomy without confirming dx

So... even though the literature says HIT takes 5 days to develop after heparin is begun and if it is less than 5 days it isn't HIT... If you have a patient who develops HIT-like symptoms in 1 day of Heparin use, what is the cause?

HIT... because the literature is always wrong never trust anything in medicine, it's all made up anyway

UWSA1 - Hypoplastic Left Heart Syndrome vs Tetrology of Fallot vs Truncus arteriosus presentstions

HLHS - Must have ASD or Patent foramen ovale to live. See cyanosis on day 1, respiratory distress and cardiogenic shock. Murmur - Single S2 (absent aortic valve), often no murmur though ToF - "tet" spells (severe cyanosis). Murmur - systolic ejection murmur? TA - neonatal signs of HF (resp distress, poor feeding, etc.). Murmur - systolic ejection murmur

UWSA2 - What suggests a more aggressive course for Rheumatoid arthritis (rapidly progressive, joint erosions, etc.)?

Having either (+) RF or (+) anti-cyclic citrullinated peptide means a more aggressive course. Having both positive is even more aggressive Being seronegative (Negative both of these) suggests less aggressive course

UWSA2 - A 15y/o has an MI while playing basketball. No underlying conditions. Echo shows aneurysmal dilations, myocardial thickening, and calcification of proximal R coronary aa. What is most likely the problem?

He had untreated Kawasaki's and it f***ed him up

A 1 week old boy with no clinical signs is brought into clinic. He had an abnormal lab test with T4 at 4 (normal 6-10 1 week after birth) and TSH of 46. What should you tell the parents?

He needs hormonal supplementation immediately to prevent developmental delay (Apparently most newborns are asymptomatic because of residual T4 from mother, so it is not a lab error)

What are the clinical manifestations of otosclerosis and who is most at risk?

Hearing loss that paradoxically improves (in regards to speech discrimination) in noisy environments (consistent with conductive hearing loss) + reddish hue behind tympanic membrane (from bony ossicle overgrowth) Normally seen in younger (mid thirties) caucasian women (Autosomal dominant with incomplete penetration) tx - hearing aides (amplification) or surgery (Stapes reconstruction)

A 79M is on a ton of opiods for pain control for metastatic prostate cancer. His caregiver says his breathing is weird. His respirations are 24/min and pO2 is 90%. What is the most likely reason?

Heart failure or PE? Get a CTA because he has heart failure increasing respiratory drive...

What are the different treatments/exams to be done for the local vascular complications of cardiac catheterization (hematoma, pseudoaneurysm, AV fistula)?

Hematoma - reassurance and observation pseudoaneurysm - US for diagnosis and US compression or thrombin injection for tx (if large or rapidly expanding surgical correction may be needed) AV fistula - dx with angiography

What is excercise-induced hematuria?

Hematuria often seen (24%) in marathon runners after a marathon. It presents with painless hematuria (cola-colored urine) after prolonged excercise. Presence of RBC's rules out myoglobinuria (rhabdo) and hemoglobinuria (from RBC lysis) It is a diagnosis of exclusion, *repeat urinalysis in 1 week to ensure resolution*

NBME5 - What vaccine should you give to someone traveling to Kenya?

Hep A

What is the treatment for peri-infarction pericarditis (pericarditis developed within 4 days of an acute MI)?

High dose aspirin (650mg 3x/day)

What tests do you need to perform to diagnose stress urinary incontinence (leakage with cough, sneeze, laughter, etc)?

History, Physical and UA THAT'S IT NOTHING ELSE!!!!!

If curative failure is futile in a newborn patient but the parents demand that it be performed against advice from multiple medical professionals what is the next best step?

Hospital ethics committee for the first time ever lol

Instead of using CAGE questions, what individual question can you ask someone to screen for Alcohol abuse screening? (because apparently UW hates CAGE or is just stupid or something)

How many times in the past year have you had 5 (or 4 for women) or more drinks in a day? a response of >/= 1 is a positive?!?!?!

A 23F G1P0 comes in at 37w gestation with severe headaches, fatigue and blurred vision. She has features of preeclampsia with severe features with a BP of 170/110 and urine protein. What should you do next?

Hydralazine and Magnesium sulfate (stabilize the mother first and then deliver) (I put Mg and C-section which they said should only happen after stabilization, and should also consider live birth if fetal maturity is good)

What does Calcium gluconate treat?

Hyperkalemia

Chronic allergic contact dermatitis

Hyperkeratosis, hypergranulosis, psoriasiform acanthosis, fibrosis of papillary dermis, mild perivascular lymphohistiocytic infiltrate of the dermis; CD8 cells in epidermis; Th1 cells in dermis

What are some of the main causes of pseudogout?

Hypothyroidism, hyperparathyroidism, iron build-up i the blood (*hemochromatosis*, etc.), hypercalcemia, (and Mg deficiency?)

15 y/o F presents with amenorrhea. She had breast development at 12. She recently had a growth spurt this year of 3.15 in. She has occasional fatigue. She is 30th % of weight and 80th in height (stable over years). Sexual maturity rating is 4, absent pubic and axillary hair. What is the most likely cause?

I have no idea how they got this without telling us if a uterus is present/absent or karyotyping BUT whatever ...*Androgen insensitivity syndrome* apparently Mullerian agenesis you have pubic hair and axillary hair

NBME5 - Which measurable factor is best used for prognosis in a patient with fulminant liver failure with encephalopathy?

INR *(MELD score includes Bilirubin, Creatinine and INR)* (not neuro exam)

How do you treat infant botulism?

IV Botulism immune globulin

What is the simplest and least toxic treatment for prolonged bleeding time in someone who is actively bleeding?

IV Desmopressin If the patient doesn't respond to transfusions and desmopressin then consider cryoprecipitate

A pregnant woman (32 w) is having an acute asthma attack. She stopped using her normal inhaled corticosteroids once she found out she was pregnant. She is given ipratropium and albruterol in the ED to some relief but is still having wheezing on exam. What is the next best step?

IV corticosteroids and observe (if the SABA and LABA had resolved the issue then maybe restart inhaled CS and discharge, but it didn't resolve) You want to maintain SpO2 >95%, because asthma attacks have a high risk for maternal and fetal death. There is a low threshold for administering IV CS in these patients as well.

What is the treatment of toxic megacolon?

IV corticosteroids are the drug of choice... Then give IVF, abx and bowel rest apparently surgery is only reserved for perforations even though that's what we've been taught since... forever

What should be given for someone with toxic shock syndrome?

IV fluids (up to 20L per day???) IV clindamycin (can possibly stop exotoxin production) IV Vanco (antistaphylococcal)

What is the management of pyelonephritis in pregnancy?

IV hydration and broad spectrum B-lactam abx (ceftriaxone, cefepime, etc) until afebrile for 48 hours Once afebrile then oral antibiotics for 10-14 days After that daily suppressive therapy (e.g. low dose nitrofurantoin, cephalexin) until 6 weeks post partum

What is the treatment for a multiple sclerosis exacerbation?

IV methylprednisone (technically oral and IV work just as fast but if the patient has optic neuritis IV should be chosen as oral corticosteroids are associated with recurrent optic neuritis. Oral GC taper can be considered after IV GC. Plasmanephresis can be considered in GC refractory patients)

If someone is on phencyclidine, what should you give them to prevent an adverse event?

IV saline infusion... to prevent rhabdomyolysis

What treatment best improves outcomes in a 4M patient who is diagnosed with septic arthritis by joint aspiration?

IV vancomycin

Third degree AV block treatment

If symptomatic: -Transcutaneous pacemaker until a temporary transvenous pacemaker can be inserted -Atropine, dopamine, epinephrine temporarily until pacing is started -Patient will need a permanent pacemaker ASAP

Why would you report the median rather than the mean value in a drug ad showing the benefits of a new drug?

If there was *Skewed distribution* of the sample in strongly skewed distributions the median is actuallly a better indicator for central tendency than the mean When is which choice the best you ask? normal skew, continuous data = mean high skew, ordinal/continuous data = median nominal data = mode (e.g. male and female)

UWSA1 - How long should a couple have to wait while trying to conceive before you can screen them for causes of infertility?

If woman is <35 years old, must wait >1 year If woman is >35 years old, must wait >6 months

What is the treatment for ITP (immune thrombocytopenia)?

Immunoglobulin

What is a Mycotic aneurysm (Note: nothing to do with fungi)?

In a patient with infective endocarditis, they have an embolism that is infected with bacteria -> destroy wall of vessel it embolized to -> lead to abscess formation (most common is from infected valve cusps) *or aneurysms*

What advice should you give to a patient with hypothyroidism (on levothyroxine) who wants to start oral contraceptives?

Increase the dose of levothyroxine before starting OCPs (OCPs include or modify amount of estrogen which can increase TBG. This would lower free T4 otherwise so you need to increase the levothyroxine to counter it)

UWSA2 - independent vs dependent variable

Independent variable - variable that is changed or controlled in a scientific experiment to test the effects on the dependent variable. Dependent variable - variable being tested and measured in a scientific experiment. Can be response to treatment or a measurable effect

transverse myelitis

It is a neurologic disease, that attacks brain and spinal cord and has varying degrees of severity -affects spinal cord bilaterally (motor and sensory) -inflammation, targets myelin *-pain, sensory and motor deficits, bowel/bladder issues*

What is Intention to treat analysis and what is the benefit of using it?

Individual outcomes in a clinical trial are analyzed according *to the group to which they have been randomized, regardless of whether they dropped out, fully complied with the intervention or crossed over to the other treatment*. By simulating practical experience intention to treat analysis provides a *better measure of effectiveness* (as opposed to efficacy).

27M with untreated HIV and nodular opacities in the lugs including subpleural opacities on the right and with SOB, dry cough and R sided CP likely has what problem?

Infective endocarditis (not PCP) leading to septic emboli in the lungs Get an echo then go f*** yourself

UWSA1 - What can varicocele lead to in an adult male?

Infertility Testicular atrophy

UWSA1 - If a 36F develops recurrent UTIs with gut bacteria, what do you suspect?

Inflammatory Bowel disease - because of f***ing fistula formation from Crohn's (not urinary tract malformation because "you should only see that in children" when really it just depends on your definition of malformation)

UWSA1 - What is the anatomical difference in location between femoral and inguinal hernias (Step 1 throwback)?

Inguinal hernia pass above inguinal ligament -indirect pass into inguinal ring and descend into the scrotum, lateral to inferior epigastric vessels -Direct through hesselbach's triangle medial to inferior epigastric vessels Femoral hernias pass below inguinal igament -medial to femoral Vein, aa., nerve Note - neither pass obturator foramen

NBME5 - What is the best next step in management of a patient with an acute asthma attack?

Inhaled SABA or *inhaled corticosteroids* are 1st line (apparently) (Antimuscarinics like ipratropium are worse than ICS because... reasons unexplained)

What is the management of Small bowel obstruction?

Initial observation and supportive tx for 12-24h If that fails to improve then consult surgery for early surgical intervention

Multiple skin tags (acrochordons) are often associated with what?

Insulin resistance pregnancy Crohn's Disease (perianal)

If a patient has post-extubation stridor, what should be done?

It is likely a sign of laryngeal edema and need to be re-intubated Note: glucocorticoids can be given in a multi-dose regimen prior to extubation to reduce risk of laryngeal edema

NBME5 - Medication to prevent episodes of MS

Interferon beta-1a or monoclonal Ab

What are the main signs and symptoms of neonatal sepsis with meningitis?

Irritability, fever/hypothermia, full fontanelle, lethargy, hypotonia

what is the effect of trastuzumab on the heart?

It causes reversible cardiotoxicity (unlike doxorubicin which is irreversible). Most patients recover after discontinuation of trastuzumab

What is the treatment for an acute hepatitis B infection?

Just let them be apparently according to UW. Even if their liver enzymes are crazy and they are symptomatic just *schedule close outpatient follow up and serial testing* antivirals are only for immunocompromized, concurrent Hep C, or fulminant liver failure (also can't be chronic HepB [only 5% of people get that] until 6 months so no need for liver biopsy or genotyping until then)

UWSA2 - You start someone with HTN and kidney disease on an ACE inhibitor for better HTN control and their Creatinine increases from 2.1 to 2.3 in 2 weeks. What should you do?

Keep them on it With initial ACE inhibitor therapy you can expect to see as uch as a 30% increase in creatinine from decreased GFR

A pregnant lady has Graves disease and is in her 2nd trimester. She is on methimazole. She has some mild hyperthyroid symptoms. Her TSH is 0.1 (normal 0.2-3.0), her T4 is 19 (normal 7.5-18) and her T3 is 290 (normal 105-285) [ranges for second trimester normal], what should you do to her medications?

Keep them the same - apparently you want mild hyperthyroidism or else baby will get hypothyroidism (Apparently if it's mild thyrotoxicosis just let it slide. They don't tell your what the difference between mild or mod/severe are but I guess you'll just know)

Whats does a Paget's disease skull CT look like?

Kind of like Multiple Myeloma to me BUTTTTT apparently those gaps and holes in the skull are thickened calvarium and inhomogenous bone (*cotton wool appearance*)

A 22 y/o college student develops a L leg arterial clot (cold leg, pain, pulseless) with sensation still in tact. The rest of his exam is normal except for a low pitched diastolic murmur in his cardiac apex. Whats the most likely cause?

L atrial myxoma (dude literally has heart cancer -> an embolus instead of Factor V leiden wft?!?) apparently if you have a murmur you have heart cancer

NBME5 - What is preferred DVT prophylaxis for a patient perioperatively?

LMWH (like enoxaparin) (Not sequential compression devices)

What is Ulipristal and how is it different from Levonorgestrel?

MOA: competitive inhibitor of progestins at progesterone receptors Use: emergency contraception Ulipristal can be used up to 5 days post coital and does not lose efficacy during that time (98-99% effective) unlike Levorngestrel (Plan B) which can only be used 72 hours after and loses efficacy as time passes.

What is the presentation of postconcussion syndrome?

MOI: occurs following a Traumatic Brain Injury (TBI) S/S: headache; impaired memory; lack of concentration; hypersensitivity to various stimuli; anxiety; irritability; giddiness; fatigue; depression; visual disturbances; sleep disturbances; vertigo Tx: treat symptoms, not allowed to return to play until ALL symptoms have resolved

How do you figure out if there is bone infiltration from a diabetic foot ulcer?

MRI I guess, I thought maybe that was too expensive for a 'next step' but whatever

How do you diagnose osteonecrosis of the hip?

MRI of the hip? apparently plain radiographs are bad for early disease and can remain negative for months after symptoms appear tx - total hip replacement

How do you confirm Mallory-Weiss syndrome vs Boerhaave syndrome?

MW - Upper GI endoscopy B - esophagography or *CT scan with water soluble contrast*

Upwards dislocation of the lens (sublaxation a.k.a. ectopia lentis) is most commonly associated with what disease? And what is another of the most common complications?

Marfan's Aortic dissection (Also being tall as shit) Berry aneurysm is associated with Ehler's Danlos which is joint hypermobility and hyperextensibility

If a patient presents with symptoms similar to an MI, with hypotension as well and on ECG you see a new onset RBBB with no ST or T wave changes, what is the most likely cause?

Massive pulmonary embolism Symptoms of MI with hypotension look like an inferior MI -> R ventricle failure, but with lack of ischemic changes on OKG (ST elevation, t wave inversion) it's more likely a PE causing R heart failure (oddly enough here it said you would detect the PE by seeing *Moderate tricuspid insufficiency* on an echo and not all the other things like RV dysfunction)

If a 65F has a DEXA scan of -.17 of the spine nd -1.8 of the hip what should you do?

Measure 10y fracture risk (Never give bisphosphonates apparently because god forbid we help a person) (Need to calculate risk before we can give them or whatever)

An asymptomatic 48F comes in with abnormal LFTs. Everything else is normal, but she has elevated AST, ALT and ALP. What is the next best step?

Measure ANA and anti-smooth muscle antibodies as she likely has autoimmune hepatitis

If someone has hypercalcemia with a 45 pack year smoking hx but is asymptomatic, what is the next best step in diagnosis?

Measure PTH to see if it's PTH dependent or independent and to rule out hyperparathyroidism Subsequently may do a CXR (for carcinoma), serum/urinary protein electrophoresis (mor Multiple Myeloma), PTH-related protein (for malignancy), and Vitamin D levels (for grnaulomatous disease or oral intake)

If a 2F has lead level of 12 confirmed with venous lead levels on exam, what should you do next?

Measure levels in 1 month (lead levels between 5-44 is 'mild toxicity' and should be re-evaluated in 1 month... 1 year eval is if their lead is <5

What is the Cobb angle, and what is its significance?

Measured to define scoliosis. > 10° is needed for dx. Moderate/severe (>20???) needs a back brace >40 degrees indicates a need for surgery

If a baby is not pooping and you see a "ground-glass" mass on the R side in the abdomen on abdominal XR, what is the most likely cause?

Meconium Illeus (think Cystic Fibrosis)

What is the most useful measure (mean, median or mode) of central tendency in a skewed graph?

Median, as mean shifts towards the skew

What are the components of Multiple Endocrine Neoplasia Type 2 A and B?

Medullary carcinoma of thyroid (calcitonin) pheochromocytoma (plasma metanepherines) hyperparathyroidism (a) mucosal neuroma, marfanoid (b)

drugs, conditions and diseases that cause digoxin toxicity?

Metabolic - hypokalemia, hypomagnesemia, hypercalcemia Drugs - *Quinidine, Thiazides, verapimil, amiodarone* Diseases - partial AV block, hypothyroidism, myocarditis, WPW syndrome, thyrotoxicosis, renal failure, hepatic failure

What is another beneficial effect of Metformin in certain patients compared to sulfonylureas (glyburide) or thiazolidinediones (pioglitazone)?

Metformin leads to weight loss so it is better in obese patients than sulfoylureas/thiazolidinediones

4 weeks ago a 75 y/o F had coronary angioplasty and stent placement for CAD. today she comes in with a 2 week history of (hyperthyroidism symptoms). On exam she has an enlarged, firm, irregular, nontender thyroid, regular tachycardia, hypertension and hyperglycemia. You give her B-blocker and follow up in 2 days. At that time she says her sxs are only partially relieved. What should you do next?

Methimazole (not radioactive iodine because apparently she has iodine induced hyperthyroidism from the contrast agents used 4 F***ING WEEKS AGO!)

Juvenile Idiopathic Arthritis (JIA) treatment

Mild (1 joint) - NSAIDs and intraarticular corticosteroids. (And strengthening exercises?) Severe (>2 joints and impaired) - same as mild + methotrexate +/- biologic agent

How do you determine prognosis in someone with end stage liver disease?

Model for End Stage Liver Disease (MELD) Score -bilirubin -INR -serum creatinine -Sodium (recently added)

UWSA2 - If a newborn patient has neonatal polycythemia (hct >65% or Hg >22) what is the next best step in management?

Monitor blood glucose as increased RBC can lead to hypoglycemia and hyperbilirubinemia (neonatal polycythemia generally self resolves in 24 hours)

UWSA2 - A 24F with SLE comes in for initial pregnancy visit. She is 12w gestation. She is relatively stable with hydroxychloroquine but still has mild SLE symptoms. She also takes baby-aspirin for "preventing blood clots". What advice should you give your patient?

Monitor you fetus for CVS abnormalities cause by SLE (congenital complete heart block). Neonatal lupus develops with passage of anti-SSA (Ro) and anti-SSB (La) antibodies through the placenta -hydroxychloroquine is actually safe in pregnancy, while SLE is more dangerous -baby-aspirin (81 mg aspirin) is fine as well

case fatality rate vs mortality

Mortality is including the whole population so much smaller CFR includes only the disease/event mentioned

What is the most common type of adverse hospital event in a hospitalized patient?

Most common - adverse drug event second most common - hospital acquired infection

What is the biggest risk factor for PID?

Multiple sexual partners (not previous hx of PID)

Neurofibromatosis type 1 vs type 2

NF1 - Autosomal dominant, chromosome 17 - HYPERpigmentated café au lait spots - OPTIC Gliomas - Cutaneous neurofibromas - Bone dysplasia NF2 - Autosomal dominant, chromosome 22 - HYPOpigmented cafe au lait spots - Bilateral acoustic neuromas (deafness) - schwanomas?

UWSA2 - A 57F underwent TAH for ovarian cancer 4 days ago. Now she has 12 episodes of bilious vomit in 12 hours and has not pooped/passed flatus since the surgery. She has a markedly distended abdomen with absent bowel sounds. XR shows dilated loops of bowel and air in the distal colon/rectum. After you give IVF, what is the next best step?

NG tube (They have pos-op ileus which is managed with supportive tx. IVF, correct electrolyte imbalances, antiemetics, bowel rest and avoiding opiates)

How do you calculate Number needed to treat?

NNT = 1/absolute risk reduction ARR = [c/c + d] - [a/a + b] ARR = control rate - treatment rate

What are the benefits of different lifestyle interventions for hypertension?

NOTE: if the question says 20kg weight los it is better than anything here (12 instead of 6). *SMOKING HAS NO EFFECT ON BP*

UWSA2 - If a kid looks like he has septic arthritis but his joint aspiration only has 10,000 WBC, what should you do as management next? And what is the likely cause?

NSAIDs They likely have transient synovitis

What is the treatment for Subacute (De Quervain's) thyroiditis?

NSAIDs and a Beta Blocker

UWSA1 - What sign on exam is most suggestive of OSA?

Neck circumference >17in (43.2cm) in men or >16in (41dm) in women Although obesity and tongue scalloping (indentations from tongue pressing on molars) are important, neck circumference is the biggest indicator apparently The explanation also says obesity is better in women possibly...

NBME5 - A 31F with chronic fibrocystic breast disease comes in with a cyst that has been present for 3 months that is 3x4cm in RUOQ of the R breast. What should you do?

Needle aspiration (Not wait... aspirate... because it's lasted 3 mo apparently)

Once a patient gets heparin induced thrombocytopenia, what is the recommendation for heparin products after that?

Never ever ever ever ever use them again

NBME5 - If a 6month Male has acute gastroenteritis and Salonella typhimurium is isolated from the stool (subspecies of S. enteritica) what should be the next step?

No abx - most gastroenteritis is self resolving only use abx with severe sxs or immunocompromised

Rabies prophylaxis guidelines

No need if animal can be isolated and monitored by animal control for 10 days OR can be tested for rabies If animal unavailable, acting abnormally, or tests positive: No prev vacc - give rabies Ig 20u/kg injected into wound and vaccine on days 0, 3, 7, and 14 Prev vacc - give rabies vaccine on days 0 and 3

if an XR of the abdomen shows bowel distension with a gas pattern in the colon and rectum, what is it showing?

No small bowel obstruction (would also rule out ileus from opioid overuse)

Are there any vitamins or supplements that are shown to reduce risk of dementia in patients if started at an early age?

None have shown actual benefit

What effects can lyme disease have on a fetus if the pregnant mother is infected?

None if properly treated (amoxicillin)

What is the prognosis of hypertrophic cardiomyopathy?

Normal life expectancy with appropriate therapy Good prognostic factors are asymptomatic and catching it in adulthood (rather than younger which would mean earlier symptoms?)

Does drug induced lupus cause kidney failure?

Not often enough for it to be correct on UW

What is the treatment of symptomatic peripheral artery disease

Note - this is the one time they care about exercise before starting meds apparently

What is the presentation of and treatment of cryptococcal meningoencephalitis in a patient with fulminant AIDS?

Note if they have worsening increased cranial pressures do serial lubar punctures, not IV dexamethazone like in bacterial meningitis Second note for HIV patients - tx lasts 2 weeks, consolidation (fluconazole) lasts 8 weeks, maintenance lasts >1 year of lower dose fluconazole

What is the management of ureteral stones?

Note, if <10mm it should also say D/C patient

Approach to adult cardiac arrest

Note: REVERSIBLE causes of asystole/PEA: -5 Hs: -hypovolemia, hypoxia, hydrogen ions (Acidosis), hypokalemia or hyperkalemia, hypothermia 5 Ts: tension pneumothorax, tamponade (cardiac), toxins (narcotics, benzodiazepines), thrombosis (pulmonary or coronary), trauma Note: pulseless VTACH would require defibrillation

A 14 y/o M is brought in by his mother who complains he has a round back. One exam he has kyphosis (hunchback) that can be corrected with hyperextension. XR shows a 35 degree kyphosis (normal <20). What is the next best step in management?

Nothing, apparently <40 degrees is fine

A 16F presents with back curvature and postural problems. She is tanner stage 5 and has a Cobb angle of 17 degrees. What is the next best step in management?

Nothing, apparently because she's tanner stage 5 she's done growing so you do nothing unless Cobb angle is >40. No follow up required... curve progression is minimal once skeletal maturity is reached

UWSA2 - A 32F at 16w gestations comes in for routine check-up. Her Hg is 10.8 and she has gingival bleeding on light contact with cotton applicator. What should you do?

Nothing, apparently gingival bleeding like this is normal in pregnancy (Mild anemia, is of course normal because it is normally lower in pregnancy, and is normal >10.5)

What is the treatment of tinea capitis?

ORALLLLLL griseofluvin or terbinafine Anti-fungal shampoo (ketoconazole) if it isn't an oral antifungal it won't penetrate the hair shafts

How is management of STEMI similar and different in patient who used cocaine prior to the incident?

Oxygen + aspirin still need to be given Go to PCI (percutaneous coronary intervention) if ST remains elevated HOWEVER - do not use beta blockers, instead use benzodiazepines to decrease sympathetic overload

What should be done for a pregnant woman (19 weeks) with increased Maternal (serum) AFP levels (>2.5 MoMs)?

Obstetric US to look for fetal neural tube defects

If a suicidal patient takes glyburide (insulin secretagogue/sulfonylurea) in a suicide attempt, what medication can be given to raise their blood sugar?

Octreotide (Somatostatin analog that decreases insulin secretion)

What is rosacea associated with?

Ocular manifestations (burning or foreign body manifestations, blepharitis, keratitis, conjunctivitis, corneal ulcers, and recurrent chalazion)

UWSA1 - if a mother gives birth to a stillborn what should be the first thing you do for counseling?

Offer to let them hold the body (apparently it helps with grieving? and decreases risk of depression, PTSD and anxiety)

NBME5 - How does chlamydial pneumonia transmitted during birth present in a baby?

Often they'll have conjunctivitis at birth treated by abx pneumonia won't present until after 4 weeks due to incubation time -staccato cough (cough followed by inspiration) -tachypnea and crackles -peripheral eosinophilia -*CKR -> hyperinflation and bilateral infiltrates*

What is different in the presentation of CAD in young patients vs patients >80?

Older patients are more liekly to experience anginal symptoms other than chest pain -Shortness of breath -lightheadedness -fatigue still need a stress test though

poor prognostic factors in low back pain

Patient hx -patient age, poor baseline, severe self rated pain + mood sx psychosocial factors -psych comorbidity, maladaptive pain behaviors, *poor recovery expectations* medical management -prolonged bed rest, opioid therapy

UWSA2 - When should you do a spontaneous breathing trial for a ventilated patient recovering from ARDS?

Once they can maintain adequate ventilation (pH >7.25) and oxygenation with minimal ventilator support (FiO2 <40%, PEEP <8 cm H20) they should start spontaneous breathing trials

What is the cause of Lichen Simplex Chronicus (Neurodermatitis) and what does it look like?

One or more localized circumscribed lichenified plaques that result from repetitive scratching or rubbing (often associated with anxiety) as occurs with chronic atopic dermatitis or other type of eczema.

How do you treat vaginal trichomoniasis? What if the patient is breast-feeding?

One single megadose of oral metronidazole if breast-feeding discard breast milk for 24 hours after taking medication then resume

If someone is positive for Gonorrhea, but negative for Chlamydia on NAAT of urine. what is the recommended treatment?

Only Ceftriaxone now Apparently because of increasing azithromycin resistance now guidelines are changing

If someone has a herpes zoster (shingles) flare up what should they be treated with?

Oral Valacyclovir 3x/day for 7 days Apparently Acyclovir is never used ever because god forbid someone take it 5 times a day. Heaven forbid these f***ing people take acyclovir ever or the world will implode

How do you treat an anorexic person who is developing refeeding syndrome with SOB, dyspnea and orthopnea along with bibasilar crackles, dullness to percussion to lung bases and pretibial pitting edema?

Oral phosphate (assuming they are already on thiamine) They are developing CHF due to hypophosphatemia

What is the treatment for herpes zoster (shingles)?

Oral valacyclovir if they have had lesions <72 hours

What is Tibial tuberosity apophisitis?

Osgood-Schlatter disease

What is acanthosis nigricans associated with?

PCOS Both of these are never the answer Insulin resistance (Type II DM) Malignancy (gastric carcinoma)

UWSA2 - Someone comes into the ED with diff breathing and cough. She is on azathioprine and prednisone. What should you assume she has?

PCP apparently (Even though her WBC are 12,000 [although 92% neutrophil]) so give TMP-SMX

What is per protocol analysis and as-treated analysis?

PP - A comparison of treatment groups that includes only those patients who completed the treatment originally allocated. Typically overestimates the real effect of intervention on the outcome AT - a subtype of PP where analysis is based on intervention received not by what they were randomized to. Therefore benefit of randomization is lost

A 22M has a midshaft fracture of his tibia and is given a full cast. The next day he has extreme pain in his calf. What physical exam finding is likely also present? What is his diagnosis?

Pain with passive muscle stretch (not loss of distal pulses) It's compartment syndrome, loss of distal pulses isn't until late-stage compartment syndrome

What is circinate balanitis, and what is it a symptom of? What should it be differentiated from?

Painless, superficial ulceration of the glans penis - Manifestation of reactive arthritis Should be differentiated from painless ulcers of syphillis which apparently don't have anything to do with arthritis. Note: Can be a symptom even if treated for chlamydia previously, and may persist for several months

What symptoms are used to determine the likelihood of someone fainting due to vasovagal syncope compared to other reasons for fainting?

Pallor and fatigue following syncope (Nausea, pallor, diaphoresis, and generalized warmth prodrome that may persist after the episode)

A 65 y/o M patient has constant epigastric pain and nausea for 4 weeks, lost 9.9 lbs and was recently diagnosed with DM2 6 months ago (low carb diet). He has a 45 pack year smoking history and mild tenderness to deep palpation of epigastric region. What is the most likely problem?

Pancreatic cancer? You sound like WebMD Apparently 25% of all PC diagnoses come within 2y of a DM2 diagnosis. In particular a thin older patient should raise concern

What is the test for Lhermitte's sign, and what does a positive test indicate?

Patient is long sitting on table. Passively flex patient's head and one hip while keeping knee in extension. Repeat this step with other hip (+) TEST: pain down the spine and into the UE or LE may indicate spinal cord compression and/or upper motor neuron lesion

UWSA1 - What are the EKG findings you would see in ESRD?

Peaked T waves from hyperkalemia (kind of a stretch but okay)

How do you treat a 27F who is 12 weeks pregnant with syphillis and a history of peniccillin allergy (Rash and difficulty breathing)?

Penicillin (Honestly, never choose anything else. I'm retty sure if someone was hospitalized for Pen allergy and was in the ICU for 12 months the correct answer would still be Penicillin) Can't do doxycycine because she's pregnant

NBME5 - If TMP-SMZ is not an option for treatment of PCP, what is another abx to use?

Pentamidine isothionate

UWSA2 - What is the management of acalculous cholecystitis?

Percutaneous cholecystostomy (gallbladder drainage) (if perforation/necrosis then cholecystectomy)

55M with 3-week hx of back pain. Worse after lifting heavy objects and at night, and not relieved by NSAIDs/Tylenol. He is a construction worker with a 40 pack-year hx. He has moderate paravertebral muscular tenderness. What should be done?

Plain X-rays and ESR (wtf?... 32% chose this) Not high-intensity physical therapy (55% chose this) because apparently nocturnal pain and smoking hx indicate cancer Red flags: -over 50 -constitutional sx (weight loss, fever, etc.) -IV drug use, immunosuppressed -*nocturnal pain* -hx of malignancy QID 5760

Someone has HIV and is not on medications. Their CD4 count is 85. they have all of the symptoms of PCP, including diffuse interstitial opacites on CXR. They also have oropharyngeal thrush. However, on induced sputum analysis they are negative for Pneumocystis. What is the most likely cause?

Pneumocystis jirovecii Apparently induced sputum has a sensitivity of 50-90% so if it's negative they automatically still have it and there's no other options ever

A 69M has bilateral shoulder and thigh pain for the past 3 months with morning stiffness. How he has new onset right sided headaches and jaw pain. Hx HTN. On acetamenophin, lisinopril, and chlorthalidone. One exam he has bilateral shoulder stiffness. What is the problem and what test should be run?

Polymyalgia rheumatica -> *giant cell arteritis?* *get an ESR* apparently GCA is associated with PMR, and even if there are no visual changes or fever, apparently this is most likely cause of a localized headache in this population

Psoriasis of extensor surfaces, pitting nails, separation of nails and nail bed (oncholysis), arthritis, morning stiffness and dactylitis are all common symptoms of what disease?

Psoriatic arthritis (generally skin conditions come first, but in 15% of patients arthritis does) Tx - methotrexate

a 19y/o M presents who just started college. He says he's not sleeping well. He wakes up often, sometimes has hypnagogic hallucinations, and falls asleep in class often. What test should be run first, what does he most likely have? Once Tx is initiated he comes in 4 weeks later with sudden onset muscle weakness and falls, especially during emotional outbursts. What is the next best step?

Polysomnography first (no sleep diary) Narcolepsy (Tx - modafinil) He has cataplexy (common in Narcolepsy) - *Tx with Venlafaxine* Cataplexy tx is SSRI's and SNRI's

When a question asks you "what is the likelihood that someone with a disease tests positive compared to someone without the disease testing positive" What the f*** are they asking about?

Positive likelihood ratio Sensitivity/(1-specificity)

So... you forgot Wallenburg syndrome (lateral medullary syndrome)...

Posterior inferior cerebellar artery stroke - loss of pain/temp ipsilateral face/contralateral body, dysphagia/hoarseness

What is the most common cause of postpartum fever, how does it present, and what is the treatment?

Postpartum endometritis Presents with fever >24h postpartum and uterine fundal tenderness. Other symptoms include purulent vaginal discharge, boggy uterus and increased vaginal bleeding *Most significant RF is Caesarean delivery, particularly if done after labor commences or after rupture of membranes* Tx - broad spectrum abx = clindamycin + gentamicin

8 days after a normal vaginal birth a 41F comes in with signs of heart failure and hyperreflexia in her ankles. She has no JVD. What is the cause?

Preeclampsia? (*apparently hyperreflexia is a sign of end organ damage*... Also apparently preeclampsia can occur up to 12 weeks post partum)

What is a normal bereavement reaction in children?

Preschool children (<7 y/o) generall don't know death is final, and may think it is temporary or reversible Older children (>7 y/o) are aware that death is irreversible and may react with sadness, crying, blaming themselves, inability to concentrate, anger, nightmares, *regression to earlier behaviors*, social isolation and school problems Note: Support from parents is usually enough to handle these effects, but psychotherapy can be tested if mood does not improve

Although multiple other questions have said that dating patients isn't entirely unethical, what type of physician should never ever date a patient or be taken out back and shot 500 times in the head because they're terrible people and should be ashamed of themselves?

Psychiatrists because their relationship with a patient is too complicated to ever date one Also, on a side note, f*** UW

What is the management of adjustment disorder?

Psychotherapy is treatment of choice (adjunctive pharmacotherapy can be used for short term rapid relief of impairing symptoms [sleep aid, anxiolytics, etc])

What are primary, secondary, tertiary and quaternary prevention?

Primary - before disease begins secondary - halt progression of disease at initial stage tertiary - taking actions to limit impairments and disabilities as disease has progressed quaternary - limit consequences of unnecessary/ excessive interventions by health care team

A 48F had abnormal serum liver values on life insurance screen. She has been feeling tired but attributed it to menopause (no sx of menopause on interview). Her father died of liver disease at 62, and she has bilateral xanthelasmata. ALP is 620. No gallstones on exam. What is the most likely cause and what would you also see on labs? How is it treated?

Primary Biliary cholangitis -> anti-mitochondrial antibodies Ursodeoxycholic acid (Note: often leads to bone loss/osteopenia)

What are the components of Multiple Endocrine Neoplasia Type 1?

Primary hyperparathyroidism (and adenomas) pituitary tumors (prolactinoma) insulinoma, VIPoma, gastrinoma, glucagonoma

UWSA1 - What is the biggest risk factor for a stroke?

Prior stroke (That's right f*** you it's not HTN like it always is!!!)

UWSA1 - Differentiate Hyper/hypocalcemia and hyper/hypokalemia using EKG findings.

Prolonged QT - hypocalcemia Shortened QT - hypercalcemia, hyperkalemia hyperkalemia - peaked T waves, lengthened PR and QRS Hypokalemia - flattened or inverted T/U waves, ventricular ectopy (Note, high Ca also causes stones, bones, groans and pschy overtones... low Ca causes neuromuscular irritability/tetany, parasthesias, muscle cramps, and perioral numbess as well)

What is the management of akithesia in a patient on a second generation antipsychotic who cannot have their dose lowered?

Propranolol (or other beta blockers?) may also use benztropine or benzodiazepines

What is the characteristic description of lichen planus and how is it diagnosed? Also, what disease is it most associated with?

Pruritic, purple, polygonal planar papules and plaques (6 P's) dx - skin biopsy (not KOH prep) associated with *Hepatitis C* apparently. Especially if other hep C risk factors like IV drug use.

NBME5 - Diffuse ST changes with PR depression signify what?

Pulmonary embolism

Symptoms of fat embolism syndrome

Pulmonary insufficiency. *Neurologic symptoms. Anemia. Thrombocytopenia. Petechial rash.*

UWSA1 - What are the criteria to diagnose a disseminated gonococcal infection?

Purulent monoarthritis OR Triad of -tenosyvitis - inflamm of distal extremities typically -dermatitis (isolated pustules/papules) on hands/feet primarily -migratory polyarthralgia (assymetric)

What is a severe complication of untreated appendicitis or other intraabdominal pelvic infections (diverticulitis)?

Pylephlebitis (infective suppurative portal vein thrombosis) Tx - broad spectrum abx as it is typically polymicrobial

What physical exam findings in a Pancoast tumor (superior pulmonary sulcus) indicate a worse prognosis in a patient?

R lower extremity hyperreflexia (because it indicates spread to the spinal cord) (other options were basically all in this table, only 19% of people got this right QID 5231)

A 4M has been awakening in the night and screaming. He doesn't seem to remember this at all. The family is currently moving houses. What advice should you give the parents?

REM-sleep disorder, he will grow out of it in 1-2 years (if he remembered the dreams it would be because of the move and often he would remember the terrors/waking up)

A 5-week old child with only a Hep B vaccination comes in with a respiratory infection in winter. He has intermittent respiratory pauses and cyanosis. his older brother and father had URIs earlier this week with cough and rhinorrhea. Pt has bilateral basilar crackles in the lungs and intermittent apnea. What is the most likely cause?

RSV influenza would most likely have a fever/body ache in adults

NBME5 - What is the best management for pain from metastitic prostate cancer that has metastasized to the spine?

Radiation therapy (According to nbme it's even better than opiods because it avoids adverse effects)

NBME5 - What is the treatment of supraventricular tachycardia?

Radio-ablative therapy (Adenosis can be used initially to stop it but it's short term. Same with BB/CCB/amiodarone)

What is the most appropriate method to measure cardiotoxicity in someone started on anthracycline chemotherapy agents (doxorubicin, daunorubicin)?

Radionuclide ventriculography a.k.a multigated acquisition (MUGA) to measure EF as closely as possible

What is the purpose of Intention to Treat (ITT) analysis?

Randomization is of paramount importance and deviation from the initial randomization can contaminate treatment comparison. It is attempting to avoid *crossover* and *dropout* e.g. - if sickest patients dropout early even an ineffective tx may appear beneficial if analysis is only performed on those who remained

What is the management of adhesive capsulitis (frozen shoulder)?

Range of motion exercises if no improvement after 2-3 months then glucocorticoid injections if there is still no response consider surgery

What is the management of superficial wound dehiscence?

Regular dressing changes only (separation of skin and subQ tissue but not fascia (e.g. rectus abdominus for abdominal surgery)

UWSA1 - A 58F presents with hypercalcemia from a routine blood test in clinic. She is currently asymptomatic but she is found to have 700 PTH. What is the most appropriate next step?

Regular follow up with DXA, serum Ca and creatinine for monitoring (Surgery not indicated until symptomatic or they develop osteoporosis)

Prehn's sign

Relief of pain when the scrotum is lifted, indicates inflammation of the epididymis. (generally rules out testicular torsion which has absent cremaster reflex)

How would you prevent recurrence in a patient who had an episode of acute pancreatitis when an abdominal ultrasound shows multiple 3-5mm stones in the gallbladder? (mild disease = no organ failure or systemic complications)

Remove the gallbladder within 1 week of symptom onset

NBME5 - A 66M presents with new onset L sided varicocele that does not go away when he lies flat. What is the most likely cause?

Renal cell carcinoma (RCC is blocking the L renal vein which is where L testicular veins drain to, leading to varicocele that won't reduce when recumbant)

A 54 y/o M with diabetes comes in and says he has great eyes. On fundoscopic exam you wee a 8mm brown pigmented lesion that looks raised approx 1mm in the peripheral R choroid. What should you do?

Repeat exam in 2-3 months (likely a choroidal pigmented nevus) If it grows to >10mm in width or 3mm in height need radiation therapy as it has likely evolved into a choroidal melanoma

A 52 y/o M comes in 1 hour after a 30 min episode of substernal chest pressure that lasted 30 minutes and was relieved by nitroglycerin. he is pain free now. currently hypertensive and tachycardic with an audible S4. ECG is normal and troponins are normal. He wants to leave. What do you do.

Repreat ECG and trops? are you kidding me? initial ECG is nondiagnostic in 50% of patients?!?!?! why the hell do we do it then?!?!?! and trops take 6-12 hours!?!? wft?! since when?!?!

What is the treatment for infectious mononucleosis?

Rest and avoid contact sports But if a F*cking question asks you that put *NSAIDs* because apparently it's not rest for some g*ddamn reason (Side note... if their tonsillar enlargement is so bad that they cannot breathe then admit them and give *corticosteroids*)

What is the most likely complication of trans urethral resection of the prostate for BPH?

Retrograde ejaculation (UTIs and urethral stricutres not as common) (ED and Urinary incontinence rare)

UWSA2 - What are Anti-citrullinated protein antibodies (ACPA) indicative of?

Rheumatoid arthritis (more specific than RF)

An R-prime wave in V1 and a widened S in V6 on ECG indicates what?

Right Bundle branch Block

A patient having an MI that began 30 min ago suddenly develops hypotension, lightheadedness, nausea and weakness. His extremities become cold and he is diaphoretic. What is the most likely problem?

Right Ventricular MI (30-50% of acute inferior MI's) -should also see JVD and clear lung fields (which weren't mentioned) Apparently RVMI patients have hypotension and shock and no other heart attack patients do... *Tx with IV fluid bolus to "optimize preload"*

What is the treatment regimen for someone with ADPKD?

Rigorous BP control with ACE inhibitors (and other meds)

What is the treatment for ALS?

Riluzole - decreases glutamate release and/or binding

UWSA2 - A 13month F has profuse watery diarrhea. She has her first hanburger at a cookout and swam in the pool in her uncle's backyard. What is the most likely cause?

Rotovirus (most common enterovirus worldwide and most common in people <2yo) Apparently because it's watery it can't be from the meat (EHEC) and she isn't likely to have any waterborne one because it's a private pool not a public one (prevent with live attenuated vaccination series

What routine tests should be run initially to test for dementia in an elderly person?

Routine - CBC, B12, TSH, CMP Selective - Folate (alcohol use), Syphillis (exposure), Vitamin D (nephrotic syndrome)

Rubella vs. Rubeola?

Rubeola (measles) - oral Koplik's spots (whitish-bluish papules), (cough, coryza, conjunctivits) Rubella (German measles) - Conjunctivitis, post auricular lymphadenopathy

NBME5 - Acute COPD exacerbation treatment

SABA and muscarinic antagonist + *ORAL corticosteroids (Uptodate kind of agrees)* +/- antibiotics inhaled corticosteroids more for chronic management

UWSA2 - A 52F is having severe hot flashes and wants treatment for it. He PMH is significant for TAH complicated by DVT and T2 diabetes. What should be done?

SSRIs or SNRIs estrogen and progesterone (HRT), while first line, are contraindicated with a history of thrombembolism (also CAD, breast cancer and endometrial cancer)

ST elevation of 1.5mm in leads I and aVL with a 1mm ST depression in leads II, III, and aVF indicates what on an ECG?

STEMI

splenic vein thrombosis vs portal vein thrombosis

SVT - Complication of chronic pancreatitis Presents with isolated greater curvature gastric variceal bleeding PVT - causes portal hypertension, splenomegaly, and esophageal varices. It DOES NOT cause histologic changes in the hepatic parenchyma (unlike Budd Chiari syndrome). Ascites is uncommon since liver is not hurt.

NBME5 - What age should women get colonoscopy screening?

Same as men, begin at 50, until 75

UWSA1 - A 43M presents with new onset SOB with exercise intolerance. He also has small joint pains. His PMH is HTN and GERD and takes HCTZ and Amlodipine. You also note telangiectasias on his lips. He has a proinent heave in the lower sternum. What is the most likely cause?

Scleroderma (Check anti-centromere antibodies) Apparently, scleroderma is causing pulmonary fibrosis (SOB, EI) and also heart failure (heave) (honestly take it or leave it with this question, I thought it was just heart failure)

A 36yo African American female presents with sudden onset headache and blurry vision with a BP of 200/110. He has a prior history of Raynauds and heartburn. BUN is 30 and Cr is 2.0. She has bilateral papilledema. What is the most likely cause? How would you treat it?

Scleroderma (systemic sclerosis) -> scleroderma renal crisis apparently Raynauds and GERD together mean it's definitely automatically systemic sclerosis because no one else could have those without having SS... (NOT FIBROMUSCULAR DYSPLASIA) Tx - ACE inhibitors are the treatment of choice for scleroderma renal crisis *THIS IS THE ONLY EXCEPTION TO THE RULE "NO ACEi" IN RENAL FAILURE* -if they have CNS symptoms (papilledema) then also add IV nitroprusside

What is a Pragmatic study design?

Seeks to determine if an intervention works in real life conditions. contrast with an explanatory study which seeks to determine if an intervention works in ideal conditions, and how/why is does/doesn't work (Randomization is considered explanatory as randomization doesn't occur in the real world)

Pityriasis Rosea

Self limiting disease of unknown etiology Presents with a herald patch (single primary patch), Christmas-tree pattern erupts 1-2 weeks later

what statistical value rules out a disease?

Sensitivity (SPin SNout?) High sensitivity means more true positives so if negative means likely not positive

A screening test must have high what?

Sensitivity (SnOut to rule OUT diseases)

Which test (sensitivity or specificity) rules in/out a disease?

Sensitivity rules out (SNout) Specificity rules in (SPin) you dumb sack of s***

What is the formula for a positive likelihood ratio (not PPV)?

Sensitivity/(1-specificity)

How do you diagnose dermatomyositis?

Serological tests -ANA ntibody is the initial test of choice -Anti-Jo or Anti-Mi2 are good for confirmation If Ab testing is negative then you can do Skin or Muscle biopsy

Patients who are critically in and being started on Total Parenteral Nutrition should have what monitored for possible deficiency?

Serum Phosphate (part of refeeding syndrome) IV dextrose -> inc. insulin -> phosphate driven into cells

An IV drug user comes in with recent onset rash, urticaria and abdominal discomfort. They initially had abdominal discomfort and rash only on lower abdomen but it quickly spread to his body and all 4 extremities. He has raised maculopapular rash over his body for the past 24 hours. Further testing reveals recent Hep B infection. What is this reaction most similar to?

Serum Sickness hypersensitivity rxn (type III) A certain percentage of Hep B patients get SS-like reactions most likely due to complement activation by circulating Hep B surface antigen

How would you treat anemia in a 44y/o F with RA on naproxen, enalapril and oral iron supplements? (Hg 8.2, MCV 84, Fe 80 (normal), iron binding capactiy 200 (normal 250-370), Ferritin 300 (normal 10-120), everything else normal)

She has anemia of chronic disease Need to treat underlying condition, so infliximab for the RA

NBME5 - Working in which location increases the risk of developing lung cancer the most? (Coal mine, cotton mill, grain elevator, power company, shipyard)

Shipyard (asbestos) Apparently coal mine and cotton mill cause pneumoconiosis and byssinosis which don't lead to cancer as often, more restrictive diseases)

UWSA2 - What is considered a major risk factor for accelerated renal disease?

Significant proteinuria (>500-1000 mg/day = +2 protein dipstick)

What is the treatment of a thyroglossal duct cyst?

Sistrunk procedure: *surgical excision*, including medial hyoid bone

If someone develops bullous pemphigoid, what is the first thing you should do?

Skin biopsy (Not allowed to start treatment until diagnosis is confirmed) (tx - Steroids, immunosuppressants)

If a 2yo M patient likely has Juvenile Idiopathic Arthritis, what test should you run on them?

Slit lamp examination - because it's associated with uveitis

A 32y/o M comes in with a 12 year history of well controlled schizophrenia with fluphenazine (1st gen). He has many negative symptoms (flat affect, amotivation, social withdrawal, alogia [quiet], etc.) but is interested in finding work. He has no extrapyrimidal side effects. What should you tell him?

Social skills training may be helpful in targeting negative symptoms Switching to a 2nd gen antipsychotic wouldn't do much. It only has perceived benefit for negative symptoms when they're caused by extrapyrimidal effects

A 40 y/o M with very well controlled diabetes bia an insulin pump (A1c 6.5%) and good lifestyle comes in. His blood tests are normal, total cholesterol is 170, LDL is 99, triglycerides are 134. What is the next best step for this patient?

Statin therapy If they have DM and they're >40y/o then they need a statin

2-step approach for screening, diagnosis of gestational DM

Step 2 called *"glucose tolerance test"* not "fasting serum glucose"

A patient with a 2 y history of severe alcoholic cirrhosis and ascites is seen for hematemesis. An EGD sees bleeding esophageal varices. What complication is this patient at most risk of developing int he hospital?

Spontaneous bacterial peritonitis (50% of patients get infections)

What is the management of dumping syndrome after having a partial gastrectomy?

Start a high protein diet with fractionated, smaller, but more frequent meals

If someone is on an antibiotic and develops C diff, what are the next best steps?

Start fidaxomicin and stop offending abx (replace with lower risk abx like TMP-SMX apparently)

A 22y/o primagravida with 12 week intrauterine pregnancy presents to your clinic. She has been on valproate the entire time, as she had no consultation before she stopped OCPs. She says she heard that valproate is bad for a fetus and asks if she should stop it. Last time she tried to switch meds she had seizures that ended up hospitalizing her. What should be done?

Stick with valproate and offer alpha-fetoprotein screening (also high dose folic acid supplementation and anatomy US) valproate is probably the worst drug so NORMALLY you should switch 6 months before trying to get pregnant. HOWEVER, after conception you should never switch antiepileptics as switching normally induces seizures

A patient has signs of Rheumatoid Arthritis in their proximal interphalangial joints. Their serum markers show negative Rheumatoid factor, Cyclic citrullinated peptide antibody and ANA. Their CRP is 55 (normal <8). XR also shows signs of join destruction of their PIP joints. What is the most likely cause and treatment?

Still RA, give them methotrexate I don't get it. They have negative to all the markers except CRP, so they have RA. Positive RF and CCP means aggressive joint destruction, while negative both (seronegative RA) means less aggressive course

If a patient has signs of meningitis and CSF points to bacterial (high leukocytes, protein, low glucose, etc.) but the gram stain is negative, what is the most likely cause?

Still bacterial CSF gram stains are 60-90% sensitive, so they often give false negatives

What si the treatment for Heparin induced thrombocytopenia?

Stop *ALL* Heparin products Start a direct thrombin inhibitor (eg. argotroban) or fondaparinaux (synthetic pentasaccharide) *Wait for platelet count to return to normal before starting Warfarin

*10 minutes following birth* for someone who had gestational DM and had an insulin drip for glycemic control, how should diabetes control be managed?

Stop insulin regimen and order glucose tolerance test at post partum visit really?!?!?! 10 minutes after?!?!?! this whole test, and the one before has made me wanna kill myself. For real. Like the entire time.

What is the most common cause of cellulitis of the skin?

Strep pyogenes

NBME5 - What is the most common cause of supraventricular tachycardia?

Structural defects (Like WPW syndrome defect in His-purkinje system)

2 weeks after a laparotomy a 22yo male patient develops swinging fevers, dry cough and pain on his right shoulder tip. He has tenderness to palpation on the 8th to 11th rib. What is the most likely cause and diagnostic test?

Subphrenic abscess dx - abdominal US

UWSA1 - how do you diagnose Hirchsprung's disease?

Suction biopsy

T wave inversion in leads II, III and aVF one ECG indicates what?

Suggestive of R heart strain from PE, although not diagnostic

UWSA2 - A 3F comes in with a 2 day history of diarrhea that had streaks of blood today. She has abd pain to deep palpation. Several other children at her daycare have similar symptoms. Stool cultures pending, what should you do?

Supportive care (fluid repletion) (She has bacterial enteritis which self-resolves normally with oral rehydration. Avoid antimicrobials because they increase the risk of developing HUS)

What is the management of salmonella enteriditis?

Supportive therapy and observation In children younger than 12 months, immunocompromised people, and adults >50 with atherosclerotic disease: -ciprofloxacin -TMP-SMX -Ceftriaxone

A 23F was involved in a house fire. She has evidence of inhalations (singed facial hair) but her ABG is normal. What is she most likely to develop in the next 24 hours?

Supraglottic edema - low threshold for intubation (causes most deaths in burn wards from inhalation)

UWSA2 - If you suspect ovarian carcinoma and pelvic US seems to confirm it, what should you do for next steps?

Surgical explanation No screening tool for ovarian ca. so just go in and cut it out

A woman comes in 12 weeks pregnant. At 6 weeks on her US a large (11cm) septated mass was seen on the R ovary. On US today the mass is still there and is now 13cm. She has no symptoms. What is the next best step?

Surgically excise it at second trimester Large size (>10cm) increases risk of torsion, rupture or labor obstruction and it is likely malignant (septated) so it must be taken out. At beginning of second trimester the uterus is still in the pelvis, organogenesis is complete and progesterone is no longer needed from the ovary Note: do not biopsy it because that allows it to spread

A patient was recently diagnosed with MDD. He was prescribed Fluoxetine and improved but developed insomnia and jitteriness. What change (if any) should there be in medications for this patient?

Switch to another SSRI (escitalopram) because he's an SSRI responder and fluoxetine is known for causing those side effects more than other SSRI's

8F presents with abnormal behavior. She has been inappropriately laughing, crying and restless/easily distracted for a few weeks. She also has rapid jerking of her feet/hands/face. She has recurrent respiratory infections. She has facial grimacing and tongue fasciculations. She has a II/IV holosystolic murmur at the apex. She also has distinct changes in her voice and pitch. Muscular tone is globally decreased. Pronator sign is positive. What is the most likely diagnosis and treatment?

Syndenham chorea (From rheumatic fever) Penicillin for 10 days

What is the difference between tourrette's, provisional tic disorder and chronic tic disorder?

T - Multiple motor & >1 vocal tics (not necessarily concurrent, >1 year). Age of onset <18 PTD - tics for <1 year, usually only a month or so CTD - one or more motor or vocal tics (but not both) for >1 year

What is the treatment of acute uncomplicated cystits (or UTI) in women?

TMP-SMX for 3 days (get a pregnancy test if they may be pregnant at all) if pregnant give amoxicillin-claulaunate cefalexin or fosfomycin

NBME5 - If a graph has "true positive rate" and "False Positive Rate" on the X and Y axis, what values is it showing?

TPR - Sensitivity apparently FPR - (1-specificity) apparently

If someone is on amiodarone what should you check on them regularly or if they experience edema, weight gain and fatigue?

TSH levels

If you suspect catheter related infection on someone with a Central Venous Catheter in their IJ, what are the next best steps?

Take two blood cultures, one from the CVC and one from a peripheral location Leave catheter in and start Vanco and cefepime (covers staph aureus and gram negative bacilli long term catheters should be emoved if there is evidence of S. aureua, psuedomonas aeruginosa or fungal infection. If they do not require removal, patients should have catheter changed over guidewire once afebrile/hemolytically stable or put on antibiotic lock post dialysis (if ESRD)

how does hyperkalemia in an EKG?

Tall peaked T waves loss of P wave widened QRS with a sine wave pattern

NBME5 - If a patient has HTN and new BPH, what is the best medication for him?

Terazosin (alpha inhibitor for HTn and BPH)

A patient comes in with Raynauds phenomenon and you give them nifedipine as treatment and tell them to avoid cold and environmental factors like smoking. On follow up 1 mo later they are still having issues, what is the next best step?

Test for underlying causes (diseases) -ANA, RF, CBC, blood chemistry, Urinalysis, complement levels

Kleihauer-Betke test

Test that detects the presence and amount of fetal blood in the maternal circulation when there's a r/o Rh isoimmunization can be used to determine the amount of Rh(D) immune globulin administered to decrease the risk of alloimunization

What is the treatment for chronic prostatitis/chronic pelvic pain syndrome (note: not chronic bacterial prostatitis which is the ddx but has bacteria on UA and culture)?

The 3 A's -antibiotics -anti-inflammatories -alpha-inhibitors (in ? stated as "drugs used for prostate enlargement")

UWSA2 - What is the treatment of anorexia nervosa?

medically stabilize, CBT, pharmacotherapy if depressed (SSRI, atypical AP) UWSA2 says no SSRI's, instead olanzipine) (HOLY SHIT UPTODATE AGREES!)

A 60 y/o M patient has signs of an MI at home. In ambulance he has nitroglycerine 2x with significant relief of symptoms. ECG shows (you had to read it) ST depression in V3-6, aVF and lead II) with no ST elevations. He was given aspirin in ambulance as well. What is the next best step?

anticoagulation, second antiplatelet and close monitoring (not cardiac cath) (also statins and beta blockers?) Crdiac cath is only for ST elevation this presentation is apparently either an NSTEMI or ACS but more likely ACS?

What is the treatment for urgency incontinence (involuntary urine leakage after strong urge to urinate)?

bladder training with timed voids if that fails antimuscarinics (tolterodine, Solifenacin, oxybutynin, etc.)

what is the major side effect of ginkgo biloba?

bleeding/platelet dysfunction

A 68F is experiencing hyperthyroid symptoms [a fib, osteopenia, weight loss, enlarged and tender thyroid] and has TSH <0.001 and free T4 of 64.7 (normal 0.9-2.4). What is the next best step in treatment?

block sympathetic hyperactivity with beta blockers?... Apparently "inhibit thyroid synthesis" is wrong because "It can take up to 2 weeks for this to take effect" so you treat other things first. I don't know what UW wants from me...

UWSA1 - What ventillator setting can decrease mortality in ARDS patients?

low tidal volume ventillation

How do you differentiate chikungunya and lyme disease based solely on presence of arthritis?

lyme - often only one joint or two large joints like the knees chikungunya - polyarthralgias

A 28 y/o M patient has a 3 month history of nonproductive cough and chest tightness (When he exercises in the morning and also sometimes at night), 5 pack year history although he quit 3 years ago, and good exercise history. He also has an inducible cough on forced expiration. What is the most likely cause?

cough-variant asthma GERD and Upper Airway Cough Syndrome are the two most common causes of chronic cough. GERD is typically related to meals and also has heartburn and reflux. UACS typically has rhinorrhea and oropharyngeal cobblestoning.

Which is performed in an emergency setting, a cricothyrotomy or a tracheostomy?

cricothyrotomy

what type of study design is meant to uncover prevalence?

cross sectional

A 60 y/o M presents with a past history of 30 years of mild goiter and new onset dysphagia and anterior neck swelling. He also has fevers, night sweats and weight loss. Prio smoking hx and 2 family members with history of hypothyroidism. He has facial plethora and venous engorgement of neck veins on lifting hands above head for 20 seconds. TSH is 26 (high), T4 is 0.4 (low) and he has high anti-TPO antibodies. What ia the most likely cause?

Thyroid lymphoma He likely had Hashimoto's for years (goiter) which is a risk factor for Thyroid lymphoma

Patients who drastically increase their excercise regimen (such as marathon runners in training) can sometimes experience sharp and bothersome pain during exercise in their lateral foot/tibial border. What is the cause, and what is seen on X-ray?

Tibial stress fracture Initially nothing is seen on x-ray After up to 4 weeks, you may develop bone sclerosis, cortical thickening, periosteal elevation or visible fracture lines

Tick paralysis

Tick paralysis is a RARE toxin-mediated cause of acute flaccid paralysis. transmitted by tick over 4-7 days and presents very acutely. Sx - brief prodrome of fatigue/parasthesias -> gait ataxia + ascending paralysis *over 1-2 hours* -> absent deep tendon reflexes RX: Tick removal results in rapid improvement of all symptoms.

What are the main symptoms of Mixed cryoglobulinemia syndrome, the blood levels of factors involved, and how do you diagnose it?

Triad of -Palpable purpura + Weakness + arthralgias -Also will see *renal failure* -Elevated rheumatoid factor -hypocomplementemia Dx - serum cryoglobulin levels (*ASSOCIATED WITH HEP C!!!!!*)

What is the treatment for anogenital warts (chondyloma acuminata)?

Trichloroacetic acid

What nerve gives sensation to cornea of eye?

Trigeminal

How do you differentiate between viral (HCV), bacterial and fungal infectious keratitis of the eye?

Viral - generally HSV, *branched/linear lesions, watery drainage,* decreased corneal sensation (Tx - topical or oral antiviral) bacterial - generally Staph aureus, central round infiltrate and positive gram stain, contacts, mucopurulent drainage (Tx - topical abx) fungal - generally candida, multiple lesions with feathery borders, immunocompromized with trauma to eye from dirty soil, gradual progression (Tx topical or oral antifungals)

What does a barking cough in a child mean?

Viral Croup (Stop thinking of pertussis you dips***)

UWSA2 - If a 7M has a 6 day history of fever (102.6), pharyngitis, myalgias and difficulty walking, and he has toe walking on exam and a CK of 2,000 IU/L, what is the most likely cause?

Viral myositis (tx - supportive, hydration) (muscular dystrophy is more chronic onset, 6 days isn't enough time)

NBME5 - Ingestion of what can cause an FOBT to be negative?

Vitamin C

Lobar carcinoma in situ treatment

cut it out

NBME5 - When should you be concerned for late postpartum hemorrhage in a woman after she gives birth?

When she soaks through a pad in <1 hour OR the bleeding persists for longer than *8WEEKS?!* -Note: Uterine size may stay large during this time as well. This question had a 12w size uterus 2 weeks after birth which seemed like a long time for me (That's such a long time to wait)

If someone has hyperglycemic hyperosmolar state and they are started on IV insulin as treatment, when would you switch their insulin up and what would you switch it to?

When they are <200 mg/dL Blood Glucose you can switch to Subcutaneous regular insulin, but you have to supplement with basal long acting insulin (e.g. detemir, glargine)

How does the likelihood ratio work?

The bigger the number the more likely to rule in the disease (for positive?) if a + result The smaller the number the more likely to rule out a disease (for negative?) if a - result

How does MS affect birth and pregnancy?

The chances of assisted delivery are higher in MS patients

What condition is dermatomyositis most often associated with?

malignancy paraneoplastic syndrome of malignancy to be precise. Anyone diagnosed with dermatomyositis should be screened for age appropriate cancers (most commonly adenoca. of the cervix, ovqaries, lung, pancreas, bladder, stomach) Dermatomyositis may resolve completely if cancer treated successfully

NBME5 - A 15M needs a kidney transplant. His mother (48), Sister (8) and brother (6) are all matches for blood type and two antigens. He has an aunt (36) who is compatible blood type but no antigens. The father is unknown. Who is the best donor?

The mother. The sister and brother are too young to give informed consent (<18) and the aunt doesn't share antigens with him

If someone was in a house fire with significant smoke inhalation and their venous lactic acid is 10mg/dL, what is the next best step?

measure carboxyhemoglobin (for CO poisoning) I'm not sure what the baseline venous lactic acid is, but 10mg/dL is "very high"

UWSA1 - Can a physician initiate an organ donation conversation with a family whose inpatient family member just died?

They can if they are specifically trained to do so

UWSA2 - What hapens to TSH levels as you age?

They go up. So an elderly person has normal TSH if it is <7 apparently

If a patient has a presentation of CAD and undergoess stress testing, which is negative, what does this mean for their prognosis?

They have a "low risk of cardiac events in the near future"

If a patient has a BCG vaccine and gets an interferon gamma release assay (IGRA) that comes back positive, what does it mean and what should be done?

They have a positive TB infection (only skin test is affected by BCG vaccine Get a chest XR -if negative tx with isoniazid for 9 months -if positive sputum culture for mycobacterial species then RIPE

If a sputum acid-fast bacillus smear comes back negative on a patient with a cavitary lesion on CXR, but their PPD shows a 15mm induration, what would you tell your patient about their current TB status?

They may have active TB and should await further testing AFB smears are the most rapid and least expensive tests, but cannot rule in/out the diagnosis due to low sensitivity and poor differentiation (other mycobacterium) Should still do (one or both of these): mycobacterial culture and a Nucleic Acid Amplification testing

What is different in the diagnosis of appendicitis in a pregnant patient?

They may not have typical signs like McBurney's tenderness as appendix is displaced 1st line imaging is a *graded compression abdominal US* and if that is inconclusive 2nd line is an MRI (CT is too much radiation for baby)

UWSA2 - What do you do if a patient has a post-oerative pleural effusion after a CABG procedure?

They occur in ~60% of patients and if left alone will self resolve If large (>25% of the hemithorax), symptomatic, late onset (>30days post CABG) or R sided in the abscence of HF then get a diagnostic thoracocentesis

What is the size comparison between an RBC and a normal lymphocyte?

They should be similar sizes

What is internal validity?

This type of validity is focused on determining whether a study's findings are accurate, or are more the result of the influence of extraneous variables. (major threat is confounding apparently)

What physical exam finding can confirm complete achilles tendon rupture?

Thompson Test - squeezing calf does not produce foot flexion

If a patient has a bicuspid aortic valve and nothing else wrong, what should they be tested for?

Thoracic aortic aneurysm - because incidence is 20-80% of bicuspid valves you always test even if they're asymptomatic

UWSA2 - How do common ventilator settings affect the patient with ARDS and what do you change for which situation? (Tidal volume, resp rate, PEEP, FiO2, plateau pressure)

Tidal volume - low tidal volume is lung protective, but also leads to CO2 retention. However this is acceptable to some extent to prevent barotrauma (permissive hypercapnia CO2 >45) respiratory rate - can only be increased so much PEEP - The best thing to increase for ARDS, can go up to 15-20 cm H2O for adequate oxygenation FiO2 - levels >60% for prolonged periods can lead to O2 toxicity so often must be lower plateau pressure - (trick) not a set parameter but a measured value for lung compliance. Should be <30 to prevent barotrauma. PEEP may increase this value

how to stop epistaxis

Tilt head forward & pinch nose and push it into their face; hold for about 15 minutes; could be more if patient on Warfarin/Heparin *If that fails, place a cotton pledget with a topical vasoconstrictor (Oxymetazoline)*

What is the meaning of red cell distribution width (RDW)?

measure of size variation of RBC's High RCDW could mean iron deficiency anemia? (at least is apparently can in a newborn)

What is the treatment for bacterial conjunctivitis and when does it differ?

Topical macrolide (erythromycin) polymyxin-trimethoprim drops azithromycin drops *IF A CONTACT USER - TOPICAL FLUOROQUINOLONES*

UWSA2 - An 82M comes in after a recent syncope. He doesn't have any dizziness or memory loss from the event. ECG shows RBBB and left anterior fascicular block. What is the most likely cause?

Transient AV block (Not VTach because he has no history of MI or heart disease)

Viral meningitis management

Treat the symptoms- Tylenol (acetaminophen) for headache, pain, hydration, position for comfort; antibiotics are used till confirmed its viral

UWSA2 - What pregnancy complication are monochorionic twins more susceptible to>

Twin-twin transfusion syndrome (monoamniotic can get cord entanglement)

Which type of error is affected by the *power* of the study (sample size)?

Type II error (false negative)

If you suspect lupus nephritis, what is the next best step?

US guided needle biopsy There's multiple types and you need to know which one to manage appropriately: -1 and 2 - not therapy needed -3 and 4 - glucocorticoids and cyclophosphamide -5 - may need immunosuppression -6 - immunosuppression not recommended as it's an advanced sclerosing disease

What are the conditions and tenents of the family medical leave act?

Up to 12 weeks of unpaid leave (where job is protected during that time) that can be continuous or intermittent >Half-time employee employed >1 year at employer employer has >50 employees covers - serious health conditions, childbirth and newborn care, and care of spouses/children/parents

How do you diagnose a urethral diverticulum?

Urinalysis Urine culture *MRI of the pelvis* Transvaginal US

If you have a 6 y/o boy who wets the bed at night, but not the day (everything else normal) what is the next best test to run on them?

Urinalysis (not fingerstick?) Also, enuresis is defined as urinary incontinence at age >5y/o Renal and bladder US indicated if they also wet the bed at daytime

NBME5 - A 4.5F presents with bed wetting. She has a normal UA with rare bacteria and 1-2 WBC/RBC. What should you do next?

Urine Cx and sensitivity (apparently lots of kids present without pyuria on UA)

ABO hemolytic disease of the newborn

Usually occur in type O mother with a type A or B fetus. Can occur in the first pregnancy. neonate present with jaundice within 24hrs of birth. Will see Unconjugated Hyperbilirubinemia. Tx phototherapy or *transfusion exchange*

What antibiotics should be given for a CF patient with pneumonia?

Vancomycin + *cefepime and amikacin* (covers S. aureus and pseudomonas aeruginosa) cefepime/ceftazedime - pseudomonal amikacin/tobromycin (aminoglycosides) - also pseudomonal

UWSA1 - A 22M patient tells you he would never want to have any extra procedures done to him (like ventiillation and tube feeding) in office. 2 weeks later he has a massive stroke and his mother comes in and tells you to ventillate him. What do you do?

Ventilate him. Apparently you need to have it in writing, OR have 2 witnesses for the event of an oral advanced directive

What is the preventative treatment for cluster headaches?

Verapimil (sumatriptan is abortive?? but has bad side effects I think) start at 240mg and titrate up -sometimes add topirimate -100% oxygen is also good for acute attacks

UWSA1 - how can a urine chloride level help you distinguish between bulemia (self induced vomiting) and other causes of metabolic alkalosis?

Vomiting - kidneys increase Cl absorption (low urine Cl) Diuretics - Na/Cl reabsorption blocked (high Cl) Na wasting disorders (Gietlman/Bartter syndromes) - Na wasting leads to high Cl levels as well (high Cl)

If the initial XR for a scaphoid fracture is negative what should be done?

Wait a week and redo the XR -never ever ever refer someone to a specialist though because f*** getting someone who knows what they're doing in there when you can just let their bone die on it's own (*Apparently the correct answer is to MRI the wrist but that wasn't an option*)

NBME5 - An 8M has tourrette like symptoms developing with grunting and outbursts. What is the most likely clinical course of the patient?

Waxing and waning course throughout childhood/adolescence (Many patients improve in teenage years and remit by adulthood)

If a patient who was hypertensive has been well controlled for 2 years on low dose medication and did well with diet/excercise and weight loss asks to be taken off their meds, what do you tell them?

We can try to wean you off but you'll have to start taking medication once every other day

Alright... You've got a 12 y/o M with PSGN. He's already one antibiotics, mildly hypertensive and signs of kidney damage on urinalysis. Cr 2.5. What should you give to him as treatment? (Spoiler alert it's dumb)

Well, because he has mild HTN he definitely has volume overload so you have to give him *furosemide* I hate my f***ing life

If someone has Medullary Thyroid cancer with no other symptoms at all, completely healthy otherwise and they also get genetic test and see RET mutation (MEN 2A/B?), what is the next best step?

measure plasma free metanephrines apparently cutting out the cancer isn't the important thing it's checking for a disease that has no symptoms so likely isn't the biggest problem...

Is someone with a shingles infection infectious even if they're on oral valacyclovir?

Yes, apparently they are still infectious

How would you differentiate between a Wilms tumor and a neuroblastoma in a 2y/o child who's parents found an abdominal mass while bathing him?

Wilms tumor - painful unilateral (renal) abdominal mass with hypertension and hematuria (age <5y/o) neuroblastoma - most common extracranial tumor in childhood. Painless adrenal mass, hypertension, flushing, sweating

When should you get percutaneous coronary intervention in a possible MI?

Within 90 minuted of *FIRST MEDICAL CONTACT* not within 90 minutes of symptom onset... if you can't mange that then fibrinolytics

Inheritance of hemophilia A

X-linked recessive

What are the main characteristics of and inheritance of Kallman syndrome?

XLR inheritance of variable mutations, incl some with mutation of KAL gene (important for neural migration Clinical feats (2 main): - Anosmia (abN development of Olfactory lobe) - Hypogonadotrophic hypogonadism (central d/t neurons secreting GnRH migrate from Olfactory placade to hypothalamus) - testes usually < 3 mL volume, infertile and failure to start/complete puberty Other feats: - cleft palate - unilateral renal agenesis - cryptorchidism - synkinesis (miror moving of hands) - congenital deafness Ix: - decreased FSH/LH - delayed bone age

UWSA2 - A study is approved by the IRB at an institution. Halfway through the study the researchers want to add a quality-of-life questionnaire to their study. Do they need approval from the IRB?

Yes, any change at all needs approval

What is the presentation of Anomalous aortic origin of a coronary artery (AAOCA)?

Young patient (<35) with sudden cardiac death (like HOCM) -exertional angina -lightheadedness -syncope (Big note, no heart murmurs on exam, unlike HOCM) QID 5574 19% got it right

What is the cause and presentation of Ecthyma gangrenosum?

a cutaneous necrotic disease associated with Pseudomonas bacteremia. These infections are common in patients who a neutropenic (chemotherapy, are hospitalized, have burns, or indwelling catheters.) normally presents as painless red macules that rapidly progres to pustules/bullae and quickly evolve into gangrenous ulcers with raised violaceous margins tx - with empiric IV antibiotics (pipercillin-tazobactam + aminoglycosides [gentamicin])

Korsakoff's syndrome

a disease that afflicts long-term alcoholics, leaving some abilities intact but including hallucinations and a tendency to repeat the same story, or stories that never happened vitamin B1 deficiency *neuroimaging may reveal problems in mammillary bodies*

What does Sebhorrheic Dermatitis look like and how is it treated?

a form of eczema that primarily affects oily areas of the face (scalp, central face, ears, chest) Erythematous, pruritic plaques with greasy scales Tx - topical antifungals (*selenium sulfide*, ketoconazole) -topical glucocorticoids - topical calcineurin inhibitors (pimecromilus) -keratolytic agents (salicylic acid) -coal tar shampoo *NOTE: It is chronic and remitting and will need treatment every 1-2 weeks (topical ketoconizole or ciclopirox)*

what even are quality adjusted life years?

a measure of how many years of high-quality life a person lives, measure of burden of diasbility in a patient (e.g. every 5 years like this is worth one year healthy)

ovarian hyperstimulation syndrome (OHSS)

a syndrome that presents sonographically as enlarged ovaries with multiple cysts, *abdominal ascites, and pleural effusions* (May have organ failure, DIC, hypercoagulability and electrolyte imbalances as well) often seen in patients who have undergone ovulation induction after the administration of follicle-stimulating hormone or a GnRH analogue followed by hCG ddx - ruptured ovarian cyst - which has acute abdomen (rebound, guarding) from hemoperitoneum, and anemia

if someone with UC has fever, tachycardia, decreased appetite and "tympanitic" sounds to percussion, what test should you run first and why?

abdominal XR to check for toxic megacolon

Painful third trimester vaginal bleeding with rigid, tender uterus means what?

abruptio placentae

UWSA1 - When do you give activated charcoal vs N-acetylcysteine for acetaminophen injection?

activated charcoal - within 4 hours after injestion A-acetylcysteine - 4-24 hours after injestion

What are the main complications for a donor for kidney transplants?

acute - DVT, infection chronic - Gestational complications (for women) [fetal loss, preeclampsia, gestational diabetes and hypertension] (no increased risk for ESRD or change in life expectancy

What is the difference between acute/chronic mitral regurgitation? What is the difference between primary and secondary Mitral regurgitation? What is impaired LV function?

acute - Severe MR without compensation. May have pulm edema, hemodynamic collapse or cardiogenic shock chronic - Severe MR in presence of associated symptoms (e.g. dyspnea, HF...) or echo fingings (LA or LV enlargement, regurgitant jet prominence) Primary - intrinsic defect in mitral apparatus Secondary - disease of the LV affecting the mitral valve impaired LV function is <60% in patients with MR, even though it is <50% in normal patients

UWSA2 - WBC casts and kidney injury suggest what etiology for nephritis?

acute interstitial nephritis

What is alcoholic ketoacidosis?

acute metabolic acidosis that typically occurs in people who chronically abuse alcohol and have a recent history of binge drinking, little or no food intake, and persistent vomiting Blood glucose <250 but ketones in the urine and possible recent drinking (limited consciousness)

At what age in a male child does the teste need to descend by before you do orchiopexy?

after 6 months it's abnormal so need to refer for surgery (needs to be done before 1 year)

UWSA2 - A mother has intrauterine fetal demise at 30 weeks. She asks if she can just wait for labor to deliver the baby then. What should you tell her?

after IUFD women have ian increased risk for coagulopathy after a few weeks of fetal retention (Does not increase risk of infection or Rh alloimmunization)

When do babies no longer require feeding every 2-4 hours?

age 4-6 months

What is the presentation of and treatment of Rubeola (measles)?

airborne prodrome: (*Cough, coryza, conjunctivitis*, fever, koplik spots) Maculopapular exanthem (Cephalocaudaul [head to extremities] and centrifugal spread) spares palms and soles Tx - vitamin A for hospitalized patients (otherwise supportive)

Which is a bigger risk factor for colon cancer, current alcohol use or prior smoking history?

alcohol (2-3 drinks/day is moderate while >4 is highest risk)

If you type and screen a patient for blood transfusion and they fail, what is the most likely cause?

alloantibodies - immune Ab's produced following exposure to foreign blood material. Exposure to "non-self" antigens from the same species Antigens E, L and K are most common antigens

attributable risk percent

among cases that are exposed, it is the percentage of those cases attributed to the exposure. It is assumed that the exposure is a cause of the outcome. ARP = 100 x (RR-1)/RR ARP = (risk exposed - risk nonexposed)/(risk exposed) x100

UWSA1 - Antibiotics for human bite

amoxicillin-clavulaunate

What is Blephorospasm and how is it treated?

an acute form of dystonia -> involuntary eye closure provoked by external stimuli (bright light, irritants) Tx - botulinum toxin injections

What medications should be given to patients with Chronic granulomatous disease as prophylaxis?

antibiotics - TMP-SMX antifungals - ipratropium

What is the management of chlamydial arthritis?

antibiotics and NSAIDs

What is an Analysis of Variance (ANOVA)?

analysis of variance test used for designs with three or more sample means

Based on the time after transfusion, what is the most common transfusion related reaction for each time period? What prevents each of them?

anaphylactic - washing cells (get rid of IgA for IgA deficient patients) acute hemolytic - careful type/crossmatching Febrile non-hemolytic - leukoreduction

What is the management of asymptomatic bacteruria in pregnancy?

anitbiotics for 3-7 days then follow up urine Cx Nitrofurantoin (5-7), amoxicillin/amoxicillin-clavulaunate (3-7), fosfomycin (single dose) and apparently cephalexin for 5 days even if it isn't listed

If someone has ED without daytime or nighttime erections, what should you do?

ankle brachial index? (Apparently testosterone deficiency when it leads to ED you still have nighttime erections so that has to be PAD)

32F complains of R eye redness and pain for past 2 days. She also has blurry vision. Exam shows conjunctival injection, especially around the cornea. R pupil is constricted and sluggishly reacts to light. Light in L eye results in pain in the R eye. patient also has small tender nodules on the shin and bilateral hilar lymphadenopathy on XR. What is the most likely cause of eye pain?

anterior uveitis (complication of sarcoidosis)

UWSA2 - A 55F comes in with post-coital bleeding. Her last menstrual period was 1 year ago. Vaginal mucosa is pale and thin. FH includes epithelial ovarian Ca in maternal aunt. What should you do next?

any postmenopausal bleeding warrents *further evaluation with pelvic US* (and if very suspicious endometrial biopsy) Only prescribe topical estrogen if cancer is ruled out

What is the most likely cause of death in an MVA with impact on the steering wheel?

aortic rupture

A 25F comes in with a UTI. In the past 7 months she has had 3 UTIs not including this one. She uses spermicide for contraception and always voids after intercourse. You give her 1 week course of antibiotics, what is the next best step in management?

apparently it's postcoital antibiotics... (if it is unrelated to sex then do daily antibiotics) Those both seem crazy

What are pearly penile papules (PPP) and how do you differentiate them from HPV warts (condyloma accuminata)?

arise in 2nd or 3rd decade of life and are a *normal variant* >1 row of small, flesh colored, dome-topped or filiform papules on the penile corona tx - reassurance (differentiated from HPV lesions by look - HPV are verrucous, papilliform growths, diagnosed with skin shave and tx with topical imiquimod)

UWSA1 - What test can determine if there is need for different management of PCP in an HIV user?

arterial blood gasses -if pO2 is <70 or an A-a gradient of >35 on room air you need to give *corticosteroids* as well

how do you differentiate the different congenital infections of a baby based solely on symptoms?

aside from things like hepatosplenomegaly, blueberry muffin rash (purpuric lesions), and hearing loss (which are seen in most if not all *Rubella* - triad of sensorineural hearing loss, *cataracts and patent ductus arteriosus* *CMV* - periventricular calcifications *Toxo* - intracerebral *HSV* - hydrops fetalis, intrauterine fetal demise, meningitis, characteristic lesions *Syphilis* - (NO heart of eye abnormalities), initially asymptomatic -> hepatomegally, jaundice, rhinitis, Maculopapular rash (not purpuric)

UWSA1 - At what age is it more favorable to give a post-menopausal woman HRT for symptoms of menopause?

at age <60 women have a much lower chance of thrombembolism, breast cancer, CAD and stroke and once theyhit 60 (most of these are age related effects to HRT) They actually decrease all cause mortality in women <60

What is a non-addictive ADHD medication for adults?

atomoxetine buproprion tricyclic antidepressants

von Hippel-Lindau disease

autosomal dominant -*cerebellar and retinal hemangioblastomas* -*Pheochromocytoma* -*Renal Cell Carcinoma (clear cell subtype = cysts)*

What is a complication of bacterial conjunctivitis?

bacterial keratitis (especially if they wear contact lenses or have decreased immunity [corticosteroids]) *refer immediately to an opthamologist if the patient develops this (feeling of foreign object in eye with difficulty opening, photophobia or impaired vision)*

what is the timeline for breastmilk jaundice?

begins after 3-5 days, *PEAKS* at 2 weeks old

A patient with a psychotic disease is relapsing after a 4 month period of not having his sustained long acting injectable antipsychotic. He is brought in by his mother and has new Catatonia on exam. What should you do for the patient?

benzodiazepine (lorazepam) is the treatment of choice (or electroconvulsive therapy) If benzo resistant or if it is an urgent clinical situation you may use ECT

nonclassic congenital adrenal hyperplasia

big note is increase in bone age. Can be confused with idiopathic premature pubarche in boys but bone age is high in this and also they get acne.

What does the EEG in a child with Juvenile Myoclonic Epilepsy (develops into genetic generalized epilepsy?) show?

bilateral polyspike and slow wave discharges

If you see a 1.3cm lesion on a 75M patient's hand that looks like actinic keratosis, what should you do?

biopsy it...? apparently if they are >1cm their risk of being SCC drastically increased (If <1cm just liquid nitrogen [cryotherapy] it)

A 71 y/o M presents with a TIA 3 hours ago that lasted 1 hour. Hx of HTN and smoking (60 pack years) stopped 8 y ago. On exam he has a bruit in the L neck. BMI 33. He has 75% stenosis of L internal carotid aa. Echo shows LV hypertrophy and mild LA hypertrophy. He also has a patent foramen ovale. He is started on aspirin and a statin. What else should be Done?

carotid endarterectomy? BUT HE'S ONLY 75%! I'VE GOTTEN THIS WRONG FOR THIS EXACT REASON! apparently it's chaged so now >70% with symptoms you need it f*** everything

If you're trying to figure out if an outbreak of hemorrhagic diarrheal illness was caused by seafood at a certain restaurant, which study design would you most likely use?

case-control study? I'll never understand this... The argument is that you know who was infected so now you "look back at risk factors"

malignant otitis externa

caused by pseudomonas. - intense pain, erythema and edema of external canal, purulent discharge, *granulation tissue at floor of canal* RF - elderly, DM and immunocompromised. Do a CT scan, emergent ENT consult, admission. IV abx with: IV aminoglycoside + anti-pseudomonal PCN, cephalosporin or fluoroquinolone

What is syringomyelia and how does it present?

cavitary exapnasion of spinal cord--destruction of gray and white matter adjacent to the central canal. see areflexic weakness in the upper extremities and dissociated anesthesia in a cape distribution causes: trauma, inflammatory spinal cord disorders, tumors or idiopathic

A 30y/o female is at 7 weeks gestation. She was diagnosed with HIV 3 years ago and last pap 4 years ago was normal. She has vaginal bleeding and on speculum exam you see a 3cm firm irregular, white mass that bleeds when touched. What should you do as the next step in management.

cervical punch biopsy apparently you do it even if they're pregnant? they don't explain that at all

If you see assymptomatic gallstones and patchy gallbladder wall calcification what are the next best steps in the patient?

cholecysectomy - apparently she's got porcelain gallbladder and is gonna get cancer from it

what is Enteropathy-associated T cell lymphoma?

chronic, untreated Celiac disease presents kind of like colon cancer, B symptoms, weight loss, blood in stools, anemia, etc.

What is the most common pathogen grown from foreign bodies in the eyes?

coagulase negative staph

What is first line treatment for insomnia?

cognitive behavioral therapy (not zolpidem or any of the hypnotic medications... also even though they're 2nd line use with caution in the elderly)

What is a pancreatic pseudocyst and what is the treatment?

collection of debris, fluid, pancreatic enzymes, and blood as a complication of acute pancreatitis (often 4 weeks post AP and are walled off) occurs in up to 10% of people after AP and can cause epigastric pain lasting months after pancreatitis if they have significant symptoms can get surgical or endoscopic drainage but mostly just supportive care

A 32 y/oF has comedonal acne that flares up during her menstrual cycle. She also eats a lot of chocolate during this time. She is currently on topical clindamycin, benzoyl peroxide and topical tretinoin. What is the best recommendation for controlling her acne?

combined estrogen/progestin oral contraceptives

How would an infant have infantile botulism?

commonly found in honey, but also *if they live on a rural farm from dust/soil* and lack of protective enteromicrobiota that would fight clostridium botulinum (it is a neurotoxin that inhibits release of acetylcholine)

What is the most common cause of trigeminal neuralgia?

compression of the nerve root (vascular compression, Multiple Sclerosis plaque, etc)

What symptoms are possible with damage to your non-dominant parietal lobe (most often R parietal lobe as most people are left dominant)?

confusion construction apraxia - difficulty copying simple line drawings (like a matchstick) dressing apraxia - difficulty wearing clothes (putting on jackets/pants)

talipes equinovarus

congenital deformity of one or both feet in which the foot is pulled downward and laterally to the side (internally rotated); also called clubfoot *It can be corrected by applying molding casts to the feet.* Usually most effective in first few weeks of life, can be corrected throughout infancy and in most toddlers

What is the strongest protective factor against suicide (b/t no prior hx of suicide attempts or connection to family)?

connection to family?!?! f*** UW apparently that not protective it's just not a risk factor

What is the treatment of breastmilk jaundice?

continue feeding normally (only do phototherapy if the levels continue to rise)

What is the first line contraceptive in adolescents?

copper containing IUD that's bold

How should you handle insulin dosages in a hospitalized patient with type I DM on insulin glargine (basal) and insulin aspart (short acting)?

decrease glargine (25-50%) and give short acting based on glucose levels (because of decreased intake, basal insulin must be adjusted but is still needed)

What do you see on radioactive iodine uptake in Subacute thyroiditis (De Quervain's)?

decreased update Low TSH -> decrased organification of Iodine and lower TH synthesis

what is the main cause of bronchopulmonary dysplasia and it's pathophysicligy? How is it treated?

diagnosed by >28 days of supplemental oxygen therapy in a neonate, who often has signs of respiratory distress present (tachypnea, retractions, etc.). -> *reduced septations of the alveoli* Tx - nutritional support and wean mechanical ventilation if possible

ST elevation of 1.5mm in V2 and V3 with asymptomatic T-wave inversion in V4-V6 indicates what on an ECG?

diagnostic of STEMI in women, but not in men of any age. Would need more prominent ST elevations for leads V2 and V3 (>2.5mm in V2/V3 for men)

UWSA2 - What does Miliary TB look like on CT or radiograph

diffuse nodular infiltrate (millet seed?)

What finding in lower back pain is highly specific for lumbar spinal stenosis?

disappearance of the pain when the patient sits or bends forward (or when using a grocery cart) Diagnose with an MRI

What are the adverse effects of Varenicline which is used for smoking cessation?

disordered sleep & abnormal dreams (or if you're UW *"no increased risk of serious complications"*)

A 27 y/o F patient from Massachusetts presents with several day hx of aching joints, mild fever and malaise. They have stiff, slightly swollen, aching left wrist that hurts on passive extension of fingers, as well as pain on moving bilateral ankles. They also have pustular lesions on distal extrimities. What is the most likely cause? How would you diagnose?

disseminated gonorrhea urine NAAT

What is the treatment for restless leg syndrome?

dopamine agonists (pramipexole) 2nd line - alpha-2-delta calcium channel ligands (gabapentin enacarbil)

Which acne medication causes photosensitivity?

doxycycline

What side effect should you look out for with hydralazine?

drug induced lupus

UWSA1 - How does treatment affect the prognosis of a herpes zoster outbreak?

early treatment decreases risk of long term complications (post herpatic neuralgia) Should be treated w/i 72 hours to help prevent this

septic abortion

emergency following elective/spontaneous abortion or pregnancy with an IUD Sx: fever, Lower abd pain, purulent vaginal discharge, boggy/tender uterus, dilated cervix Tx - suction curretage, broad spectrum abx, blood/endometrial cx, hysterectomy

what are contraindications to the DTaP vaccine?

encephalopathy after a previous dose anyphylaxis to vaccine component

What is Chronic spontaneous urticaria and how is it managed?

episodic urticaria lasting >6 weeks with no apparent trigger, and episodes lasting <24 hours. intensely puritic, round/serpinguous erythematous plaques with possible associated angioedema initial tx - 2nd gen antihistamines avoid aggravating triggers (heat, NSAIDs, etc.) *Spontaneously resolves in 2-5 years*

What does ankle brachial index measure?

evaluates risk for PAD

NBME5 - how often do adults need lipid testing?

every 4-6 years (5 years) (Note this is from google not a question, the question didn't actually say)

UWSA2 - explanatory vs pragmatic study designs?

explanatory - effectiveness in ideal conditions pragmatic - effectiveness in real life clinical practice

What test do you use to confirm eradication of H. pylori after 3 drug combo?

fecal antigen testing 4 weeks after completion of antibiotics Can also do breath test but according to UW it is "more readily available even though breath test is more accurate" I hate this test sometimes

UWSA2 - what is included in hormone replacement therapy?

female hormones *estrogen and progestin*

What is the diagnosis and treatment of ulcerative colitis?

flexible sigmoidoscopy with biopsy Tx: Topical (suppository) Mesalamine (with 5-ASA compounds), taper 4-6 weeks

For someone with insurance from the healthcare marketplace, when do you not have copays?

for preventative services (like screening mammogram) insurance must cover all costs

Supracondylar fracture

fracture of the distal end of the humerus; it is common in children and occurs when the child falls on the outstretched hand with the elbow partially flexed and may injure the median nerve non-displaced - long arm splint and sling displaced - surgical reduction and pinning

How do you monitor respiratory status in someone with developing Guillane-barre syndrome?

frequent measurement of *vital capacity* and negative inspiratory force

NBME5 - In a patient with elevated cholesterol and hypertriglyceridemia what is the best medication to give them?

gemfibrozil (fibrate) best for triglycerides and LDL

What is external validity?

generalizability - the degree to which the investigator can extend or generalize a study's results to other subjects and situations. (for instance a study solely on white patients lacks external validity if you have black/asian/etc patients, or a study only on patients with normal kidney function lacks external validity for ESRD patients)

UWSA1 - How do you diagnose Polycythemia vera?

genetic analysis (Leukocyte Adhesion Phosphatase used to be used but now it lack specificity?... EPO is generally low in these patients and non-diagnostic.)

A 29 y/o woman with prior hx of 3 manic episodes presents with a manic episode. She stopped lithium 5 mo ago because she wanted to start a family/have kids. What treatment should be initiated on her?

haloperidol - apparently 1st gen antipsychotics are fine in pregnant woman and actually work for mania? (not electroconvulsive therapy because it is reserved for people that don't respond to lithium/anticonvulsants)

What do early decelerations mean?

head compression, which can be normal and even part of category 1 fetal tracings (normal) can be seen in late stages of labor or early birth, OR if to many tocolytics are added

What is empiric antibiotic therapy for osteomyelitis in children (in healthy children vs sickle cell children)?

healthy - staph aureus --low likelihood of MRSA - nafcillin/oxacillin OR cefazolin --high likelihood of MRSA - clindamycin OR vancomycin sickle cell - salmonella or staph aureus --as above PLUS --third gen cephalosporin (ceftriaxone, cefotaxime)

A husband and wife are renovating their house when they both develop throbbing frontal headaches. The man comes into the clinic for it, no other symptoms or signs of infection. He says it must be the flu because his wife has it too. What is the next best step?

high flow oxygen from a rebreather mask They have CO poisoning

What is the name of the treatment for mild to moderate plaque psoriasis?

high potentcy corticosteroids -fluocinonide 0.05% cream -augmented betamethazone dipropionate 0.05% cream Topical Vitamin D derivatives

What symptoms are possible with damage to your dominant temporal lobe?

homonymous upper quadrantanopia Wernicke's aphasia - impaired comprehension of spoken/written language. Difficulty expressing thoughts in a meaningful manner

What symptoms are possible with damage to your non-dominant temporal lobe?

homonymous upper quadrantanopia auditory agnosia - impaired perception of complex sounds

Guillain-Barre syndrome treatment

hospitalization is necessary Monitoring respiratory function (vital capacity and negative inspiratory force) mechanical ventilation may be required plasmapheresis---take out plasma and put in new plasma high dose immunoglobulin and steroids used

What exposure has the strongest association with asthma?

house dust mite

What are the main symptoms of Serotonin syndrome?

hypertension, tachycardia; *myoclonic jerking, tremors*; nausea, diarrhea, sweating, hyperthermia; agitation, confusion

What heart disease in a newborn can be caused by maternal gestational diabetes?

hypertrophic cardiomyopathy (hypertrophic interventricular septum) tx - *"Most children recover without surgery, even if they have symptoms"*

What is the most common side effect of glyburide (sulfonylurea)?

hypoglycemia (it's an insulin secretagogue)

When treating someone with vitamin B12 for moderate to severe deficiency, what should you watch out for on their labs?

hypokalemia as newly formed cells uptake K+ rapidly and can lead to deficiency

What is the treatment for tinea pedis?

if mild, topical antifungals (terbinifine, miconazole, clotrimazole) for 1-4 weeks if extensive or with failure of topicals (Or anyone with onchomycosis) need oral agents (terbinifine, itraconazole, fluconazole)

When would you give antibiotics to a patient with an acute COPD exacerbation?

if they have 2 out of the following 3 -increased sputum purulence -increased sputum volume -increased dyspnea

UWSA1 - When can someone return to work after an MI?

if uncomplicated (w/o recurrent angina, arrhythmia or heart failure) and successful revascularization they have a low rick of recurrent MI and can return after 2 weeks medium risk (stable angina, incomplete revascularization, LVEF <40%) should get a stress test first High risk need to be evaluated and stabilized first

What is the management of an auricular hematoma (near the ear)?

immediate incision and drainage with a pressure dressing to prevent permanent deformity (cauliflower ear, abscess)

A 16 y/o has been in the ICU for 3 weeks for quadriplegia after an MVA. He has recent diagnosis of ventillator associated pneumonia and is given an antibiotic. His serum Ca today was 12.6, PO4 is normal, PTH is low, 1,25vitD is low. What is the most likely cause of high Ca?

immobilization

A 27M comes in with HCM signs and symptoms. He has new onset HTN and FH of sudden cardiac death in a paternal uncle. What is the treatment of HCM i a patient recently diagnosed?

implantable cardioverter-defibrillator (because of the FH of SCD) (also BB as medical management for the HTN but idk why they can't just say that)

What is the management of a patient with acute gout? How does it change with renal failure? What if they also had a kidney transplant and are on cyclosporin?

normally indomethacin with ESRD use low dose colchicine (cyclosporin causes delayed clearance of colchicine) with cyclosporin use intra-articular triamcinolone (glucocorticoid) NOTE!!!! (QID 4998 disagrees with this) - this question says don't use colchicine in renal failure because it's CI, instead go with intra-articular GCs)

A 26F is in a stable dose of levothyroxine with no adverse effects. She tales her pills on an empty stomach before meals. Her TSH is 4.8 (normal 0.5-5.0) and her free T4 is 1.3 (normal 0.8-1.8). What advice should you give your patient?

increase levothyroxine dose to get her TSH to "low end of normal" Apparently suppressing TSH decreases the risk of cancers really well?

How do you reduce Type II error?

increase power/increase sample size

What is hyperviscosity syndrome? and what is it a manifestation of?

increased plasma protein content and large size leads to impaired microcirculation nasal or oral bleeding, blurry vision, neurological symptoms (confusion, headache), heart failure, -may occur in high level paraproteinemia (Multiple Myeloma, Waldenstrom's macroglobulinemia, Hyperleukocytosis, AML, severe polycythemia) tx - plasmapheresis

How long should you treat someone for schizophrenia after their first episode?

indefinitely

UWSA1 - What are some of the main causes of Erythema multiforme? (And what does it look like?)

infections - *herpes Simplex virus* and mycoplasma pneumoniae drugs - allopurinal, abx diseases - autoimune disorders, malignancies

What is the Jarisch-Herxheimer reaction? How can it be prevented?

inflammatory reaction that occurs after treatment for syphilis or other spirochetes; fever, chills, malaise, HA, myalgia, rash, pharyngitis It can't be prevented

UWSA1 - What are the main ways enterohemorrhagic e-coli (shiga toxin producing) spreads?

ingestion of contaminated/undercooked beef *Contact with farm animals (like a petting zoo)*

What is the main presentation of and diagnosis of laryngomalacia?

inspiratory stridor worse when supine (on back) but better when prone (on stomach) dx - flexible fiberoptic laryngoscopy

what does the kappa statistic represent?

inter-rater reliability - reflects the extent to which inter-rater agreement represents an improvement on chance agreement alone. 1 = perfect agreement -1 = perfect disagreement

What test should be run before a TNF alpha inhibitor is given to a patient (i.e. infliximab/etanercept for RA)?

interferon gamma release assay to screen for latent TB

UWSA2 - A 29F comes in with constant lower abdominal pain throughout the day only relieved by voiding, needing to pee every 30 minutes for the past 6 months. What is the most likely cause?

interstitial cystitis (bladder pain syndrome) (Dx - bladder pain with no other cause of >6 weeks and normal UA)

1 hour post air enema for treatment of intussiception, a 2 y/o male child experiences sudden onset severe abdominal pain with guarding and rebound tenderness. What is the most likely cause and how would you diagnose it?

intestinal perforation (<1% incidence in air enemas) and presents with signs of peritonitis dx confirmed by *radiographs* of the abdomen (upright and lateral decubitus) to confirm pneumoperitoneum tx - immediate surgical repair

What is the treatment for alopecia areata and what advice should be given about it's prognosis to patients?

intralesional corticosteroids Patients will have a high chance of recurrence even after successful treatment

What is the treatment for keloid scars?

intralesional glucocorticoids (surgical excision used if GCs fail)

UWSA2 - Bipolar depression treatment

lamotrigine (They never give Li as an option) + second gen antipsychotics (quetiapine) (lithium, anti-convulsants, atypical anti-psychotics)

An unconcious patient, with a history of hypotensive episodes during this hospitalization, is started on enteric feeding develops melenic stools. How can you differentiate the cause of melena from ischemic colitis and peptic ulcer/stress ulcer?

ischemic colitis would be hematochizia and bright red blood, not melena, while ulcers would be melenic stools

When can you give anticoagulation after tPA administration in a stroke patient? What other medical management should be considered

it is contraindicated within 24 hours intense BP control also necessary. Keep BP <180/105 to decrease the risk of hemorrhagic transformation

What anesthesia is used for awake intubation (used for difficult airways so that they remain patent)?

ketmine

What is the best treatment for recurrent pancreatitis due to high triglycerides?

long term fibrate therapy Note: Bariatric surgery is only indicated for patients ?40 BMI or >35 with certain comorbidities (DM, HTN, OSA)

NBME5 - What are jugular cannon waves?

large amplitude waves that can present in certain cardiac arrhythmias (Complete heart block or ventricular tachycardia)

What are the cause of late decelerations and recurrent late decelerations in labor?

late - uteroplacental insufficiency recurrent = >50% of contractions have late decels -> impending fetal hypoxemia and acidemia If you have late you don't necessarily need to worry but recurrent is cause for concern. Excessively frequent contractions can cause this, so stop uterotonic meds

UWSA2 - ddx active vs latent TB via CXR

latent - fibronodular disease, calcified granulomas active - cavitations, hilar lymphadenopathy, infiltrate, pleural effusions

What is the benefit of using NPH (intermediate insulin) compared to metformin and sitagliptin?

less chance of hypoglycemia

If a 76 year old woman is on a statin and she develops muscle pain around her shoulders and pelvic gurdle, but has a normal CK, what is the next best step in her management?

measure ESR because she has polymyalgia rheumatica

There are 3 classifications of asthma exacerbations in the ED: mild to moderate, moderate to sever, and impending or actual respiratory distress. What Peak expiratory flow (PEF) or FEV1 values correlate to each?

mild to mod - 40-69% PEF or FEV1 -inhaled SABA +/- PO corticosteroids mod to sev - <40% PEF or FEV1 -Tx - high dose SABA + ipratropium -PO or IV corticosteroids resp dist - not defined other than respiratory distress... -Tx - high dose SABA + ipratropium -IV corticosteroids + Magnesium -+/- IV terbutaline or epinepherine *-intubation as a last resort (or if decompensating?)* ALL - Oxygen if pO2 <90%

What is the advice for breastfeeding and antidepressant use?

most antideressants are secreted in breast milk but are fine to use SSRI's are still first line Buproprion is not recommended initially and may have some risk of seizures

What is the latest a child can start walking before it becomes suspicious?

normally walk between 9-16 months

Is a confidence interval of 95% wider or narrower than a CI of 99%?

narrower So what you think is wrong. the CI is similar to the distance from the mean. so 95% means it catches 95% of thing from the mean, while 99% catches 99% of things far from from the mean. So since 99% catches more things it is wider

What is the difference between a near-miss event and a sentinel event?

near miss - a mistake that does not reach the patient sentinel event - unanticipated event that results in death or serious physical/psychological damage to the patient

What is a positive straight leg raise test at 60 degrees characteristic of?

nerve root inflammation

What are the brain death clinical criteria?

neuro exam showing irreversible absence of cerebral and brainstem reflexes including pupillary, oculocephallic, oculovestibular (caloric), corneal, gag, sucking, swallowing, extensor posturing

Physiologic gynecomastia

new onset bilateral gynecomastia with otherwise normal exam. Firm and circumscribed behind the nipple (diffuse enlargement is pseudogynecomastia from being fat) normal response to increased estrogen levels in ... • newborn • puberty • elderly (because of decrease testosterone from testicles?)

What is the pathophysiology of Psychogenic Non-Epileptic Seizures (PNES), and how can they be differentiated from other seizures?

no abnormal activity in the brains but appear to have seizure activity and considered a conversion disorder; don't need to be on seizure meds but need psych counseling to determine cause; remember that these pts. aren't practicing having seizures May be identified by: forceful eye closure, side-to-side head or body movements, rapid alerting and reorienting, and memory recall of the event (although these are not always true indicators) video-electroencephalogram is gold standard test - shows no abnormal activity (tyically occur in front of a witness and they typically model and friend/relative with hx of seizures)

How do you prevent recurrent esophageal varices in an alcoholic with liver cirrhosis who had esophageal varices and was treated with band ligation?

non selective beta blocker (propranolol, nadolol, etc)

concurrent acute otits media and purulent conjunctivits are normally caused by what pathogen?

non-typeable Haemophilis influenzae (strep pneumo and moraxella catarrhallis are the other two major causes of AOM) tx - 1st line amoxicillin, 2nd amoxicillin-clavulaunate, penicillin allergic - clindamycin or azithromycin

2 weeks after birth a patient comes in with stress urinary incontinence (leakage of urine with valsalva). What is the next best step in management?

observation and reassurance. While it does increase risk of chronic SUI, most cases of SUI after birth resolve in <6 weeks postpartum tx - if it turns into chronic can use continence pessary or midurethral sling

If a mother has a (+) GBS screen before birth and is only given ampicillin IV 2 hours before giving birth to a healthy looking child, what should be done next?

observation only (In order for abx tx to be considered adequate it must be given >4h before deliver. However, even if it is inadequate you don't treat baby just observe to see if they develop symptoms first because most will) (key word, most...)

transient tachypnea of the newborn

respiratory condition that results from incomplete reabsorption of fetal lung fluid in full term newborns, usually disappears within 24 to 48 hours (most common cause of respiratory distress in newborns)

What is Length time bias and how is it different from lead time bias?

occurs when the survival benefits of a screening test are overstated due to the detection of a disproportionate number of slowly progressive benign cases lead time - Bias introduced when screening detects a disease earlier and thus lengthens the time from diagnosis to death. Note - length time detects benign cases with lead time detects same cases earlier

UWSA1 - when should you initiate oral nutrition to a patient who has acute pancreatitis?

once their appetite returns (low fat, soft diet) (Old recomendations were bowel rest for 1-2 dys to reduce exocrine function but they changed it apparently? Take with a grain of salt this is a SA who knows when it was updated)

what is the treatment for rosacea?

only erythema/telangiectasias - mild cleansers and sunscreen and avoidance of triggering factors *Papulopustular - Metronidazole (metrogel) topical* (though azelaic acid, topical clindamycin and benzoyl peroxide are also useful)

When should you use alteplase in a STEMI patient?

only if PCI (percutaneous coronary intervention) cannot be started within 2 hours of 1st medical contact

How do you treat an acute sickle cell crisis in an 8 month old child who has been taking over the counter NSAIDs and tylenol to no relief?

oxycodone Hydroxyurea is not short term, tramadol and codeine are not safe if <12 y/o

UWSA2 - What is the treatment of spontaneous pneumomediastinum (not pneumothrorax)?

oxygen and supportive care (usually self-resolves) - only use a chest tube if they have concomitant pneumothorax

What is the manifestation of sporotrichosis?

painless papule at site of innoculation -> drains nonpurulent, odorless fluid ->similar lesions appear along proximal lympahtic chain NOTE: If someone comes in with a branch that fell on their neck and this presentation, it ay just describe a "cluster of lesions" but it's still this... ddx - cervicofacial actinomycosis - abscesses, yellow drainiage with granules ddx - histoplasmosis - systematic symptoms and clinically ill

How do you differentiate between someone who has chordae tendonae rupture vs papillary muscle rupture on someone with acute mitral regurgitation with decompensation?

papillary rupture apparently only occurs in MI, while Chordae tendonae rupture can be anytime by itself (but is associated with Marfans, Ehler-Danlos and connective tissue diseases)

What is the initial tx for ADHD for children 3-5 (preschool age)?

parent-Child behavioral therapy Then amphetamines

first degree relatives

parents, *siblings*, and children (share about 50% of genes)

What bone is broken in spondylolisthesis

pars interarticularis

What is the time range of postpartum blues?

peaks in 5 days, should be gone by 2 weeks

A patient with CAD comes in for an ecercise stress test and you see 1-mm ST depression in leads I, aVL and V5-6 with minimal exertion. You prescribe aspirin, atorvastatin and metoprolol and counsel on smoking cessation. What is the next best step in management?

percutaneous coronary angiogram? Apparently that ST depression at minimal excercise is a "high risk feature" so you need PCA to check for severe stenosis/calcification of coronary aa.

A patient with a kidney stone in the proximal ureter develops significant hydronephrosis. What is the next best step in management?

percutaneous nephrostomy (retrograde ureteral stent) only use shock wave lithotripsy in uncomplicated stones

Diffuse 1-mm ST elevations with PR depression indicated what ECG finding?

pericarditis

What is the presentation and diagnosis of Gilbert Syndrome?

periods of intermittent jaundice by mild unconjugated bilirubinemia (<3 unconj) without evidence of hemolysis (normal Hg and reticulocytes) triggered by stress - fasting, dehydration, febrile illness, vigorous excercise, menstruation, surgery dx - nothing, I guess it's just clinical

A patient has a presentation similar to acute mesenteric ischemia but there is no blood in the stool. What other signs could indicate it?

periumbilical pain, out of proportion to exam *anion gap lactic acidosis* *elevated amylase*

What is the treatment for urinary/urethral/uterine (pelvic organ prolapse in general) prolapse into the vagina?

pessary placement

what diabetes drug can cause pulmonary edema?

pioglitazone (PPAR-gamma agonists a.k.a. thiazolidinediones)

painless third trimester vaginal bleeding often means what?

placenta previa

A woman comes in 35 weeks pregnant with an infant that has had fetal growth restriction (low birth size) documented since 20w. She has new onset reversed end diastolic flow in the umbilical artery. What does this mean and what should be done?

placental insufficiency with impending fetal hypoxia (indicated by absent or reversed en-ddiastolic flow of umbilical aa.) Admit to hospital and induce labor

UWSA1 - After a kidney transplant the patient begins to develop kidney failure again (6 months later) and on biosy you find renal tubular damage with prominent basiphillic intranuclear inclusions. What is the most likely cause?

polyoma virus infection (BK virus)

If you see popcorn calcification on lung radiography, what does it most likely indicate? What about eccentric calcification?

popcorn - Lung hamartoma (benign) eccentric calcification is malignancy

net clinical benefit

possible benefit minus possible harm

What is the statistical equation for power?

power = 1-B B = type II error (probability of incorrectly failing to reject a false null hypothesis)

what is the treatment for systemic lupus erythematosus?

prednisone and hydroxychloroquine (for severe cases cyclophosphamide) (for pts with severe organ damage and response to prednisone alone can add methotrexate)

what do lymphocytes look like in infectious mononucleosis?

predominant cytoplasm and irregular nucleus

NBME5 - When does a breast cancer diagnosis actually indicate increased risk for family members with BRCA status unknown?

premenopausal diagnosis

what is the treatment of preseptal cellulitis vs orbital cellulitis?

preseptal - oral abx (broad spectrum) orbital - IV abx +/- surgery (ceftriaxone or ampicillin-sulfbactam... or Vanco if they have MRSA. Most common bugs are S aureus, S pneumo and H. influenzae)

What can chlamydia in pregnancy cause?

preterm prelabor rupture of membranes preterm labor postpartum endometritis

What is complex regional pain syndrome and how does it normally present?

previously known as reflex sympathetic dystrophy it usually occurs after an injury and presents with *pain out of proportion to injury, temperature change, edema, abnormal skin color* Likely from an injury causing increased sensitivity to sympathetic nerves and can cause burning pain to light touch (*allodynia*) In severe cases it can lead to restriction of movement and bone demineralization dx - autonomic testing or MRI tx - nerve block

A 31F has antiphospholipid syndrome and history of blood clots, for hich she is on anticoagulation. She wants to be on birth control. What is the best for her?

progestin containing IUD (Not copper because copper increases menstrual bleeding and dysmenorrhea, which is CI if she is on anticoagulation)

When should you use Cabergolin (dopamine agonist) vs. trans-sphenoidal resection for a pituitary tumor with high prolactin?

prolactinoma's have prolactin >200. If it is less than 200 it is probably 'non-functional' and should be resected Note: Also, 'non-functioning' generally emit only alpha subunit of FSH or LH rather than alpha and beta

If a fully vaccinated cat bites someone on the hand and it punctures the skin, what is the next best step in management?

prophylactic amoxicillin-clavulaunate (1st line for mammalian bites) azithromycin is for cat scrath disease

What kind of bias is a funnel plot useful in determining?

publication bias (apparently also small sample size and dots near the top are large samples/powerful while towards the base are small samples/less powerful)

How do you prevent the spread of disseminated herpes zoster (lesions outside primary or immediately adjacent dermatomes) in a patient with active lesions?

put them on contact *and airborne* precautions tx - hospitalize and IV acyclovir

What is the presentation of Septic pelvic thrombophlebitis and the major risk factor? How is it treated?

px - fever (unresponsive to abx) and bilateral lower abdominal pain after a C section with no signs of other infections (dx of exclusion) thrombosis in the veins of the pelvis due to infection. The *most important risk factor is caesarean section*, especially if complicated by infection. In fact, almost 90% of cases occur after caesarean delivery. The overall incidence is low, affecting only approximately 1 in every 2000 pregnancies. The mainstays are anticoagulation with heparin and broad-spectrum antibiotics (including coverage for anaerobes and common Enterobacteriaceae). Within 48-72 hours of initiation of heparin therapy, fever should resolve. Treatment usually is empirically continued for 7-10 days, although the optimal duration of therapy is not well defined.

What is a Nested study design?

retrospective observational study in which subsets of controls are matched to cases and analyzed for variables of interests

What is Photoaging and what is the main treatment of the disease?

rather than fine wrinkles of normal aging, causes deep, coarse wrinkles and rough skin. Often marked with actinic keratosis, telangiectasias and brown ("liver") spots. Due to UV light overexposure May be exacerbated by tobacco use tx - tretinoin (all-trans-retinoic acid)

If you have a neonate with neonatal polycythemia (Hg >65%) through heel prick method what is the next step you should do in management?

recheck hematocrit from peripheral blood (there can be significant variability in heel prick measurements)

A patient with parkinson's is on amantidine and carvidopa/levodopa. You recently increased the dose of C/L and his symptoms improved greatly. However now he develops psychotic symptoms. What should you do?

reduce dosage and taper off amantidine apparently C/L decreasing his symptoms is too important so keep that and get rid of amantidine (increases dopamine release) that may be increasing dopamine levels

A 43 y/o M has recurrent back injuries from heavy lifting. They normally resolve with just rest and NSAIDs. He asks how he can best prevent these recurrent injuries. What should you say.

regular excercise (apparently lumbar support braces lead to more injuries from 'deconditioning and lack of movement' f*** UW)

If a mother refuses vaccines for her child what should you do in subsequent visits in regards to vaccine information?

reopen discussion at each subsequent visit

What is Sensitivity Analysis?

repeating primary analysis calculations after modifying certain criteria or variable ranges goal is to detect whether such modifications significantly affect the results initially obtained (if they are similar to initial results it demonstrates *robustness* of their results)

What shape are the crystals in pseudogout?

rhomboid (this one was a brain fart)

What is the treatment for prevention of meningococcal meningitis in exposed persons?

rifampin 600mg oral 2x/day for 2 days --INTERACTS WITH OCPs SO USE CIPRO INSTEAD Ciprofloxacin 500mg single oral dose Ceftriaxone 250mg single dose

Post intensive care syndrome

risks - After ICU care, ARDS or prolonged ventillation patho - CNS hypoxia, microinflammation, metabolic disruption Features - >50% major depression, PTSD -Dec attention/memory, executive function and processing speed - >50% dec mobility and independence management - early therapy participation (PT/OT) - multidisciplinary post-ICU clinic, pharmacotherapy for psych disorders - prognosis - most require additional home care & never return to work

A patient with Lewey Body dementia begins treatment with donepizil, carbidopa-levodopa and risperidone then develops worsening symptoms, parkinsonism and autonomic dysfunction (psotural hypotension), what is the most likely cause?

risperidone - patients with DLB are extremely sensitive to antipsychotics (quetiapine might be better because it is lower potency) 1st gen antipsychotics should be avoided entirely

How would you describe a poison ivy rash?

scattered areas of erythematous papules with several overlying vesicles filled with clear fluid. Many linear erythematous lesions some of which have vesicles oozing serous fluid

UWSA1 - A 53F comes in with fatigue, thirst and polyuria. She has been treated for depression and hospitalized for it multiple times. She also has a history of cocaine use disorder and a FH of bipolar. Her HbA1c is elevated to 7.5%. What drug is most likely causing this?

second gen antipsychotic (apparently she's getting a 2nd gen antipsychotic because of treatment resistant depression and it's causing DM/metabolic effects)

Immune Reconstitution Inflammatory Syndrome (IRIS)

severe exacerbation of disease symptoms due to an over robust immune response in previously immunocompromised patients with some disease (most commonly seen in HIV patients started on HAART) Tx - continue current management and treat symptoms (Note: This Q was about worsening TB/pneumonia after 6 weeks of HAART for HIV, but others may be for cancers, etc.)

Which lung cancer causes SIADH?

small cell

If someone has an abdominal aortic aneurysm and they still smoke 1-2 PPD, drink 3-4 beers per night, have uncontrolled diabetes, uncontrolled hypertension and uncontrolled hyperlipidemia, what is the best way to reduce the risk of the AAA expansion?

smoking cessation is the most important modifiable risk factor (NOT BP CONTROL!!!)

NBME5 - A 92F is about to die in the hospital and her family previously requested last admission that she be kept alive at all costs. What should you do?

speak with the family about her prognosis and the code status (Re-evaluate it at every hospitalization... I thought it was the same one but I missed that part oof)

How would you test a patient with suspected TB (from cavitary lesion on CXR) for active TB infection?

sputum for acid fast bacilli and culture

A patient comes in with new unexplained symptoms for heart failure over the past 6 months. No family history, no causes seemingly. You see a dilated LV on echo and EF of 38%. What is the next best step in management?

stress test or coronary angiography (to possibly figure out a cause??) -Apparently don't need a BNP because that's only for acute HF to rule out other causes of dyspnea... even though you'd definitely still order it

UWSA2 - If a baby has hypospadias and bilaterally descended testes, what is the next best step?

surgical correction at 6 months (only get a pelvic US if they have cryptorchidism)

UWSA2 - What advice can you give a 25F nulligravida for how to prevent ovarian cancer in the future? (She has FH but negative for BRCA mutations)

take combined OCPs (because they reduce ovulation

How do you differentiate between a patellar fracture and a patellar tendon tear?

tendon tear is more acute while fracture is chronic. Also fractures are uncommon without precipitating factors like extensive glucocorticoid use in the last 6 months

What are the main initial symptoms of Digoxin toxicity?

the GI ones

UWSA1 - If someone has atrial flutter what are the next best steps?

thrombembolism risk increases immensely with cardioversion, SO give riveroxaban first and keep them on it for >3 weeks prior to cardioversion

If you have a patient with heart disease and new depression diagnosis, what should you say to them if they're concerned about medication interactions? (note: no med names are given)

untreated depression leads to increased morbidity and mortality from heart disease SSRI's generally do well with meds for CVD with no major interactions

When should you give oseltamivir for the flu?

to anyone who is presenting within 48h of symptom onset, OR anyone high risk (see image) regardless of symptom onset

If you have a patient with atopic dermatitis on the face, eyelids or flexural areas that topical emolients and oral antihistamines are not working for, how would you treat it?

topical calcineurin inhibitors (topical triamcinolone) are good because topical GCs are contraindicated in those regions

What is the treatment for tinea versicolor?

topical miconazole (ketoconazole, itraconizole, fluconazole), selenium sulfide (selsun)

Going against everything UW has ever said, when you have a recent Graves' diagnosis and put the patient on propranolol and methimazole, what should you measure in the 2 month follow-up to check for treatment response?

total T3 and free T4... you guessed it of course it isn't TSH like it always has been Apparently, TSH remains suppressed for 2-3 months so you can't use it reliably until after that. On another note, F*** UW!!!

how much weight loss is expected in infants after birth?

up to 10% of birth weight in the first week (after 5 days breast milk production increases in mother then newborn weight should surpass birth weight in 2 weeks)

How long can an effusion (tympanic membrane retracted and yellow color behind membrane) last in a child who was treated for acute otitis media (amoxicillin for 10 days)?

up to 3 months after treatment ends -> watchful waiting

If you suspect peripheral neuropathy from DM (numbness + pins/needles), what is the next best test to run?

tuning fork test? what the actual hell? to check for concurrent loss of vibratory sense)

What is the presentation of Thrombotic thrombocytopenic purpura and MOA of the treatment?

tx MOA - increasing amount and activity of plasma metalloproteinase? Whaaaaaaaaaaaaaaat? -plasma exchange reduces autoantibody levels against ADAMTS13 and replenishes it from donor serum

What is the treatment for CML? (with a bcr-abl translocation)

tyrosine kinase inhibitor

UWSA1 - how do you treat vaginal candidiasis during pregnancy?

vaginal clotrimazole

A 5 year old boy is in office to get his second MMR vaccine today. In his previous one he had a fever of 103F. He is with his mother who is 38 weeks pregnant. He has also had a runny nose and cought for the past 3 days. Should he receive the vaccine today? why or why not?

yes, none of those are contraindications. CI - in pregnant women, with prior anaphylaxis (to prior vaccine, neomycin or gelatin) or with immunodeficiency

How do you diagnose posterior urethral valves?

voiding cystourethrogram

A 7 year old girl has a concussion playing soccer and comes to you. You initially tell her to wait for 48hours then return to light play and increase from there. 7 days later she returns to medium play and develops nausea and diziness. What is the best recommendation for her now?

wait >24 hours then return to light play again If symptoms worsen at any level of exertion a 24 hour rest period should be observed before returning to the previous step in the protocol that didn't exacerbate symptoms

How do you diagnose scabies?

when burrows are seen or with *skin scraping -> glass slide + microscope* tx - 5% permethrin cream

When would you not do a fiberoptic bronchoscopy on a patient with a 2.2 cm spiculated and calcified new onset lung nodule before you surgically esxcise it?

when it is peripherally located so the bronch wouldn't be practical

Right Bundle Branch Block ECG

widened QRS (>0.12s); double peaked QRS in V1 (best to look in V1 and V2); T wave can look abnormal - inverted (does not indicate ischemia)

NBME5 - When do you give alteplase in a stroke?

within 3-4.5 hours

verification bias

workup bias, a type of measurement bias that occurs when a study uses gold standard testing selectively in order to confirm a positive (or negative) result of preliminary testing; this can result in overestimates (or underestimates) of sensitivity (or specificity) overcome by randomly selecting non-selected patients and performing the gold-standard test on them as well

What lab values are seen in Pseudohypoparathyroidism and what is the cause?

↓ Ca2+ with normal to ↑ PTH; end-organ resistance to PTH (Also see hyperphosphatemia)

Multifocal Atrial Tachycardia

∙Patient with severe pulmonary disease such as *COPD.* ∙*Variable P-wave* morphology and variable PR and RR intervals. ∙*At least 3 different P-wave*morphologies are required to make accurate diagnosis. -*ANOTHER CAUSE IS HYPOKALEMIA OR HYPOMAGNESEMIA!!!!!!!* (specifically not hyponatremia so if they have 120 Na and 3.1 K don't replace Na, only K because f*** their sodium apparently) tx - *treat underlying condition* and AV node blockade (verapimil) if persistent


Kaugnay na mga set ng pag-aaral

Science - Skeletal System (What does the skeleton do? Pages 36-40)

View Set

Photosynthesis & Cellular Respiration - Quiz

View Set

Chapter 4 Basics of Behavior Change and Health Psychology

View Set

Introduction to Law: Alternative Dispute Resolution

View Set

AP STATS multiple choice questions

View Set

Licensing of Agents, Brokers, Limited representatives and adjusters

View Set

Pharmacology - Chapter 56 - Dermatologic Drugs

View Set

Small Business Management Exam #3

View Set